High-yield Acid-base (High-Yield Acid-Base) (High-Yield Series) [2 ed.] 0781796555, 9780781796552

The goal of this book is to provide a bridge between the acid-base physiology taught in the classroom and the evaluation

229 62 3MB

English Pages 128 [102] Year 2007

Report DMCA / Copyright

DOWNLOAD PDF FILE

Table of contents :
High-Yield Acid–Base, SECOND EDITION
Half Title Page
Title Page
Copyright
Dedication
Preface
Acknowledgments
Contents
Chapter 1: Acid–Base Physiology
Definitions
Overview of Acid–Base Physiology
INTRACELLULAR ACID AND BASE PRODUCTION
THE BUFFER SYSTEM: TRANSPORTING ACID AND MITIGATING pH CHANGES
THE RESPIRATORY COMPONENT: ELIMINATION OF CO2 BY THE LUNGS
THE METABOLIC COMPONENT: ELIMINATION OF NONVOLATILE ACIDS
THE INTERFACE BETWEEN THE RESPIRATORY AND METABOLIC COMPONENTS
Common Laboratory Tests in Acid–Base Evaluation
THE ARTERIAL BLOOD GAS (ABG)
THE SERUM ANION GAP
THE SERUM OSMOLAL GAP
THE URINE pH
THE URINE ANION GAP AND OSMOLAL GAP
THE BASE EXCESS
Chapter 2: The Primary Acid–Base Disorders
Metabolic Acidosis
RENAL RESPONSE TO A METABOLIC ACIDOSIS
RESPIRATORY RESPONSE TO A METABOLIC ACIDOSIS
TYPES OF METABOLIC ACIDOSES
Metabolic Alkalosis
RENAL RESPONSE TO A METABOLIC ALKALOSIS
RESPIRATORY RESPONSE TO METABOLIC ALKALOSIS
Respiratory Acidosis
THE BUFFER RESPONSE TO RESPIRATORY ACIDOSIS
THE RENAL RESPONSE TO RESPIRATORY ACIDOSIS
Respiratory Alkalosis
THE BUFFER RESPONSE TO RESPIRATORY ALKALOSIS
THE RENAL RESPONSE TO RESPIRATORY ALKALOSIS
Chapter 3: The Basics of Compensation
Understanding Compensation
Predicting the Degree of Compensation
IMPORTANT CAVEAT
METABOLIC ACIDOSIS
METABOLIC ALKALOSIS
RESPIRATORY ACIDOSIS
RESPIRATORY ALKALOSIS
The Formulas Needed to Evaluate Most Acid–Base Disorders
A Word about Confidence Limits
Chapter 4: Mixed Acid–Base Disorders
The Basics of Mixed Disorders
Patterns of Mixed Acid–Base Disorders
Identifying Mixed Metabolic Acid–Base Disorders
TYPES OF MIXED METABOLIC DISORDERS
THE “DELTA–DELTA” CALCULATION
Identifying Triple Acid–Base Disorders
Chapter 5: A Practical Approach to the Arterial Blood Gas
Chapter 6: Differential Diagnosis
Metabolic Acidosis
THE ELEVATED-GAP METABOLIC ACIDOSES
THE NORMAL-GAP METABOLIC ACIDOSES
Metabolic Alkalosis
“SALINE-RESPONSIVE” (CHLORIDE-RESPONSIVE) METABOLIC ALKALOSES
THE “SALINE–NON-RESPONSIVE” METABOLIC ALKALOSES
HYPOKALEMIA AND METABOLIC ALKALOSIS
Respiratory Acidosis
Respiratory Alkalosis
Chapter 7: Tutorial
Case 1
Case 2
Case 3
Case 4a
Case 4b
Case 4c
Case 4d
Case 5
Case 6a
Case 6b
Case 7
Case 8
Case 9
Case 10
Case 11
Case 12
Appendix: List of Abbreviations
Suggested Reading
Index
Recommend Papers

High-yield Acid-base (High-Yield Acid-Base) (High-Yield Series) [2 ed.]
 0781796555, 9780781796552

  • 0 0 0
  • Like this paper and download? You can publish your own PDF file online for free in a few minutes! Sign Up
File loading please wait...
Citation preview

Longenecker_FM.qxd

10/20/06

10:37 AM

Page i

High-Yield Acid–Base SECOND EDITION

Longenecker_FM.qxd

10/20/06

10:37 AM

Page ii

Longenecker_FM.qxd

10/20/06

10:37 AM

Page iii

High-Yield Acid–Base SECOND EDITION

J. Craig Longenecker, M.D., M.P.H. Kuwait University Faculty of Medicine Revised by: Todd R. Nelson, M.D., M.S. University of Iowa Department of Anesthesiology

Longenecker_FM.qxd

10/25/06

3:59 PM

Page iv

Acquisitions Editor: Donna M. Balado Managing Editor: Tenille A. Sands Marketing Manager: Jennifer Kuklinski Production Editor: Paula C. Williams Designer: Terry Mallon Compositor: International Typesetting and Composition Printer: Data Reproductions Corporation Copyright © 2007 Lippincott Williams & Wilkins 351 West Camden Street Baltimore, MD 21201 530 Walnut Street Philadelphia, PA 19106 All rights reserved. This book is protected by copyright. No part of this book may be reproduced in any form or by any means, including photocopying, or utilized by any information storage and retrieval system without written permission from the copyright owner. The publisher is not responsible (as a matter of product liability, negligence, or otherwise) for any injury resulting from any material contained herein. This publication contains information relating to general principles of medical care that should not be construed as specific instructions for individual patients. Manufacturers’ product information and package inserts should be reviewed for current information, including contraindications, dosages, and precautions. Printed in the United States of America First Edition, 1998 Library of Congress Cataloging-in-Publication Data Longenecker, J. Craig. High-yield acid-base / J. Craig Longenecker ; revised by Todd R. Nelson. — 2nd ed. p. ; cm. Includes bibliographical references. ISBN 0-7817-9655-5 (alk. paper) 1. Acid-base imbalances. 2. Acid-base equilibrium. I. Nelson, Todd R. II. Title. [DNLM: 1. Acid-Base Imbalance. 2. Acid-Base Equilibrium. WD 220 L852h 2007] RC630.L66 2007 616.3'992—dc22 2006035066 The publishers have made every effort to trace the copyright holders for borrowed material. If they have inadvertently overlooked any, they will be pleased to make the necessary arrangements at the first opportunity. To purchase additional copies of this book, call our customer service department at (800) 638-3030 or fax orders to (301) 223-2320. International customers should call (301) 223-2300. Visit Lippincott Williams & Wilkins on the Internet: http://www.LWW.com. Lippincott Williams & Wilkins customer service representatives are available from 8:30 am to 6:00 pm, EST. 06 07 08 09 10 1 2 3 4 5 6 7 8 9 10

Longenecker_FM.qxd

10/20/06

10:37 AM

Page v

Dedicated with love to my wife, Ruth. What joy there is in the journey! –Ecclesiastes 4:9–10

Longenecker_FM.qxd

10/20/06

10:37 AM

Page vi

Longenecker_FM.qxd

10/20/06

10:37 AM

Page vii

Preface One may well ask why we need another book on acid–base disorders. After all, medical libraries and bookstores abound with textbooks presenting the pathophysiology of acid–base disorders and their relationship to medical illness. This book is not intended to replace those textbooks or to provide a shortcut around the many hours of study required to understand acid–base syndromes as they relate to the broader context of medical illness. Rather, the goal of this book is to provide a bridge between the acid–base physiology taught in the classroom and the evaluation of a patient on the wards. When I was a medical student, one of the hurdles I faced in learning acid–base pathophysiology was the multitude of perspectives from which it was taught, each with its own unique list of equations and nomograms. As I dutifully waded through this morass, I found that the confusion I faced was shared by others as well. After encountering real patients with acid–base disorders, I began to see that all of the equations, mechanisms, and nomograms that I had been taught could be condensed into a relatively straightforward method, which is the basis of this book. It is a practical method with which to evaluate acid–base problems that enables the reader to establish a standard approach to acid–base pathophysiology as he or she begins to apply it to clinical situations. This method certainly is not the only way to approach acid–base problems, but it is coherent and trimmed down to the basics of the acid–base theory. Chapters 1 through 4 of this book provide the reader with a concise, organized review of the basics of acid–base theory. Chapter 5, which is the heart of this manual, introduces the reader to the step-by-step approach to acid–base problems. Chapter 6 follows with a rudimentary framework for differential diagnosis and a brief discussion of the most important acid–base disorders. Given its limited scope, this chapter should not be used to generate complete differential diagnoses or to guide diagnostic paths or therapies. Perhaps the most useful chapter is Chapter 7, in which the reader is led carefully through 12 cases that illustrate the step-by-step approach to simple and mixed acid–base disorders. The reader can also find additional resources at the back of the book. There is a list of abbreviations to familiarize the reader with the common terms associated with acid–base pathophysiology. There is also a substantial list of suggested readings, with brief comments about the strengths and weaknesses of each book or article. This list can be used independently or in conjunction with the text, as each entry is referenced within the text. These references alert the reader that there are additional resources provided for specific topics. It is my hope that this book will ultimately benefit the patients for whom we as health professionals care. If it enables the reader to develop a practical and reasoned approach to the patient with an acid–base disorder, I will have accomplished my goal.

vii

Longenecker_FM.qxd

10/20/06

10:37 AM

Page viii

Longenecker_FM.qxd

10/20/06

10:37 AM

Page ix

Acknowledgments While I have many professors and colleagues to thank for teaching me acid–base concepts, I especially want to thank the three Assistant Chiefs of Service on the Osler House Staff at Johns Hopkins Hospital during my residency training: Drs. Thomas Disalvo, Shawn Stinson, and Landon King. I greatly appreciate their extraordinary skill, professionalism, and care in teaching and mentoring me and the other residents serving on the Barker Firm. I also want to thank my wife, Ruth, for supporting me through the seemingly endless hours it took to prepare this book for publication. It has truly been a joint effort.

ix

Longenecker_FM.qxd

10/20/06

10:37 AM

Page x

Longenecker_FM.qxd

10/20/06

10:37 AM

Page xi

Frontmatter Title Contents Preface . . . . . . . . . . . . . . . . . . . . . . . . . . . . . . . . . . . . . . . . . . . . . . . . . . . . . . . . . . . . . . . . . .vii Acknowledgments . . . . . . . . . . . . . . . . . . . . . . . . . . . . . . . . . . . . . . . . . . . . . . . . . . . . . . . . .ix

1 Acid–Base Physiology

. . . . . . . . . . . . . . . . . . . . . . . . . . . . . . . . . . . . . . . . . . . . . . . . . . . .01

I. Definitions . . . . . . . . . . . . . . . . . . . . . . . . . . . . . . . . . . . . . . . . . . . . . . . . . . . . . . . . . .01 II. Overview of Acid–Base Physiology . . . . . . . . . . . . . . . . . . . . . . . . . . . . . . . . . . . . . . .01 III. Common Laboratory Tests in Acid–Base Evaluation . . . . . . . . . . . . . . . . . . . . . . . . . .08

2 The Primary Acid–Base Disorders I. II. III. IV.

. . . . . . . . . . . . . . . . . . . . . . . . . . . . . . . . . . . . . . . . . .12

Metabolic Acidosis . . . . . . . . . . . . . . . . . . . . . . . . . . . . . . . . . . . . . . . . . . . . . . . . . . . .12 Metabolic Alkalosis . . . . . . . . . . . . . . . . . . . . . . . . . . . . . . . . . . . . . . . . . . . . . . . . . . .15 Respiratory Acidosis . . . . . . . . . . . . . . . . . . . . . . . . . . . . . . . . . . . . . . . . . . . . . . . . . . .17 Respiratory Alkalosis . . . . . . . . . . . . . . . . . . . . . . . . . . . . . . . . . . . . . . . . . . . . . . . . . .19

3 The Basics of Compensation . . . . . . . . . . . . . . . . . . . . . . . . . . . . . . . . . . . . . . . . . . . . . . . .20 I. II. III. IV.

Understanding Compensation . . . . . . . . . . . . . . . . . . . . . . . . . . . . . . . . . . . . . . . . . . .20 Predicting the Degree of Compensation . . . . . . . . . . . . . . . . . . . . . . . . . . . . . . . . . . .21 The Formulas Needed to Evaluate Most Acid–Base Disorders . . . . . . . . . . . . . . . . . .26 A Word About Confidence Limits . . . . . . . . . . . . . . . . . . . . . . . . . . . . . . . . . . . . . . . .26

4 Mixed Acid–Base Disorders . . . . . . . . . . . . . . . . . . . . . . . . . . . . . . . . . . . . . . . . . . . . . . . .28 I. II. III. IV.

The Basics of Mixed Disorders . . . . . . . . . . . . . . . . . . . . . . . . . . . . . . . . . . . . . . . . . . .28 Patterns of Mixed Acid–Base Disorders . . . . . . . . . . . . . . . . . . . . . . . . . . . . . . . . . . . .29 Identifying Mixed Metabolic Acid–Base Disorders . . . . . . . . . . . . . . . . . . . . . . . . . . .31 Identifying Triple Acid–Base Disorders . . . . . . . . . . . . . . . . . . . . . . . . . . . . . . . . . . . .33

5 A Practical Approach to the Arterial Blood Gas 6 Differential Diagnosis I. II. III. IV.

. . . . . . . . . . . . . . . . . . . . . . . . . . . . . . .35

. . . . . . . . . . . . . . . . . . . . . . . . . . . . . . . . . . . . . . . . . . . . . . . . . . . .40

Metabolic Acidosis . . . . . . . . . . . . . . . . . . . . . . . . . . . . . . . . . . . . . . . . . . . . . . . . . . . .40 Metabolic Alkalosis . . . . . . . . . . . . . . . . . . . . . . . . . . . . . . . . . . . . . . . . . . . . . . . . . . .46 Respiratory Acidosis . . . . . . . . . . . . . . . . . . . . . . . . . . . . . . . . . . . . . . . . . . . . . . . . . . .49 Respiratory Alkalosis . . . . . . . . . . . . . . . . . . . . . . . . . . . . . . . . . . . . . . . . . . . . . . . . . .51 xi

Longenecker_FM.qxd

xii

10/20/06

10:37 AM

Page xii

CONTENTS

7 Tutorial . . . . . . . . . . . . . . . . . . . . . . . . . . . . . . . . . . . . . . . . . . . . . . . . . . . . . . . . . . . . . . . . .52 Appendix: List of Abbreviations . . . . . . . . . . . . . . . . . . . . . . . . . . . . . . . . . . . . . . . . . . . . . .79 Suggested Reading . . . . . . . . . . . . . . . . . . . . . . . . . . . . . . . . . . . . . . . . . . . . . . . . . . . . . . . . .81 Index . . . . . . . . . . . . . . . . . . . . . . . . . . . . . . . . . . . . . . . . . . . . . . . . . . . . . . . . . . . . . . . . . . .86

Longenecker_CH01_001-011.qxd

10/7/06

2:47 PM

Page 1

Chapter 1

Acid–Base Physiology I

Definitions

The first goal in mastering acid–base physiology is to establish working definitions of acid–base terms. The symbol [X] denotes the concentration of X in the serum. The pH of the blood is the negative logarithm (– log) of the hydrogen ion (H) concentration at any given time. Acidemia and alkalemia refer to the actual pH of the blood (Table 1-1). If the [H] increases, the pH decreases and the serum becomes acidemic. Conversely, if the [H] decreases, the pH increases and the serum becomes alkalemic. Acidoses and alkaloses are pathophysiologic processes that occur in the body (see Table 1-1). A pathophysiologic process causing the pH to decrease is an acidosis. A pathophysiologic process causing the pH to increase is an alkalosis. Both acidoses and alkaloses may be generated by metabolic or respiratory disorders. Metabolic processes cause primary changes in bicarbonate (HCO3) concentration, whereas respiratory processes cause primary changes in the serum partial pressure of carbon dioxide (pCO2) [see Table 1-1]. When two or more primary acid–base disorders occur simultaneously, a mixed disorder is present. A mixed disorder occurs, for example, when a patient with diabetic ketoacidosis (DKA) [a metabolic acidosis] presents with pneumonia, which causes a respiratory alkalosis. If the metabolic acidosis is present to a greater degree than the respiratory alkalosis, the serum pH will be acidemic. If the metabolic acidosis is present to a lesser degree than the respiratory alkalosis, the serum pH will be alkalemic. Any change in the serum pH due to an acidosis or alkalosis activates a process called compensation, which returns the pH toward the normal range. Compensation is the body’s response to a pathophysiologic process. In response to a primary respiratory process, the body activates the metabolic system to compensate for the change. In response to a primary metabolic process, the body activates the respiratory system to compensate for the change. Compensation does not usually bring the pH back to normal. Rather, compensation acts to bring the pH toward normal. The degree to which the pH is returned toward normal can be predicted by certain equations that apply to each type of primary change (see Chapters 3 and 4). Table 1-2 lists normal reference values for pH, pCO2, and [HCO3]. Note that [H] is measured in nanoequivalents per liter (nEq/L) and [HCO3] is measured in milliequivalents per liter (mEq/L). See the Appendix for a list of all abbreviations used in this book. II

Overview of Acid–Base Physiology1,2,3,4,5,6

Figure 1-1 presents a “bird’s-eye view” of acid–base physiology and illustrates some of the biochemical processes one needs to understand in order to approach acid–base problems. 1

Longenecker_CH01_001-011.qxd

2

10/7/06

2:47 PM

Page 2

CHAPTER 1

TABLE 1-1

DEFINITIONS OF ACID–BASE TERMS

Serum pH  7.36

Acidemia: Alkalemia:

Serum pH  7.44

Acidosis:

A primary pathophysiologic process that increases the [H] and decreases the serum pH

Alkalosis:

A primary pathophysiologic process that decreases the [H] and increases the serum pH

Metabolic acidosis:

A primary process that causes [HCO3] to fall

Metabolic alkalosis:

A primary process that causes [HCO3] to rise

Respiratory acidosis:

A primary process that causes the pCO2 to rise

Respiratory alkalosis:

A primary process that causes the pCO2 to fall

Mixed disorder:

A condition in which more than one primary acid–base process is occurring

Compensation:

A physiologic response to an acidosis or alkalosis which partially returns the pH towards normal

[H]  hydrogen ion concentration; [HCO3]  bicarbonate concentration; pCO2  partial pressure of carbon dioxide.

In general, normal acid–base metabolism can be divided into three phases, illustrated as three levels in Figure 1-1: (1) intracellular production of acids and bases as by-products of metabolism; (2) intravascular transport of acids (or bases); and (3) elimination of acids (or bases) by the kidneys and carbon dioxide (CO2) by the lungs. The acid–base system in the body can be divided into a respiratory component (see Figure 1-1, left side) regulated by the lungs and a metabolic component (see Figure 1-1, right side) regulated by the kidneys. INTRACELLULAR ACID AND BASE PRODUCTION Intracellular aerobic metabolism of carbohydrates, fats, and proteins yields CO2, a volatile acid that is eliminated by the lungs (see “The Respiratory Component: Elimination of CO2 by the Lungs”). A massive amount of CO2 [approximately 15,000 millimoles (mmol)] is produced by body tissues each day (see Figure 1-1, upper left) and is eliminated by the lungs (see Figure 1-1, lower left). Anaerobic metabolism of carbohydrates and aerobic metabolism of fats and proteins yield nonvolatile acids, such as β-hydroxybutyric acid (a ketoacid), lactic acid, sulfuric acid, and phosphoric acid (see Figure 1-1, upper right). Nonvolatile acids are eliminated by the kidney (see Figure 1-1, lower right). Although the body produces nonvolatile acids and bases as by-products of intermediate metabolism, the usual net

NORMAL SERUM VALUES FOR pH, [Hⴙ], [HCO3ⴚ], AND pCO2

TABLE 1-2 Normal serum pH: ⴙ

Normal serum [H ]: Normal serum

[HCO3ⴚ]:

Normal serum pCO2:

7.367.44 40 nEq/L 24 mEq/L 40 mm Hg

[H]  hydrogen ion concentration; [HCO3]  bicarbonate concentration; nEq/L  nanoequivalents per liter; mEq/L  milliequivalents per liter; mm Hg  millimeters of mercury; pCO2  partial pressure of carbon dioxide.

Longenecker_CH01_001-011.qxd

10/7/06

2:47 PM

Page 3

ACID–BASE PHYSIOLOGY Respiratory component

3

Metabolic component

Intracellular production of acid Oxidation of glucose, fats, and protein

Metabolism of proteins

CO2 (H2 CO3) Volatile acid

HA Nonvolatile organic acids, sulfuric acid, and phosphoric acid H+ + A−

Intravascular transport of acids

HCO3− + H+

Renal regulation − of serum HCO3

A−

Kidney

Bicarbonate reclamation and generation CO2 Pulmonary regulation of pCO2 Ventilation

CO2 Exhaled

HCO3− Na+ HCO3−

A− HCO3− HCO Reclamation + Na − 3

NH3− HCO3− Generation H+

Proximal tubule

Lung

H2 CO3

Distal tubule

CO2 + H2 O

H+ primarily buffered by HCO3−

Other buffers

NH4+ A− Excreted in urine

● Figure 1-1 Overview of acid production and regulation of the partial pressure of carbon dioxide (pCO2) and bicarbonate concentration [HCO3]. The respiratory component (left side) processes volatile acid (CO2) and the metabolic component (right side) processes nonvolatile acids. Any change on one side will affect the other because the CO2/HCO3 equilibrium (dashed box) forms a “connection” between the respiratory side and the metabolic side. The carbon dioxide (CO2) level in the blood is maintained in the steady state at 40 millimeters of mercury (mm Hg) by increasing or decreasing pulmonary ventilation (lower left). The HCO3 level in the blood is maintained at 24 milliequivalents per liter (mEq/L) by increasing or decreasing reclamation and generation of HCO3 in the renal tubules (lower right). By regulating these two mechanisms, hydrogen ion concentration [H] is maintained at 40 nanoequivalents per liter (nEq/L) [pH  7.40]. A  anion; H2CO3  carbonic acid; Na  sodium; NH3  ammonia; NH4  ammonium ion.

Longenecker_CH01_001-011.qxd

4

10/7/06

2:47 PM

Page 4

CHAPTER 1

nonvolatile acid production is about 50 to 70 mEq of H per day for an adult on an average North American diet. Pathophysiologic processes have the potential to increase nonvolatile acid production dramatically. THE BUFFER SYSTEM: TRANSPORTING ACID AND MITIGATING pH CHANGES7,8 Because the pK values for most of the nonvolatile acids (HA) produced by cellular metabolism are much lower than 7.40 (the pH of the blood), they are forced to dissociate into H ions and anions (A–). If the H ions produced this way were free to accumulate in the blood, the pH would fall dramatically. Therefore, the body provides a different anion, a buffering anion (Bⴚ), to blind the H ions and to transport them in the blood as HB. The buffer system serves as an acid or base “sink” or “storage” system and is the first line of defense against a change of pH from the value of 7.40. Buffers are weak acids, meaning that at a pH of 7.40 the acid exists in approximately equal amounts of HB and B–. The B– can readily accept a H ion, thus eliminating that ion’s contribution to [H] and providing a vehicle for its transport in the blood. By accepting H ions, buffers mitigate decreases in pH due to acid production. Similarly, because HB is also abundant, it can donate H ions to bases that are produced, thereby blunting the rise in pH associated with increased base production. Buffers do have a limit, however. As they become depleted, the pH changes to a greater and greater extent. If the production of acid is greater than the regeneration of buffers, then the blood becomes increasingly acidemic. The predominant buffer in the extracellular fluid is HCO3. When H reacts with HCO3, H2O and CO2 are produced (see Figure 1-1, leftward progression of the chemical equation in the dashed box). In this case, HB is carbonic acid (H2CO3). The CO2 produced is transported as HCO3 by red blood cells (RBCs) to the lungs, where it is converted back to CO2 and exhaled. Thus, when a nonvolatile acid is produced, HCO3 is consumed and CO2 is produced and exhaled by the lungs. The blood contains about 400 mEq of HCO3, while a net amount of 70 mEq of H (by nonvolatile acids) is generated per day. If the body did not regenerate the HCO3 lost in buffering the nonvolatile acids, the entire buffer capacity would be used up in about 6 days (assuming a normal rate of acid production). Other, less prominent buffers include proteins, phosphate, bone, and hemoglobin. THE RESPIRATORY COMPONENT: ELIMINATION OF CO2 BY THE LUNGS When CO2 (and H2O) is produced by intracellular metabolism (see Figure 1-1, upper left), a small amount is dissolved in the serum. However, most CO2 (and H2O) produced in the tissues is converted into HCO3 and H by a carbonic anhydrase-dependent process within RBCs. The CO2 exerts a partial pressure (pCO2) and exists in equilibrium with HCO3 and H within the vascular space (see Figure 1-1, dashed box). As the venous blood passes through the lungs, the HCO3 and H are transformed by the RBC carbonic anhydrase back into CO2, which then diffuses into the alveolus and is exhaled by the mechanisms of alveolar ventilation. The net effect of ventilation is to eliminate CO2 from the body. Alveolar ventilation is measured by the amount of air (in liters) that enters and leaves the alveoli in one minute of breathing. The greater the alveolar ventilation, the more CO2 is removed from the body per unit time. In the normal steady state, the brain regulates ventilation to keep the pCO2 at 40 millimeters of mercury (mm Hg). Hypoventilation is defined by a pCO2 greater than 45 mm Hg (hypercapnia), and hyperventilation is defined by a pCO2 less than 35 mm Hg (hypocapnia). THE METABOLIC COMPONENT: ELIMINATION OF NONVOLATILE ACIDS9,10 The strong nonvolatile acids (HA) produced by intracellular metabolism immediately dissociate into A– and a H ion after being produced (see Figure 1-1, upper right). The anion (e.g., sulfate, phosphate) is transported by the blood to the kidneys, where it is then filtered and excreted in the urine. When the H ion is buffered by HCO3, CO2 is produced and eliminated from the body (see “The Respiratory

Longenecker_CH01_001-011.qxd

10/7/06

2:47 PM

Page 5

ACID–BASE PHYSIOLOGY

Capillary

5

HCO3−

HCO3− Distal tubular cell

Distal renal tubular lumen

Carbonic anhydrase

+

CO2 + H2O

H

HPO42−

H+

H2PO4−

Distal tubular cell

Excreted in urine

● Figure 1-2 Bicarbonate (HCO3) generation (i.e., acid secretion) [mechanism #1]. Hydrogen ion (H) is secreted and combines with divalent inorganic phosphate (HPO42) to form monovalent inorganic phosphate (H2PO4). This mechanism is limited by the amount of HPO42 that is filtered by the glomerulus into the tubular lumen. This process occurs in the distal tubule and is dependent on carbonic anhydrase.

Component: Elimination of CO2 by the Lungs”). The HCO3 molecule thus consumed and lost from the body must be replaced in order to maintain acid–base balance. The kidneys maintain acid–base balance by generating new HCO3ⴚ. As illustrated by Figures 1-2 and 1-3, HCO3 is generated by the tubular cells of the kidney via two mechanisms. The first mechanism (see Figure 1-2) uses a carbonic anhydrase–dependent process in the distal tubules whereby HCO3 is secreted into the blood stream and H is secreted into the tubular lumen. This H ion is titrated by divalent inorganic phosphate and excreted in the urine. The amount of HCO3 generated by this mechanism is limited by the supply of divalent inorganic phosphate delivered to the tubule and is insufficient to replenish the HCO3 lost during an average day. The second mechanism by which the renal tubular cells generate HCO3 is shown in Figure 1-3. In the proximal tubular cells, glutamine is converted into two molecules of ammonium ion (NH4)

Capillary 2HCO3



2HCO3 Tubular cell



Glutamine +

Tubular cell

2NH4

Renal tubular lumen

+

2NH4

Excreted in urine

● Figure 1-3 Bicarbonate (HCO3) generation (i.e., acid secretion) [mechanism #2]. Glutamine is converted into two molecules of HCO3 and two molecules of ammonium ion (NH4). The net effect of this process is absorption of new HCO3 into the blood stream and excretion of hydrogen ions (H) in the form of NH4 in the urine. The entire process is complex, involving both the proximal and distal tubular cells and carbonic anhydrase.

Longenecker_CH01_001-011.qxd

6

10/7/06

2:47 PM

Page 6

CHAPTER 1

Capillary



HCO3

HCO3− Proximal tubular cell

Renal tubular lumen

+

H

+ Filtered H − HCO3

Carbonic anhydrase

CO2 + H2O Proximal tubular cell

CO2 + H2O

● Figure 1-4 Bicarbonate (HCO3) reclamation. A large amount of HCO3 is filtered through the glomerulus. In the proximal tubular cell, carbonic anhydrase catalyzes a process that produces HCO3 (which is absorbed into the blood stream) and hydrogen ion (H) [which is secreted into the tubular lumen to titrate the filtered HCO3]. The net effect of this carbonic anhydrase–dependent process is the reclamation of filtered HCO3. This process can be stimulated or inhibited, depending on the body’s need for HCO3.

and two molecules of HCO3. The HCO3 is absorbed into the blood stream, thereby replenishing the HCO3 lost by buffering nonvolatile acid. Through a complex process whereby the NH4 reenters the tubular cells in the loop of Henle, a countercurrent gradient of ammonia (NH3) is created in the medulla of the kidney. In the distal tubule, the H is secreted once again in the form of NH4, a process that depends on carbonic anhydrase. Figure 1-3 represents a simplified net effect of the entire process. By regulating the metabolism of glutamine in the proximal tubule and the function of carbonic anhydrase in the distal tubule, the kidney maintains tight control over excretion of acid (generation of HCO3). The generation of new HCO3 can be stimulated dramatically in the event of pathologic degrees of acid production. The kidneys regulate serum [HCO3] not only by generating new HCO3 but also by reclaiming HCO3ⴚ filtered through the glomerulus into the renal tubule. In the proximal tubule, 80% to 90% of the HCO3 that has been filtered through the glomerulus is normally reclaimed by the carbonic anhydrase–dependent process illustrated in Figure 1-4. In brief, within the tubular cell, carbonic anhydrase catalyzes the production of H and HCO3 from H2O and CO2. The H produced is secreted into the tubular lumen and neutralizes the filtered HCO3. The CO2 produced by the neutralization of HCO3 in the lumen returns to the tubular cell as H2CO3, where it sustains the formation of more HCO3 and H. On the capillary side of the tubular cell, the HCO3 produced is transported into the blood stream. Note that this reclamation of HCO3 is actually a recycling of filtered HCO3. This process is tightly controlled by the kidney by raising or lowering of the threshold for reclamation. The remaining 15% of filtered HCO3 not reclaimed in the proximal tubule is normally reclaimed in the distal tubule by the same mechanism. In acidemia, the kidneys maximize HCO3 reclamation. In alkalemia, the kidneys shut off HCO3 reclamation, thus allowing for elimination of large amounts of HCO3 if needed. These renal processes (i.e., generation and reclamation of HCO3) are regulated by many different factors in the body that exert their effects simultaneously. These factors include the blood pH; serum [HCO3]; the glomerular filtration rate (GFR); the filtered load of HCO3; the blood pCO2; and aldosterone, angiotensin II, and serum potassium (K) levels (among others). By regulating these processes, the kidney is able either to reclaim and generate or to excrete large amounts of HCO3. Therefore, the body is able to tightly regulate the serum [HCO3] under normal conditions and to respond to a wide range of acid–base disorders.

Longenecker_CH01_001-011.qxd

10/7/06

2:47 PM

Page 7

ACID–BASE PHYSIOLOGY

7

THE INTERFACE BETWEEN THE RESPIRATORY AND METABOLIC COMPONENTS Although we have divided the body’s acid–base system into respiratory and metabolic components, these components are closely related via the equilibrium between CO2 and HCO3: CO2  H2O ∆ H  HCO3

(eq 1-1)

In the normal steady state, the pCO2 is held constant at 40 mm Hg by ventilatory mechanisms, and the [HCO3] is held constant at 24 mEq/L by the renal mechanisms. Because of the regulation of these two molecules, the [H] (and thereby, the pH) is tightly maintained at 40 mEq/L (pH  7.40). This equilibrium between CO2, HCO3, and H can be mathematically defined by the HendersonHasselbalch equation: pH  pK  log

[HCO3] (0.03)  pCO2

(eq 1-2)

For the purpose of evaluating the acid–base status of a patient, the Henderson-Hasselbalch equation can be simplified to the following relationship: pH r

[HCO3] pCO2

(eq 1-3)

The main points to note about the Henderson-Hasselbalch equation are: • The pH is positively related to [HCO3]. • The pH is inversely related to the pCO2. • The ratio of HCO3 to the pCO2 defines what the pH will be. Figure 1-5 presents the Henderson-Hasselbalch equation in graphical form, with pCO2 and pH as continuous variables, and [HCO3] at three distinct values (16, 24, and 32 mEq/L). Note that a

● Figure 1-5 Graphical representation of the Henderson-Hasselbalch equation. Note that when the partial pressure of carbon dioxide (pCO2) is 40 and bicarbonate (HCO3) concentration is 24, pH is required by the Henderson-Hasselbalch equation to be 7.4. If pCO2 varies while [HCO3] is constant, the acid–base system (i.e., pH, [HCO3], and pCO2) moves along one of the curves. If [HCO3] varies while pCO2 is constant, the acid–base system moves horizontally from one curve to the next. mEq/L  milliequivalents per liter; mm Hg  millimeters of mercury.

Longenecker_CH01_001-011.qxd

8

10/7/06

2:47 PM

Page 8

CHAPTER 1

curvilinear relationship exists between the pH and pCO2. If the pCO2 increases while [HCO3] is constant, the acid–base system (i.e., the pH, [HCO3], and pCO2) moves upward along the curve and the pH decreases. However, if the [HCO3] increases while pCO2 is constant, the curve shifts to the right and the pH increases. The lungs, by regulating pCO2, and the kidneys, by regulating [HCO3], therefore ultimately determine the serum pH. This relationship provides a connection between the respiratory and metabolic components (see Figure 1-1, dashed box). The two components (i.e., respiratory and metabolic) work in parallel in the homeostatic state. Because the two components are “connected,” when a significant perturbation occurs in one component, the other component can be recruited to blunt the change in pH. For example, if a patient with chronic obstructive pulmonary disease hypoventilates, the pCO2 increases and causes a decrease in the blood pH. In this case, the kidneys are recruited to reclaim and generate more HCO3, thus reversing the fall in pH by a specified amount. The body’s response to acid–base perturbations is called compensation (see Chapter 3). III

Common Laboratory Tests in Acid–Base Evaluation

THE ARTERIAL BLOOD GAS (ABG) Measurement of the ABG is the primary laboratory test used to evaluate acid–base status. Blood is drawn from an artery and transported rapidly to the laboratory on ice. Most blood gas machines measure the pH and the pCO2. Then, using these values in the Henderson-Hasselbach equation, the machine calculates the [HCO3]. Because slight errors in the pH and pCO2 can compound and result in large errors in the calculated HCO3, it is more accurate to obtain and use a venous total [HCO3] measured on a chemistry panel drawn at the same time as the ABG. The calculated [HCO3] from the ABG report can be used if no simultaneous venous [HCO3] is available. Another value the ABG provides is the partial pressure of oxygen (pO2). Because the concept of oxygenation is more familiar to students than acid–base, many students are tempted to begin discussing the pO2 when asked about the acid–base status of a patient. It is important to recognize, however, that oxygenation is an issue distinct from a patient’s acid–base status; thus, oxygenation should generally not be part of the acid–base discussion (unless poor oxygenation—as in ischemia—is causing an acid–base disturbance). Most errors in the measurement of the ABG have more of an impact on the pO2 than the pH and pCO2. For example, mistakenly obtaining a venous sample does not usually affect the pH or pCO2 significantly, whereas the pO2 differs greatly between arterial and venous blood. If the sample is not sent to the laboratory on ice, pO2 can drop significantly, whereas the pH and pCO2 are not affected as much. If a patient is febrile or hypothermic, the corresponding measured pCO2 may be slightly off unless the laboratory takes the patient’s temperature into account. This source of error, however, is usually small enough to ignore. Most clinicians do not take it into account. THE SERUM ANION GAP11,12,13,14,15 To maintain electrical neutrality, the serum contains equal concentrations of anions and cations. The serum sodium (Na) accounts for most of the cations present (140 mEq/L). Other cations include K, magnesium (Mg), and calcium (Ca), among others. On the other hand, serum HCO3 and chloride (Cl–) account for only about 128 mEq/L of the balancing anions. The remaining anions (about 12 mEq/L) include negatively charged proteins, phosphates, and sulfates. This difference between the measured [HCO3] and [Cl–] and the measured [Na] is termed the serum anion gap. The serum anion gap is not a mysterious phenomenon; it exists simply because our standard electrolyte panels do not measure all the anions present. Therefore, the anion gap represents a group of anions that are present but not identified. If all anions in the serum were measured and accounted for, there would be no gap. The normal electrolyte pattern and the equation for the anion gap are represented in Figure 1-6. At the normal steady state, the anion gap is 12  3 if K is not included in the calculation.

Longenecker_CH01_001-011.qxd

10/7/06

2:47 PM

Page 9

ACID–BASE PHYSIOLOGY

9

K+= 4.0

+ Na = 140

HCO3−= 24

− CI = 104

Cations

Anions

+ − − Serum anion gap = Na − (HCO3 + CI )

● Figure 1-6 The anion gap. Note that the normal range for the anion gap may be lower, depending on the instrumentation used. Cl  chloride; HCO3  bicarbonate; K  potassium; Na  sodium.

If K is included, the normal range for the anion gap is 16  3. Due to improvements in the laboratory instruments used to measure the ions, the range for the normal anion gap may be lower. It is important to determine what the normal range is at your hospital. For the purpose of this book, we will use 12  3 mEq/L. An elevation of the anion gap almost always indicates the presence of a gap metabolic acidosis (see Chapter 2), although some other processes can change the anion gap. In the presence of these other disorders, the assessment of a gap metabolic acidosis can be clouded. For example, if a patient concomitantly has one of the disorders that decreases the anion gap in addition to a gap metabolic acidosis, the anion gap may be in the normal range. Tables 1-3 and 1-4 list the most important processes that can increase or decrease the anion gap. If a patient has a low anion gap, it is important to consider the differential diagnosis in Table 1-4. THE SERUM OSMOLAL GAP Both ionic and nonionic substances in the serum contribute to the serum osmolality. Under normal conditions, the serum ions and anions, glucose, and blood urea nitrogen (BUN) contribute to the serum osmolality. The osmolality will rise if the concentration of any of the nonionic substances increases, or if other osmotically active exogenous substances are added to the serum. The normal serum osmolality is between 280 and 300 milliosmoles per kilogram (mOsm/kg) and is calculated by the following equation: Calculated serum osmolality  2[Na] 

[Glucose] 18



[BUN] 2.8

(eq 1-4)

Other substances that can cause the serum osmolality to rise include ethanol, methanol, ethylene glycol, isopropanol, mannitol, excess serum lipids, and excess serum proteins. Equation 1-4 is used to compare the actual measured serum osmolality (which accounts for all osmotically active

TABLE 1-3

CAUSES OF AN INCREASED ANION GAP



Elevated-gap metabolic acidosis (most common)



Therapy with sodium citrate or sodium lactate (if shock or hypoxia decreases these anions’ metabolism to HCO3)



Blood transfusion (large amount of citrate in PRBCs)



Alkalosis (causes a small increase in serum lactate, elevating the anion gap about 2–3 mEq/L)

HCO3  bicarbonate; mEq/L  milliequivalents per liter; PRBCs  packed red blood cells.

Longenecker_CH01_001-011.qxd

10

10/7/06

2:47 PM

Page 10

CHAPTER 1

TABLE 1-4

CAUSES OF A DECREASED ANION GAP



Hypoalbuminemia (Albumin, as a negatively charged anion, makes up about 75% of the anion gap.)



Severe dilution of the extracellular fluid



Bromism (Laboratory machines read bromide as chloride, thus overestimating the chloride concentration.)



Multiple myeloma and states of hyperparaproteinemia (Protein cations balanced with chloride increase the calculated chloride, thereby reducing the anion gap.)

substances) with the calculated value (which only accounts for the osmotic activity of the serum ions, glucose, and BUN). The difference between these two values normally is no more than 10 mOsm/kg and is called the osmolal gap. If a patient has an elevated osmolal gap, it is important to determine the substance causing it. For example, if a patient has an osmolal gap of 35 mOsm/kg and an ethanol level of 0 milligrams per deciliter (mg/dL), the differential diagnosis would include other substances such as ethylene glycol (antifreeze), methanol, isopropanol, hyperlipidemia, or hyperproteinemia. See Chapter 7 for further discussion of the serum osmolal gap. THE URINE pH Evaluation of the urine pH can be useful in assessing a patient’s acid–base status. As described earlier in this chapter, normal protein metabolism produces nonvolatile acids (about 70 to 100 mEq of H per day), which are excreted in the urine. Therefore, the urine pH normally is acidic (pH  5.0). If the serum pH is acidemic, the urine pH normally becomes maximally acidic. If the serum pH increases into the alkalemic range, the kidneys minimize the production and reclamation of HCO3. As a result, the HCO3 is excreted in the urine, and the urine pH increases into the alkaline range (pH  8.0). In a renal tubular acidosis (RTA) the kidneys inappropriately excrete HCO3 despite the presence of acidemia. In this case, the urine may be alkaline despite the acidemic serum. See Chapter 6 for a discussion of RTA. It is important to keep in mind that the urine can be alkalinized by the presence of urea-splitting organisms (such as Proteus sp) in the urinary tract. THE URINE ANION GAP AND OSMOLAL GAP16,17 Sometimes it is not clear whether a normal gap metabolic acidosis (see Chapter 2) is due to an extrarenal process (e.g., gastrointestinal loss of HCO3) or defective generation of HCO3 in the kidney (e.g., RTA). The urine anion gap can differentiate between these two situations by indirectly testing for the presence of urinary NH4, which is not generally measured. In the case of an extrarenal cause of a metabolic acidosis and normal renal function, the urine will contain increased amounts of NH4. However, if the kidneys are not producing NH4 sufficiently because of a defect in the tubular function (e.g., RTA), the NH4 will be lower than expected. The primary cations in the urine are NH4, Na, and K; the primary anion is Cl– (HCO3 is not present if the urine pH is  6.5). Other cations and anions exist in the urine, but they usually do not contribute significantly to the electrical charge compared to the primary ions. If urinary [Cl–] is greater than urinary [Na  K], then an unmeasured cation or cations are present to a large degree. This unmeasured cation is usually NH4. If, on the other hand, the urinary [Cl–] is less than urinary [Na  K], it is likely that very little NH4 is present. Urinary anion gap  [Na  K]Urine  [Cl]Urine

(eq 1-5)

In the setting of a typical North American diet, about 50 mmol of NH4 is produced daily. At this level of production, the normal urinary anion gap is around 0 mEq/L. If the kidney is functioning normally in the setting of an acidosis, up to 250 to 300 mmol of NH4 will be produced per day,

Longenecker_CH01_001-011.qxd

10/7/06

2:47 PM

Page 11

ACID–BASE PHYSIOLOGY

11

resulting in a very large negative urinary anion gap. If the urinary anion gap is positive or a small negative value in the setting of an acidosis, the kidney is not appropriately producing NH4, indicating that the kidney is contributing to the acidosis (i.e., RTA). The calculation of the urinary anion gap assumes that no anions other than Cl– are significantly present. In a pure RTA (normal serum anion gap), this is true. However, in the setting of an elevated serum anion gap acidosis (see Chapter 2), other anions such as acetoacetate (in ketoacidosis) or lactate (in lactic acidosis) are present, thus rendering the urinary anion gap a useless test. In this case, the serum anion gap will be elevated as well as the urinary osmolar gap, which is the difference between the measured urine osmolality and the calculated urine osmolality: Calculated urine osmolality 

[Urea]Urine 6.0



[Glucose]Urine 18

 2[Na  K]Urine

(eq 1-6)

THE BASE EXCESS The base excess is a measure of all buffers in the blood, not just the HCO3. Base excess reflects on the metabolic (nonrespiratory) component of an acid–base disturbance and is defined as the quantity (mM) of strong acid or base needed to restore a blood sample to normal pH. Positive base excess signifies an alkalosis while negative base excess signifies an acidosis. The normal value is 0, with a range of 2 to 2. In addition to using base excess to quickly determine the presence of a metabolic abnormality, it may also be used to calculate the amount of sodium bicarbonate needed to normalize acidotic blood or the amount of ammonium chloride needed to normalize alkalotic blood.

Longenecker_CH02_012-019.qxd

10/12/06

1:55 PM

Page 12

Chapter 2

The Primary Acid–Base Disorders Disturbances in either the metabolic component [a change in the bicarbonate (HCO3⫺) concentration] or the respiratory component [a change in the partial pressure of carbon dioxide (pCO2)] lead to changes in the serum pH. In this chapter, we discuss each of the four primary acid–base disorders that result from these changes: • • • •

Metabolic acidosis Metabolic alkalosis Respiratory acidosis Respiratory alkalosis

Many different pathophysiologic processes can produce these four primary acid–base disturbances. Specific differential diagnoses of these disorders are discussed in Chapter 6. I

Metabolic Acidosis18,19

A metabolic acidosis results from any disorder that causes a primary decrease in [HCO3⫺]. In all metabolic acidoses, a nonvolatile acid is added to the body’s acid–base system, either directly through the addition of the acid (such as lactic acid or ketoacids) or indirectly through the inappropriate wasting of HCO3⫺ by the kidney or gastrointestinal tract [essentially the addition of hydrochloric acid (HCl)]. The addition of acid (HA) drives the pCO2/HCO3⫺ equilibrium toward carbon dioxide (CO2) [see Figure 1-1 and Equation 1-1]. The following equation represents the net biochemical change that occurs: HCO3⫺ ⫹ HA S A ⫺ ⫹ CO2 ⫹ H2O

(eq 2-1)

This equation demonstrates that as the HA is buffered, it consumes HCO3⫺ and produces H2O and CO2. The CO2 is transported by red blood cells (RBCs) [see Chapter 1] to the lung and exhaled. The acid is thus removed at the expense of a HCO3⫺ ion. Because [HCO3⫺] is decreased, the Henderson-Hasselbalch equation dictates that the pH will decrease. However, this decrease in pH is much less than it would be without the buffering action of HCO3⫺. The buffer system can mitigate the fall in pH only up to a point. If a large amount of acid is produced, it can overwhelm the buffer system, resulting in a large fall in the pH and [HCO3⫺]. Another way to conceptualize the effect of a metabolic acidosis is to graph the changes that occur using the Henderson-Hasselbalch equation (see Equation 1-2). A metabolic acidosis shifts the pH/pCO2 curve to the left (Figure 2-1). In this example, the acidosis causes a decrease in [HCO3⫺] from 24 milliequivalents per liter (mEq/L) to 16 mEq/L, and a pH drop from 7.40 to 7.24. This graph represents the change in pH due to the acidosis without the compensatory pulmonary response. RENAL RESPONSE TO A METABOLIC ACIDOSIS With increasing acidemia, the proximal tubule reclaims practically all of the HCO3⫺ filtered through the glomerulus. The low pH and serum 12

Longenecker_CH02_012-019.qxd

10/12/06

1:55 PM

Page 13

THE PRIMARY ACID–BASE DISORDERS

13

● Figure 2-1 The effect of metabolic acidosis on the Henderson-Hasselbalch relationship if the partial pressure of carbon dioxide (pCO2) remains constant. ① ⫽ the system at baseline; ➁ ⫽ the change in pH associated with a decrease in bicarbonate (HCO3⫺) concentration from 24 to 16 milliequivalents per liter (mEq/L); mm Hg ⫽ millimeters of mercury.

[HCO3⫺] also stimulate renal tubular HCO3⫺ generation to replenish the HCO3⫺ consumed by the acid. Ammonium ion (NH4⫹) production is greatly increased, providing for a massive increase in hydrogen ion (H⫹) excretion in the urine. Net acid secretion (HCO3⫺ generation) can increase up to ten times the normal rate through these mechanisms, although it may take up to several days to increase to that level. However, if a metabolic acidosis is severe enough it can overwhelm even this renal mechanism with acid production and the pH can fall dramatically. Table 2-1 outlines the renal responses to a metabolic acidosis. RESPIRATORY RESPONSE TO A METABOLIC ACIDOSIS When the buffer system begins to fail and the pH begins to fall due to nonvolatile acid production, the body recruits the pulmonary

TABLE 2-1

RENAL RESPONSE TO A METABOLIC ACIDOSIS

Glomerulus

Proximal Tubule

Less HCO3⫺ is filtered because serum [HCO3⫺] is lower than normal.

The low pH and [HCO3⫺] stimulate reclamation of filtered HCO3⫺.

Proximal and Distal Tubule

The low pH and [HCO3⫺] stimulate generation of new HCO3⫺. Renal production of NH4⫹ is greatly increased (up to 10-fold).

H⫹ ⫽ hydrogen ion; HCO3⫺ ⫽ bicarbonate; NH3 ⫽ ammonia; NH4⫹ ⫽ ammonium ion.

Net Effect

Excretion of H⫹ (as NH4⫹) in the urine is greatly increased.

Longenecker_CH02_012-019.qxd

14

10/12/06

1:55 PM

Page 14

CHAPTER 2 Normal anion profile

+

Gap = 12

Add acid HA

=

End result

Gap = 20 −

A HCO3−=24

HCO3−=16

CI−=104

CI−=104

HA + HCO3−



CI stays the same

− A + H2O + CO2

● Figure 2-2 Anion (A⫺) patterns in an elevated-gap acidosis. When a strong acid is added to the serum (e.g., ketoacids), bicarbonate (HCO3⫺) is consumed. An unmeasured anion is added to the serum, so the gap increases. The end result is an exchange of HCO3⫺ for A⫺. Cl⫺ ⫽ chloride; HA ⫽ acid.

component to help mitigate the decrease in pH by increasing ventilation (i.e., decreasing pCO2). This recruitment is called respiratory compensation (see Chapter 3). TYPES OF METABOLIC ACIDOSES The metabolic acidoses can be divided into two types: the elevated-gap and the normal-gap metabolic acidoses (see Figure 1-6 for the definition of the anion gap). The difference between the two types lies in the identity of the anion (A–) accompanying the H⫹ ion. For the elevated-gap metabolic acidoses, the A⫺ is an unmeasured anion (e.g., ketoacids in diabetic ketoacidosis) [Figure 2-2]. For the normal-gap acidoses, the A⫺ is chloride (Clⴚ) [Figure 2-3], which is measured on the electrolyte panel. A normal-gap acidosis essentially adds HCL to the body’s acid–base system. Another name for a normal-gap acidosis is hyperchloremic acidosis. The end result of an elevated-gap acidosis is an exchange of HCO3⫺ for A⫺ (an anion not normally measured), which increases the anion gap. The end result of a normal-gap acidosis is an exchange of HCO3⫺ for Cl⫺, and the anion gap does not increase from its normal level.

Normal anion profile

+

Gap = 12 HCO3−=24

CI−=104

HCO3− + HCI

Lose − HCO3 or add HCI Gap = 12 − 3

=

End result Gap stays the same

HCO =16

CI−=112

− CI + H2O + CO2

● Figure 2-3 Anion (A⫺) patterns in normal-gap acidosis. In a normal-gap acidosis, bicarbonate (HCO3⫺) is lost from serum and exchanged for chloride (Cl⫺). In effect, hydrochloric acid (HCl) is added to the serum.

Longenecker_CH02_012-019.qxd

10/12/06

1:55 PM

Page 15

THE PRIMARY ACID–BASE DISORDERS

15

A renal tubular acidosis (RTA) is a special case of a normal-gap acidosis that is caused by a defect in the renal tubular HCO3⫺ reclamation or generation processes. This and other examples of the different types of metabolic acidoses are discussed in Chapter 6. II

Metabolic Alkalosis 20,21,22

A metabolic alkalosis results from any process that causes a primary increase in the serum [HCO3⫺]. In all metabolic alkaloses, acid is removed from the system by either loss of acid or inappropriate addition of HCO3⫺. The metabolic alkaloses can be divided into two types: • Metabolic alkalosis associated with sodium chloride (NaCl) and extracellular fluid (ECF) volume depletion (“saline-responsive”) • Metabolic alkalosis associated with mineralocorticoid excess (“saline–non-responsive”) Figure 2-4 graphically illustrates the effect of a metabolic alkalosis on the HendersonHasselbalch relationship. A metabolic alkalosis shifts the pH/pCO2 curve to the right. RENAL RESPONSE TO A METABOLIC ALKALOSIS METABOLIC ALKALOSIS IN EUVOLEMIA AND NORMAL RENAL FUNCTION: ALMOST IMPOSSIBLE TO ACHIEVE If the body experiences a net loss of H⫹ ions for any reason (e.g., in emesis or continuous nasogastric suction) or a net gain of HCO3⫺ (e.g., in sodium bicarbonate, citrate, or acetate ingestion), the [HCO3⫺] will initially begin to rise. In this situation, the kidneys will normally eliminate all extra HCO3⫺ in the urine by shutting off the reclamation and generation of HCO3⫺ by the renal tubule cells. The kidney has a remarkable ability to eliminate massive amounts of HCO3⫺ from the

● Figure 2-4 The effect of a metabolic alkalosis on the Henderson-Hasselbalch relationship if no buffering occurs and partial pressure of carbon dioxide (pCO2) remains constant at 40 millimeters of mercury (mm Hg). ① ⫽ system at baseline; ➁ ⫽ change in pH associated with an increase in bicarbonate (HCO3⫺) concentration from 24 to 32 milliequivalents per liter (mEq/L).

Longenecker_CH02_012-019.qxd

16

10/12/06

1:55 PM

Page 16

CHAPTER 2 RENAL REGULATION OF SERUM [HCO3ⴚ] IN VARIOUS TYPES OF METABOLIC ALKALOSES

TABLE 2-2 Type of Metabolic Alkalosis

Glomerulus

Proximal Tubule

Proximal and Distal Tubule

Net Effect

Large, acute HCO3⫺ load (euvolemia)

HCO3⫺

More is filtered because serum [HCO3⫺] is higher than normal.

The high serum pH and high [HCO3⫺] inhibit HCO3⫺ reclamation.

The high serum pH and high [HCO3⫺] inhibit generation of new HCO3⫺.

Large amounts of [HCO3⫺] are excreted in the urine.

Saline-responsive (ECF depletion)

GFR is reduced resulting in a decreased filtered load of HCO3⫺.

Angiotensin II stimulates the Na⫹/H⫹ antiporter, resulting in a net reclamation of HCO3⫺, despite an already elevated serum [HCO3⫺].

Aldosterone stimulates HCO3⫺ generation, NaHCO3⫺ absorption into the blood, and H⫹ excretion in the urine.

Very low amounts of HCO3⫺ and Cl⫺ are present in the urine.

Saline–nonresponsive (mineralocorticoid excess with volume overload)

The filtered load of HCO3⫺ is normal.

Aldosterone stimulates reclamation of HCO3⫺, despite the elevated serum [HCO3⫺].

Aldosterone stimulates generation of new HCO3⫺, despite the elevated serum [HCO3⫺].

The amount of HCO3⫺ in the urine is low despite high serum [HCO3⫺].

Cl⫺ ⫽ chloride; ECF ⫽ extracellular fluid; GFR ⫽ glomerular filtration rate; H⫹ ⫽ hydrogen ion; [HCO3⫺] ⫽ bicarbonate concentration; Na⫹ ⫽ sodium.

body through this mechanism. The [HCO3⫺] remains normal as long as the patient does not become ECF volume depleted (Table 2-2, “Large, acute HCO3⫺ load”). Thus, even in the setting of greatly elevated intake of HCO3⫺ or large losses of H⫹, it is difficult to develop and maintain a significant metabolic alkalosis if the patient is euvolemic and renal function is normal. When renal function is compromised, however, a load of HCO3⫺ or its precursors can lead to a metabolic alkalosis in the setting of euvolemia (see Chapter 6 for a discussion of milk-alkali syndrome). In order to develop and maintain a metabolic alkalosis in the setting of normal renal function, the patient must be either (1) ECF volume depleted and NaCl depleted (“saline-responsive”) or (2) in a state of mineralocorticoid excess (“saline–non-responsive”). Both of these states can force the kidney to reclaim or generate HCO3⫺ in the face of an elevated HCO3⫺, a situation in which the kidney should be eliminating HCO3⫺. METABOLIC ALKALOSIS IN ECF VOLUME DEPLETION: THE “SALINE-RESPONSIVE” METABOLIC ALKALOSIS Extracellular volume depletion (i.e., loss of intravascular Na⫹, Cl⫺, and H2O) stimulates the production of angiotensin II. This increase in angiotensin II results in a dramatically increased renal tubular avidity for Na⫹. Normally, Cl⫺ is reclaimed in the proximal tubule as the balancing anion with Na⫹. However, in volume depletion, the Cl⫺ concentration in the proximal tubule is very low, while the HCO3⫺ concentration is normal or high. Thus, HCO3⫺ becomes the anion of choice to accompany Na⫹ reabsorption in the proximal tubule. This forced absorption of HCO3⫺ increases both [HCO3⫺] and pH. The drive to maintain intravascular volume (reclamation of Na⫹) overrides the drive to keep the pH within the normal range (excretion of HCO3⫺).

Longenecker_CH02_012-019.qxd

10/12/06

1:55 PM

Page 17

THE PRIMARY ACID–BASE DISORDERS

17

The distal tubule also contributes to the maintenance of the metabolic alkalosis, especially in volume depletion caused by loop diuretics. In volume depletion, renin and aldosterone levels are elevated. Aldosterone stimulates not only the Na⫹/potassium (K⫹) antiporter but also HCO3⫺ generation and H⫹ secretion, thus providing another source of Na⫹ reclamation. Sodium accompanies HCO3⫺ into the blood stream when it is generated by the distal tubular cells. The alkalosis that results from these processes is called a contraction alkalosis (see Table 2-2, “Saline-responsive”). When a volume-depleted patient with a metabolic alkalosis is given NaCl solution (normal saline), the metabolic alkalosis rapidly reverses as large amounts of HCO3⫺ are excreted in the urine. The important point to remember is that the ability of the kidney to excrete HCO3⫺ depends on adequate intravascular volume, Na⫹, and Cl⫺. Once Cl⫺ is repleted, it can balance the absorption of Na⫹, and HCO3⫺ is free to be excreted in the urine. METABOLIC ALKALOSIS IN MINERALOCORTICOID EXCESS: THE “SALINE–NON-RESPONSIVE” METABOLIC ALKALOSIS Metabolic alkalosis can also occur as a result of mineralocorticoid excess, which causes excessive stimulation of Na⫹ resorption and H⫹ excretion in the distal tubule. Aldosterone and its analogues not only drive the Na⫹/H⫹ exchange in the distal tubule, but also stimulate HCO3⫺ generation (see Table 2-2, “Saline–non-responsive”). If excessive, this process can produce a metabolic alkalosis in the setting of increased intravascular volume. Clearly, this kind of metabolic alkalosis would not respond to saline infusion. Conditions in which mineralocorticoid excess occurs include Cushing’s syndrome, primary hyperaldosteronism, and adrenocorticotropic hormone (ACTH)–secreting tumors. REGULATION OF [HCO3⫺] BY THE KIDNEY IN METABOLIC ALKALOSIS In regulating [HCO3⫺] levels, the kidney responds to a diverse range of factors, including the filtered tubular load of HCO3⫺, the glomerular filtration rate (GFR), the blood pH, the serum [HCO3⫺], the pCO2 level, serum postassium (K⫹), angiotensin II levels, and aldosterone. Table 2-2 describes the effect of some of these factors in the setting of metabolic alkalosis. RESPIRATORY RESPONSE TO METABOLIC ALKALOSIS When a metabolic alkalosis due to any cause occurs, the body recruits the respiratory component to help mitigate the increase in pH by decreasing ventilation (increasing pCO2). This recruitment is called respiratory compensation (see Chapter 3). III

Respiratory Acidosis23

A respiratory acidosis results from any process that causes a primary increase in pCO2. The increased pCO2, by mass action, will drive the CO2 and HCO3⫺ equilibrium to the right (see Equation 1-1). According to the Henderson-Hasselbalch equation, the pH will fall (see Equations 1-2 and 1-3). Figure 2-5 graphically illustrates the effect of a respiratory acidosis on the HendersonHasselbalch relationship. A close examination of Equation 1-1, however, indicates that if CO2 increases, the amount of H⫹ produced by the change will equal the amount of HCO3⫺ produced. Why then does pH fall? Do not the H⫹ and HCO3⫺ “balance” each other? The answer to these questions lies in the actual concentrations of these ions. The [H⫹] in the normal equilibrium state is 40 nanoequivalents per liter (nEq/L), whereas the [HCO3⫺] is regulated to 24 mEq/L. A change of pH from 7.40 to 7.20 requires a rise in [H⫹] from 40 to 60 nEq/L (a 50% increase in extracellular [H⫹]). The total amount of HCO3⫺ also changes by the same amount (20 nEq/L). However, because the amount of HCO3⫺ in the ECF is large (24 mEq/L), this change is relatively insignificant. The actual change in [HCO3⫺] is on the order of a 0.001% increase.

Longenecker_CH02_012-019.qxd

18

10/12/06

1:55 PM

Page 18

CHAPTER 2

● Figure 2-5 The effect of respiratory acidosis on the Henderson-Hasselbalch relationship if no buffering occurs and the bicarbonate (HCO3⫺) concentration remains constant at 24 milliequivalents per liter (mEq/L). ① ⫽ system at baseline; ➁ ⫽ change in pH associated with an increase in partial pressure of carbon dioxide (pCO2) from 40 to 60 millimeters of mercury (mm Hg).

THE BUFFER RESPONSE TO RESPIRATORY ACIDOSIS In the acute phase of a respiratory acidosis, pCO2 increases and the pH falls. However, the buffer system diminishes the decrease in pH, so that HCO3⫺ levels rise by a small amount (see Chapter 3). THE RENAL RESPONSE TO RESPIRATORY ACIDOSIS The renal tubular cells are extremely sensitive to changes in pCO2. A primary increase in pCO2 stimulates proximal tubular reclamation of HCO3⫺ and distal tubular generation of HCO3⫺ (Table 2-3). This process, which occurs over a period of 12 to 24 hours, is called renal compensation (see Chapter 3).

TABLE 2-3 Acid–Base Disorder

THE MECHANISMS OF RENAL COMPENSATORY RESPONSE TO RESPIRATORY ACIDOSIS AND RESPIRATORY ALKALOSIS Proximal and Distal Tubule

Glomerulus

Proximal Tubule

Respiratory acidosis

Normal amount of HCO3⫺ is filtered.

Hypercapnia stimulates HCO3⫺ reclamation.

Hypercapnia stimulates generation of new HCO3⫺.

Serum [HCO3⫺] increases as compensation for elevated pCO2.

Respiratory alkalosis

Normal amount of HCO3⫺ is filtered.

Hypocapnia inhibits HCO3⫺ reclamation.

Hypocapnia inhibits generation of new HCO3⫺.

Serum [HCO3⫺] decreases as compensation for low pCO2.

H⫹ ⫽ hydrogen ion; HCO3⫺ ⫽ bicarbonate; pCO2 ⫽ partial pressure of carbon dioxide.

Net Effect

Longenecker_CH02_012-019.qxd

10/12/06

1:55 PM

Page 19

THE PRIMARY ACID–BASE DISORDERS

19

● Figure 2-6 The effect of a respiratory alkalosis on the Henderson-Hasselbalch relationship if no buffering occurs and the bicarbonate HCO3⫺ concentration remains constant at 24 milliequivalents per liter (mEq/L). ➀ ⫽ system at baseline; ➁ ⫽ change in pH associated with a decrease in partial pressure of carbon dioxide (pCO2) from 40 to 20 millimeters of mercury (mm Hg).

IV

Respiratory Alkalosis23

A respiratory alkalosis results from any process that causes a primary decrease in pCO2. The decreased pCO2, by mass action, will drive the CO2 and HCO3⫺ equilibrium to the left (see Equation 1-1). According to the Henderson-Hasselbalch equation (see Equations 1-2 and 1-3), the pH will rise. Figure 2-6 illustrates the effect of a respiratory alkalosis on the Henderson-Hasselbalch relationship. Just as with the changes in respiratory acidosis, the drop in [H⫹] and [HCO3⫺] is measured in nEq, resulting in a significant increase in pH and an insignificant change in the [HCO3⫺], which is measured in mEq. THE BUFFER RESPONSE TO RESPIRATORY ALKALOSIS In the acute phase of a respiratory alkalosis, pCO2 decreases and the pH increases. However, the buffer system inhibits the increase in pH, so that HCO3⫺ levels fall only by a small amount (see Chapter 3). THE RENAL RESPONSE TO RESPIRATORY ALKALOSIS Renal tubular cells are extremely sensitive to changes in pCO2. A primary decrease in pCO2 inhibits the proximal tubular reclamation of HCO3⫺ and the distal tubular generation of HCO3⫺ (see Table 2-3). This process is called renal compensation and occurs over a period of 12 to 24 hours (see Chapter 3).

Longenecker_CH03_020-027.qxd

10/12/06

1:58 PM

Page 20

Chapter 3

The Basics of Compensation I

Understanding Compensation

The Henderson-Hasselbalch equation (see Equations 1-2 and 1-3) provides the framework for the concept of compensation. In a metabolic acidosis or alkalosis, the primary change occurs in the bicarbonate (HCO3) concentration. The brain senses the ensuing change in pH and adjusts the patient’s ventilatory pattern, which results in a change of the partial pressure of carbon dioxide (pCO2) in the same direction as the change in [HCO3]. This response is called respiratory compensation for the metabolic process. Conversely, in a respiratory acidosis or alkalosis, the primary change occurs in the pCO2. The renal tubular cells of the kidney sense this change in pH and adjust the threshold for reclamation and generation of HCO3, which results in a change of [HCO3] in the same direction as the change in pCO2. This response is called metabolic compensation for the respiratory process. Table 3-1 lists the primary and compensatory changes for the four primary acid–base disorders. Note that the compensatory processes cause changes in the pCO2 and [HCO3]. Although some physiologists refer to these compensatory responses as “secondary” or “compensatory” acidoses or alkaloses, it is preferable to refer to them simply as “compensation.” Compensation is a response to a primary pathophysiologic process. Only pathophysiologic processes causing a primary change in the [HCO3] or pCO2 should be termed “acidoses” or “alkaloses.” Confusion often arises when someone refers to compensation as an acidosis or alkalosis. For example, the respiratory compensation for a simple metabolic acidosis is hyperventilation (decreased pCO2), which also happens to be the primary action of a respiratory alkalosis. Referring to this compensatory process as a “secondary respiratory alkalosis” can easily mislead one to evaluate this simple disorder as a “mixed metabolic acidosis and respiratory alkalosis,” which it is not. In this case, only a metabolic acidosis (along with its physiologic compensatory response, hyperventilation) is present. It is also important to note that “compensation” does not mean the pH will be normalized or even near-normalized. Many students incorrectly assume that if the pH is not near normal, full compensation has not occurred. However, compensation rarely returns the pH to a near-normal value. For example, a patient with a severe acidosis may have a pH of 7.20 and be completely compensated. With very few exceptions, compensation does not return the pH to 7.40. If it did, it would remove the stimulus for the compensation! The degree to which the pCO2 changes in compensation for metabolic processes and the degree to which the [HCO3] changes in compensation for respiratory processes can be predicted by a small set of simple formulas, which are presented in this chapter. If the formula’s calculated (i.e., predicted) value does not match the actual value, then one can conclude: 1. Another primary process is present, or 2. The body has not yet had time to achieve full compensation

20

Longenecker_CH03_020-027.qxd

10/12/06

1:58 PM

Page 21

THE BASICS OF COMPENSATION

TABLE 3-1

21

PRIMARY AND COMPENSATORY CHANGES IN ACID–BASE DISORDERS

Disorder

Primary Process

Compensation

Metabolic acidosis

HCO3 ↓

pCO2 ↓

Metabolic alkalosis

HCO3 ↑

pCO2 ↑

Respiratory acidosis

pCO2 ↑

HCO3 ↑

Respiratory alkalosis

pCO2 ↓

HCO3 ↓

HCO3  bicarbonate; pCO2  partial pressure of carbon dioxide.

Full compensation usually requires 12 to 24 hours, depending on the acid–base disorder. The formulas for the metabolic processes assume that compensation has already been achieved ( 12 hours of acidosis or alkalosis). In most cases, this assumption is correct. Most metabolic acidoses or alkaloses develop slowly, allowing time for the compensatory processes to work before the patient presents with symptoms. For example, patients with the metabolic acidosis associated with renal failure may take a week or more to become symptomatic. Because of the long period of progression, compensation to this form of metabolic acidosis occurs simultaneously with the development of the primary process. Exceptions to this assumption include ingestion of toxins and sudden events. In these cases, adjustments need to be made because the formulas do not apply. For example, if a patient ingests a large amount of ethylene glycol, the resulting metabolic acidosis will likely occur before the respiratory compensation has had time to develop fully. In this case, the usual formula for a metabolic acidosis (which assumes full compensation) may not be valid. The formulas for respiratory processes are divided into acute [ 8 hours (i.e., uncompensated)] and chronic [ 24 hours (i.e., fully compensated)] changes and can be used according to the clinical situation. It is important to remember, however, that an intermediate phase falls between the acute and chronic phases. During this intermediate phase (12 to 24 hours), the actual values will fall between the values predicted by the two formulas.

II

Predicting the Degree of Compensation

If a primary acidosis or alkalosis is present, one can predict the expected degree of compensation using equations that describe a normal response to the given degree of acidosis or alkalosis. IMPORTANT CAVEAT These formulas provide only an approximation of the true expected compensation. At extreme pH values (e.g., pH  7.10 and  7.60), these equations are not valid for predicting the expected compensation for the acid–base disturbance. METABOLIC ACIDOSIS The body compensates for a metabolic acidosis (primary change: [HCO3] decreases) by increasing ventilation (pCO2 decreases). Figure 3-1 illustrates the respiratory compensation for a metabolic acidosis. When dealing with a metabolic acidosis, it is helpful to ask, “For this degree of metabolic acidosis (i.e., the lowering of the HCO3), what is the expected pCO2 (the respiratory compensation)?” The equation for calculating this expected respiratory compensation for a metabolic acidosis (Winter’s formula) is:24 pCO2  1.5 [HCO3]  8 (; 2)

(eq 3-1)

Equation 3-1 is used to evaluate whether or not another process is present in addition to the metabolic acidosis. The calculated (expected) pCO2 is compared to the measured pCO2. If the two values do not match, then another primary process is present. Mixed disorders are diagnosed in

Longenecker_CH03_020-027.qxd

22

10/12/06

1:58 PM

Page 22

CHAPTER 3

● Figure 3-1 The effect of metabolic acidosis with respiratory compensation on the Henderson-Hasselbalch relationship. The degree to which the partial pressure of carbon dioxide (pCO2) changes after compensation is predicted by Winter’s formula (see Equation 3-1). ➀  system at baseline; ➁  change in pH associated with a decrease in bicarbonate (HCO3) concentration from 24 to 16 milliequivalents per liter (mEq/L) due to a metabolic acidosis; ➂  effect of respiratory compensation for the metabolic acidosis; mm Hg  millimeters of mercury.

this way. Remember that for Equation 3-1 to be valid, enough time must have elapsed since the onset of the acidosis (12 to 24 hours) for compensation to occur. In the setting of a metabolic acidosis:

• If the pCO2 is less than expected, a respiratory alkalosis is also present in addition to the metabolic acidosis. Only a respiratory alkalosis could lower the pCO2 beyond the expected degree of respiratory compensation. • If the pCO2 is greater than expected, a respiratory acidosis is also present in addition to the metabolic acidosis. Only a respiratory acidosis could raise the pCO2 beyond the expected degree of respiratory compensation. • If the pCO2 equals the value calculated by Equation 3-1, a pure metabolic acidosis is present. No mixed disorder is present. METABOLIC ALKALOSIS The body compensates for a metabolic alkalosis (primary change: HCO3 increases) by decreasing ventilation (pCO2 increases). Figure 3-2 illustrates the respiratory compensation for a metabolic alkalosis. The equation used to estimate the expected pCO2 for a given degree of metabolic alkalosis (increase in HCO3) is:25 pCO2  0.9 [HCO3]  16

(eq 3-2)

Note that Equation 3-2 does not have a confidence interval. No equation, including this one, has been found to be completely reliable in predicting respiratory compensation for a metabolic alkalosis.

Longenecker_CH03_020-027.qxd

10/12/06

1:58 PM

Page 23

THE BASICS OF COMPENSATION

23

● Figure 3-2 The effect of metabolic alkalosis with respiratory compensation on the Henderson-Hasselbalch relationship. The degree to which the partial pressure of carbon dioxide (pCO2) changes after compensation is predicted by Equation 3-2. ➀  system at baseline; ➁  change in pH associated with an increase in bicarbonate (HCO3) concentration from 24 to 32 milliequivalents per liter (mEq/L) due to a metabolic alkalosis; ➂  effect of respiratory compensation for the metabolic alkalosis; mm Hg  millimeters of mercury.

In the setting of a metabolic alkalosis:

• If the pCO2 is less than expected, a respiratory alkalosis may be present in addition to the metabolic alkalosis. • If the pCO2 is much greater than expected, a respiratory acidosis may be present in addition to the metabolic alkalosis. • If the pCO2 equals the value calculated by Equation 3-2, a pure metabolic alkalosis is probably present. Because of the low reliability of Equation 3-2, several other “looser” rules can be applied to improve confidence in your conclusions. In the setting of a metabolic alkalosis:

• If the pCO2 is below 40 millimeters of mercury (mm Hg), you can conclude that a respiratory alkalosis definitely exists in addition to the metabolic alkalosis. A metabolic alkalosis should cause the pCO2 to be higher, not lower, than 40 mm Hg. • If the pCO2 is elevated above 50 mm Hg, you can conclude that a respiratory acidosis probably exists in addition to the metabolic alkalosis. It is highly unusual (although possible on occasion) for respiratory compensation to raise the pCO2 above 50 to 55 mm Hg in response to a metabolic alkalosis. If the pCO2 is between 40 and 50 mm Hg, the type of additional disorders, if any exist, is anybody’s guess!

Longenecker_CH03_020-027.qxd

24

10/12/06

1:58 PM

Page 24

CHAPTER 3

RESPIRATORY ACIDOSIS The body compensates for a respiratory acidosis (primary change: pCO2 increases) by increasing renal generation and reclamation of HCO3 ([HCO3] increases). Figure 3-3 illustrates the changes seen with metabolic compensation in the setting of respiratory acidosis. In the acute setting ( 8 hours), the [HCO3] rises minimally due to the buffer system. But because it takes 12 to 24 hours for the kidney to achieve maximal compensation, two sets of equations need to be used: one for the acute setting and one for the chronic setting. In evaluating a respiratory acidosis, therefore, knowing the clinical history is crucial to the proper application of these equations. Many textbooks provide a host of equations for predicting compensation in a respiratory acidosis, many of which are difficult to remember. The easiest method for analyzing compensation in a respiratory acidosis is to convert these complicated formulas into the following simple method: 1. Calculate how many units of 10 mm Hg the pCO2 has risen. For example, if the pCO2 is 60, it has risen 2 units of 10 mm Hg (20 mm Hg) above its normal value of 40. 2. Apply the formulas26,27,28 in Table 3-2 to determine the expected renal compensation (“acute” implies “uncompensated,” and “chronic” implies “compensated”). In the setting of a respiratory acidosis:

• If the pH or [HCO3] is lower than the value you calculate, then a primary metabolic acidosis is present in addition to the respiratory acidosis. Only a metabolic acidosis could lower the pH or [HCO3] beyond the expected degree of renal compensation. A mixed disorder is present.

● Figure 3-3 The effect of respiratory acidosis with renal compensation on the Henderson-Hasselbalch relationship. The degree to which the bicarbonate (HCO3) concentration and pH change after compensation is predicted by the formulas in Table 3-2. In this example, the [HCO3] changes from 24 to 26 (acute change, before compensation) to 32 (chronic change, after compensation). The pH changes from 7.40 to 7.24 (acute change, before compensation) to 7.34 (chronic change, after compensation). ➀  system at baseline; ➁  change in pH associated with an increase in the partial pressure of carbon dioxide (pCO2) from 40 to 60 millimeters of mercury (mm Hg), if no buffering or compensation occurs; ➂  effect of buffering (the “acute” change of a respiratory alkalosis); ➃  effect of renal compensation (the compensation associated with a “chronic” respiratory alkalosis); mEq/L  milliequivalents per liter.

Longenecker_CH03_020-027.qxd

10/12/06

1:58 PM

Page 25

THE BASICS OF COMPENSATION

TABLE 3-2

25

FORMULAS FOR RESPIRATORY ACIDOSIS

Phase of Respiratory Acidosis

Expected Changes for Each Incremental Increase of pCO2 by 10 mm Hg

Acute

pH decreases by 0.08 [HCO3] increases by 1 mEq/L

Chronic

pH decreases by 0.03 [HCO3] increases by 4 mEq/L

HCO3  bicarbonate; mEq/L  milliequivalents per liter; mm Hg  millimeters of mercury; pCO2  partial pressure of carbon dioxide.

• If the pH or [HCO3] is higher than the value you calculate, then a primary metabolic alkalosis is present in addition to the respiratory acidosis. Only a metabolic alkalosis could raise the pH or [HCO3] beyond the expected degree of renal compensation. A mixed disorder is present. • If the values are equal, then a pure respiratory acidosis is present. RESPIRATORY ALKALOSIS The body compensates for a respiratory alkalosis (primary change: pCO2 decreases) by decreasing renal generation and reclamation of HCO3 ([HCO3] decreases). Figure 3-4 illustrates the renal compensation that occurs in the setting of a respiratory alkalosis.

● Figure 3-4 The effect of respiratory alkalosis with renal compensation on the Henderson-Hasselbalch relationship. The degree to which the bicarbonate (HCO3) concentration and pH changes after compensation is predicted by the formulas in Table 3-2. In this example, the [HCO3] changes from 24 to 20 (acute change, before compensation) to 14 (chronic change, after compensation); the pH changes from 7.40 to 7.56 (acute change, before compensation) to 7.46 (chronic change, after compensation). ➀  system at baseline; ➁  change in pH associated with a decrease in the partial pressure of carbon dioxide (pCO2) from 40 to 20 millimeters of mercury (mm Hg), if no buffering or compensation occurs; ➂ effect of buffering (the “acute” change of a respiratory alkalosis); ➃  effect of compensation (the “chronic” change of a respiratory alkalosis); mEq/L  milliequivalents per liter.

Longenecker_CH03_020-027.qxd

26

10/12/06

1:58 PM

Page 26

CHAPTER 3

TABLE 3-3

FORMULAS FOR RESPIRATORY ALKALOSIS

Phase of Respiratory Alkalosis

Expected Change for Each Incremental Decrease of pCO2 by 10 mm Hg

Acute

pH increases by 0.08 [HCO3] decreases by 2 mEq/L

Chronic

pH increases by 0.03 [HCO3] decreases by 5 mEq/L

HCO3  bicarbonate; mEq/L  milliequivalents per liter; mm Hg  millimeters of mercury; pCO2  partial pressure of carbon dioxide.

In the acute setting, the [HCO3] falls slightly due to the action of the buffer system. But because it takes 12 to 24 hours for the kidney to achieve maximal compensation, two sets of equations need to be used: one for the acute setting and one for the chronic setting. In evaluating a respiratory alkalosis, therefore, knowing the clinical history is crucial to the proper application of these equations. As in the case of respiratory acidoses, many complicated equations are used for determining the expected compensation in respiratory alkalosis. The simplest method for evaluating these equations, however, is similar to that used for a respiratory acidosis: 1. Calculate how many units of 10 mm Hg the pCO2 has fallen. For example, if the pCO2 is 20, it has fallen 2 units of 10 mm Hg (20 mm Hg) below its normal value of 40. 2. Apply the formulas29,30,31 in Table 3-3 to determine the expected compensation (“acute” implies “uncompensated,” and “chronic” implies “compensated”). In the setting of a respiratory alkalosis:

• If the measured pH or [HCO3] is lower than the values you calculate, then a metabolic acidosis is present in addition to the respiratory alkalosis. Only a metabolic acidosis could lower the pH or [HCO3] beyond the degree of expected renal compensation. A mixed disorder is present. • If the measured pH or [HCO3] is higher than the values you calculate, then a metabolic alkalosis is present in addition to the respiratory alkalosis. Only a metabolic alkalosis could raise the pH or [HCO3] beyond the degree of expected renal compensation. A mixed disorder is present. • If the values are equal, then a pure respiratory alkalosis is present. III

The Formulas Needed to Evaluate Most Acid–Base Disorders

It is possible to approach the majority of simple and mixed acid–base disorders using only four simple equations and two memorized numbers (0.08 and 0.03). Although you may find other formulas helpful, the six formulas in Table 3-4 represent the minimum needed to approach the majority of acid–base problems. The only situations in which more equations are needed are a triple acid–base disorder (see Chapter 4) and when you want to check the consistency of reported values of the pH, HCO3, and pCO2 (use the Henderson-Hasselbalch equation to do this). IV

A Word about Confidence Limits

When beginning to use these formulas, many students wonder about the significance of small differences between the calculated (expected) compensation value and the actual value of the compensating entity. Two important rules of thumb are:

Longenecker_CH03_020-027.qxd

10/12/06

1:58 PM

Page 27

THE BASICS OF COMPENSATION

TABLE 3-4

27

THE MINIMAL SET OF FORMULAS NEEDED TO EVALUATE MOST ARTERIAL BLOOD GASES

The general relationship between pH, [HCO3], and pCO2

pH ∝ [HCO3]/pCO2

The anion gap

AG  Na  (HCO3  Cl)

Equation for metabolic acidosis

pCO2  1.5 [HCO3]  8 (2)

Equation for metabolic alkalosis

pCO2  0.9 [HCO3]  16

Formula for respiratory acidosis

For every increment of pCO2 by 10 mm Hg Acute process: expect the pH to decrease by 0.08 Chronic process: expect the pH to decrease by 0.03

Formula for respiratory alkalosis

For every decrement of pCO2 by 10 mm Hg Acute process: expect the pH to increase by 0.08 Chronic process: expect the pH to increase by 0.03

Cl  chloride; HCO3  bicarbonate; mm Hg  millimeters of mercury; Na  sodium; pCO2  partial pressure of carbon dioxide.

1. The smaller the difference between the calculated value and the actual value, the less likely a second disorder is present, and, if one is present, the less likely it is clinically significant. 2. The larger the difference between the calculated and the actual value, the more likely a second disorder is present, and, if one is present, the more likely it is clinically significant. For example, Winter’s formula (see Equation 3-1) has a confidence interval of  2. Therefore, given a [HCO3] of 10 milliequivalents per liter (mEq/L), one would expect the pCO2 to be 23  2 mm Hg (i.e., 21 to 25 mm Hg). Technically, if the pCO2 is less than 21 mm Hg, then a respiratory alkalosis is present. One may reasonably ask, however, “If the pCO2 is 19 or 20 mm Hg, is a respiratory alkalosis really present?” The answer to this question is that an acidosis or an alkalosis can be present to a very small degree. The working principle to apply to all of the formulas presented in this chapter is that if the actual value of pCO2, [HCO3], or pH is only slightly (e.g., 2 to 4 units) different than the expected value, it is reasonable to question whether a mixed disorder in fact exists. You may also conclude that even if one did exist, it may not be clinically significant because it is so small. It is important not to lose sight of the overall acid–base situation by becoming too focused on small differences between the expected and actual values. The greater the difference between the expected value and the actual value, however, the more clinically important the second disorder becomes. In Chapter 4, we discuss the principles of mixed disorders in further depth. In addition, these equations only approximate the true expected values. In the normal range of pH, the approximation is very good. However, the farther the pH is away from 7.40 the looser the approximation becomes, and at extreme pH values ( 7.10 and  7.60), the approximation is not very good at all.

Longenecker_CH04_028-034.qxd

10/12/06

2:01 PM

Page 28

Chapter 4

Mixed Acid–Base Disorders The purpose of the formulas outlined in Chapter 3 is to determine whether one or multiple acid–base disturbances are present. This chapter discusses how mixed acid–base disorders either augment or inhibit each other’s compensatory changes, thus leading to pH, bicarbonate (HCO3⫺), or partial pressure of carbon dioxide (pCO2) values that are different from those predicted by the formulas for simple acid–base disturbances.

I

The Basics of Mixed Disorders

The first concept to understand about mixed acid–base disorders is that when two or more disorders are present, they may be present to differing degrees (Figure 4-1). Figure 4-1A illustrates a diabetic patient with a severe pneumonia that causes a large respiratory alkalosis (which greatly increases pH) and a mild diabetic ketoacidosis (DKA) that causes a small metabolic acidosis (which slightly decreases pH). The net effect of these two disorders is an increased pH, but the increase is not as great as it would be if the small metabolic acidosis were not present. Now suppose the same patient (see Figure 4-1B) presents with severe DKA (which greatly decreases pH) and a mild pneumonia (which slightly increases pH). The net effect of these two disorders is a decreased pH, but the decrease is not as great as it would be if the pneumonia and resultant respiratory alkalosis were not present. Although both cases have a combination of a metabolic acidosis and a respiratory alkalosis, the resultant pH in the two cases is different. Finally, suppose the DKA causes a metabolic acidosis to the same degree that the pneumonia causes the respiratory alkalosis (see Figure 4-1C). In this case, the pH changes in the two disorders balance each other out, resulting in a pH in the normal range. With regard to nomenclature, many people refer to the predominant disorder as the primary disorder, and the smaller disorder as the secondary disorder. However, it is important to understand that from an acid–base perspective, both disorders are primary disturbances. It just so happens that one of the primary disturbances is larger than the other. It is possible to say that one of the two disorders present is clinically more important than the other. In this sense, one disorder might be considered primary and the other secondary. However, in describing a mixed acid–base picture, it is better to refer to both processes as “primary.” Because the term “secondary” is sometimes used to describe either compensation or a second, less clinically important primary disorder, it is preferable to avoid the term “secondary” in such discussions altogether. The terms “compensation” and “mixed disorder” are more specific and, therefore, should be used instead.

28

Longenecker_CH04_028-034.qxd

10/12/06

2:01 PM

Page 29

MIXED ACID–BASE DISORDERS Large Small High respiratory + metabolic = pH alkalosis acidosis 7.60 7.50 7.40

29

Small Large Low Large Large Normal respiratory + metabolic = pH respiratory + metabolic = pH alkalosis acidosis alkalosis acidosis

7.30 7.20 pH A

B

C

● Figure 4-1 Various combinations of a respiratory alkalosis and metabolic acidosis.

II

Patterns of Mixed Acid–Base Disorders

A combination of an acidosis and an alkalosis tends to balance the pH toward 7.40, whereas combinations of acidoses or combinations of alkaloses tend to drive the pH to more extreme values. Because ventilation can only be increased or decreased, a combination of a respiratory acidosis and a respiratory alkalosis is impossible. Conversely, multiple metabolic disorders (acidoses or alkaloses) can coexist [e.g., alcoholic ketoacidosis (AKA) and a contraction alkalosis due to vomiting]. Table 4-1 outlines all of the possible combinations of two acid–base disorders. (See “Identifying Triple Acid–Base Disorders” for a discussion of triple acid–base disorders.) Tables 4-2 through 4-4 compare the changes in pH, pCO2, and [HCO3⫺] for the three combinations that tend to balance the pH. The reason this effect occurs (i.e., balancing of the pH) is that the primary change in each disorder augments the compensatory change in the other. The remaining two combinations (Tables 4-5 and 4-6) result in the opposite: a driving of the pH to more extreme values. The reason this effect occurs is that the primary change in each disorder causes a change in pH in the same direction. In addition, the primary change in each disorder inhibits the compensation of the other. Note also that while these changes result in an exaggeration of the pH change, the resultant changes in [HCO3⫺] and pCO2 tend to balance each other and their values tend to shift toward the normal range. When dealing with any of these combinations of acid–base disorders, it is important to remember that all of the acid–base equations presented in Chapter 3 predict the degree of compensation for a single disorder. If that degree of compensation is not matched, then a second disorder is present. The second disorder can be identified by determining which disorder could change the compensating variable in that direction.

TABLE 4-1

THE FIVE POSSIBLE COMBINATIONS OF TWO ACID–BASE DISORDERS

Combinations that tend to balance the pH Mixed metabolic acidosis and metabolic alkalosis Mixed metabolic acidosis and respiratory alkalosis Mixed metabolic alkalosis and respiratory acidosis Combinations that tend to drive the pH to more extreme values Mixed metabolic acidosis and respiratory acidosis Mixed metabolic alkalosis and respiratory alkalosis

Longenecker_CH04_028-034.qxd

30

10/12/06

2:01 PM

Page 30

CHAPTER 4

TABLE 4-2

CHANGES IN pH, HCO3ⴚ, AND pCO2 IN METABOLIC ACIDOSIS AND METABOLIC ALKALOSIS

Disorder

pH

HCO3ⴚ

pCO2

Pure metabolic acidosis

Decreases

Decreases (primary change)

Decreases (compensation)

Pure metabolic alkalosis

Increases

Increases (primary change)

Increases (compensation)

Mixed metabolic acidosis and metabolic alkalosis

No change*

No change*

No change*

HCO3⫺ ⫽ bicarbonate; pCO2 ⫽ partial pressure of carbon dioxide. * This information is based on the assumption that the disorders are present to the same degree. If the two disorders are not present to the same degree, the pH, HCO3⫺, and pCO2 will be on the side of the larger disorder.

TABLE 4-3

CHANGES IN pH, HCO3ⴚ, AND pCO2 IN METABOLIC ACIDOSIS AND RESPIRATORY ALKALOSIS

Disorder

pH

HCO3ⴚ

pCO2

Pure metabolic acidosis

Decreases

Decreases (primary change)

Decreases (compensation)

Pure respiratory alkalosis

Increases

Decreases (compensation)

Decreases (primary change)

Mixed metabolic acidosis and respiratory alkalosis

No change*

Decreases (more than would be expected with either disorder alone)

Decreases (more than would be expected with either disorder alone)

HCO3⫺ ⫽ bicarbonate; pCO2 ⫽ partial pressure of carbon dioxide. *

This information is based on the assumption that the disorders are present to the same degree. If the two disorders are not present to the same degree, the pH will be on the side of the larger disorder.

TABLE 4-4

CHANGES IN pH, HCO3ⴚ, AND pCO2 IN METABOLIC ALKALOSIS AND RESPIRATORY ACIDOSIS

Disorder

pH

HCO3ⴚ

pCO2

Pure metabolic alkalosis

Increases

Increases (primary change)

Increases (compensation)

Pure respiratory acidosis

Decreases

Increases (compensation)

Increases (primary change)

Mixed metabolic alkalosis and respiratory acidosis

No change*

Increases (more than would be expected with either disorder alone)

Increases (more than would be expected with either disorder alone)

HCO3⫺ ⫽ bicarbonate; pCO2 ⫽ partial pressure of carbon dioxide. *

This information is based on the assumption that the disorders are present to the same degree. If the two disorders are not present to the same degree, the pH will be on the side of the larger disorder.

Longenecker_CH04_028-034.qxd

10/12/06

2:01 PM

Page 31

MIXED ACID–BASE DISORDERS

TABLE 4-5

31

CHANGES IN pH, HCO3ⴚ, AND pCO2 IN METABOLIC ACIDOSIS AND RESPIRATORY ACIDOSIS

Disorder

pH

HCO3ⴚ

pCO2

Pure metabolic acidosis

Decreases

Decreases (primary change)

Decreases (compensation)

Pure respiratory acidosis

Decreases

Increases (compensation)

Increases (primary change)

Mixed metabolic acidosis and respiratory acidosis

Decreases (more than would be expected with either disorder alone)

No change*

No change*

HCO3⫺ ⫽ bicarbonate; pCO2 ⫽ partial pressure of carbon dioxide. * This information is based on the assumption that the disorders are present to the same degree. If the two disorders are not present to the same degree, the HCO3⫺ and pCO2 will be on the side of the larger disorder.

III

Identifying Mixed Metabolic Acid–Base Disorders

TYPES OF MIXED METABOLIC DISORDERS The combination of a metabolic acidosis and a metabolic alkalosis results in a balancing of the pH, pCO2, and [HCO3⫺] (see Table 4-2). In fact, if a normal-gap metabolic acidosis and a metabolic alkalosis are present to the same degree, the anion gap, the pH, the pCO2, and the [HCO3⫺] will all be in the normal range. These values will not indicate the presence of the two acid–base disorders; thus, it is necessary to look for other clinical clues to their presence. If an elevated-gap metabolic acidosis is present, however, it is sometimes possible to differentiate the following two mixed metabolic disorders: • Elevated-gap metabolic acidosis and normal-gap metabolic acidosis • Elevated-gap metabolic acidosis and metabolic alkalosis

TABLE 4-6

CHANGES IN pH, HCO3ⴚ, AND pCO2 IN METABOLIC ALKALOSIS AND RESPIRATORY ALKALOSIS

Disorder

pH

HCO3ⴚ

pCO2

Pure metabolic alkalosis

Increases

Increases (primary change)

Increases (compensation)

Pure respiratory alkalosis

Increases

Decreases (compensation)

Decreases (primary change)

Mixed metabolic alkalosis and respiratory alkalosis

Increases (more than would be expected with either disorder alone)

No change*

No change*

HCO3⫺ ⫽ bicarbonate; pCO2 ⫽ partial pressure of carbon dioxide. * This information is based on the assumption that the disorders are present to the same degree. If the two disorders are not present to the same degree, the HCO3⫺ and pCO2 will be on the side of the larger disorder.

Longenecker_CH04_028-034.qxd

32

10/18/06

2:22 PM

Page 32

CHAPTER 4 Normal anion profile

+

Gap = 12

Elevated-gap metabolic acidosis

+

Gap = 20

Metabolic alkalosis

=

End result

Gap = 20



A −

HCO3 =24

− HCO3 =16

HCO3−=24

CI−=104

CI−=104

CI =96





CI decreases to 96

● Figure 4-2 Anion patterns in a mixed elevated-gap metabolic acidosis and a metabolic alkalosis. The elevated-gap metabolic acidosis exchanges bicarbonate (HCO3⫺) for an unmeasured anion (A⫺). The metabolic alkalosis exchanges chloride (Cl–) for HCO3⫺, restoring [HCO3⫺] to normal (assuming the acidosis and alkalosis are of equal degree). Given the ∆ gap of ⫹8, we expect a ∆ [HCO3⫺] of ⫺8. Instead, the [HCO3⫺] does not change at all. With this “delta–delta” discrepancy of 8 milliequivalents per liter (mEq/L), a mixed elevated-gap metabolic acidosis and metabolic alkalosis can be diagnosed.

THE “DELTA–DELTA” CALCULATION The classic acid–base literature11,32,33,34 states that identification of either of the two mixed metabolic disorders associated with an elevated-gap metabolic acidosis is accomplished by the use of the “delta–delta,” a calculation that compares the change in [HCO3⫺] with the change in the anion gap. In a simple elevated-gap metabolic acidosis, HCO3⫺ is exchanged for an unmeasured anion (A–). This exchange results in an increase in the anion gap (a “delta”) and a decrease in the [HCO3⫺] (a “delta”). The classic literature teaches that in a simple elevated-gap metabolic acidosis these two “delta” values will roughly equal each other. Figures 4-2 and 4-3 illustrate this concept. In the presence of a pure elevated-gap metabolic acidosis both the gap and the [HCO3⫺] change by a value of 8 (see Figures 4-2 and 4-3, “Elevated-Gap Metabolic Acidosis” column). However, when a second metabolic disorder is added (a metabolic alkalosis in Figure 4-2 and a normal-gap metabolic acidosis in Figure 4-3), the two “delta” values will not equal each other.

Normal anion profile Gap = 12

+

Elevated-gap metabolic acidosis Gap = 20 −

+

Normal-gap metabolic acidosis

=

End result

Gap = 20

A − HCO3 =24

CI−=104

− HCO3 =16

CI−=104

HCO3−=8

CI−=112

CI− increases to 112

● Figure 4-3 Anion patterns in a mixed elevated-gap metabolic acidosis and a normal-gap metabolic acidosis. The elevated-gap metabolic acidosis exchanges bicarbonate (HCO3⫺) or an unmeasured anion (A⫺), thus increasing the gap. The normal-gap metabolic acidosis exchanges chloride (Cl⫺) for HCO3⫺, restoring [HCO3⫺] to normal (assuming the acidosis and alkalosis are of equal degree). Given the ∆ gap of ⫹8, we expect a ∆ [HCO3⫺] of ⫺8. Instead, the [HCO3⫺] changes by –16. With this “delta–delta” discrepancy of 8 milliequivalents per liter (mEq/L), a mixed elevated-gap metabolic acidosis and a normal-gap metabolic acidosis can be diagnosed.

Longenecker_CH04_028-034.qxd

10/12/06

2:01 PM

Page 33

MIXED ACID–BASE DISORDERS

33

Consider the situation in which an elevated-gap acidosis coexists with a metabolic alkalosis (see Figure 4-2). The elevated-gap acidosis causes a decrease in the [HCO3⫺] and an equal increase in the gap due to the addition of the A–. The metabolic alkalosis exchanges chloride (Cl–) for HCO3⫺, thus increasing the [HCO3⫺]. If these two processes exist to an equal extent, the pH, [HCO3⫺], and pCO2 will be normal. However, the gap will be increased. In this case, the change in the gap is greater than the change in the [HCO3⫺]: the gap changes by 8 milliequivalents per liter (mEq/L) and the [HCO3⫺] does not change at all. The discrepancy between the two “deltas” identifies a metabolic alkalosis in addition to the more easily identifiable elevated-gap metabolic acidosis. Now consider the situation in which an elevated-gap metabolic acidosis coexists with a normalgap metabolic acidosis (see Figure 4-3). Both cause a decrease in [HCO3⫺], but only the elevatedgap metabolic acidosis causes an increase in the anion gap. In this case, the change in the gap is less than the change in the [HCO3⫺]: the gap changes by 8 mEq/L and the [HCO3⫺] changes by 16 mEq/L. The discrepancy between the two “deltas” identifies a normal-gap metabolic acidosis in addition to the more easily identifiable elevated-gap metabolic acidosis. While the stoichiometry of the “delta–delta” calculation makes sense, it must be used very judiciously in the clinical setting because the assumptions on which it rests are imperfect.14,35 Nonbicarbonate and intracellular buffers contribute to the buffering of newly formed acid so that the actual drop in [HCO3⫺] is usually less than the increase in the anion gap due to a simple elevated-gap metabolic acidosis. The anion gap itself also increases and decreases by many factors (see Tables 1-3 and 1-4). Furthermore, the laboratory values on which the anion gap is based (i.e., [HCO3⫺], [Na⫹], and [Cl⫺]) contribute small errors (about ⫾ 2 mEq/L) to the calculation. Given these limitations, reliable conclusions can be made only when the difference between the two “delta” values is large. For example, a recent study conducted by Paulson and Gadallah found that if a discrepancy of more than 8 mEq/L exists between the two “delta” values in patients with DKA, then a mixed metabolic disorder is likely to be present.36 These same researchers also found that a blood urea nitrogen (BUN) level greater than 23 milligrams per deciliter (mg/dL) [as occurs in dehydration] lowers the sensitivity and specificity of the “delta–delta” calculation.37 The following guidelines are useful when using the “delta–delta” calculation: • It can be used only if an elevated-gap metabolic acidosis is present. • It should not be applied rigidly when evaluating a mixed metabolic acid–base disorder, and should probably not be used if the serum BUN is greater than 23 mg/dL. • It is generally only useful when the discrepancy between the two “delta” values is very large (at least 8 to 10 mEq/L). • The decrease in the [HCO3⫺] is usually less than the increase in the anion gap (can be up to 50% less) in the setting of a simple elevated-gap metabolic acidosis. • Although it can be useful in DKA and AKA due to ethanol, the “delta–delta” calculation is less reliable in other elevated-gap metabolic acidosis (e.g., uremia, lactic acidosis, salicylate overdose). • When calculating the “delta–delta,” it is important to use the normal range for the serum anion gap in the laboratory at your hospital. Different laboratory instruments provide different normal ranges. IV

Identifying Triple Acid–Base Disorders

From a practical standpoint, triple acid–base disorders are not common. The only way a patient can develop a triple acid–base disorder is if a mixed metabolic disorder coexists with a respiratory acidosis or alkalosis. Triple acid–base disorders are usually seen in patients with severe metabolic abnormalities such as AKA (elevated-gap metabolic acidosis), dehydration (metabolic alkalosis), and respiratory depression (respiratory acidosis). Another typical combination is DKA (elevated-gap metabolic acidosis), dehydration (metabolic alkalosis), and hyperventilation from pneumonia or sepsis (respiratory alkalosis).

Longenecker_CH04_028-034.qxd

34

10/12/06

2:01 PM

Page 34

CHAPTER 4

As explained in the section, “Identifying Mixed Metabolic Acid–Base Disorders,” a mixed metabolic disorder can only be diagnosed if an elevated-gap acidosis is present and the “delta–delta” calculation is used. The corollary to this rule is that a triple acid–base disorder can only be diagnosed if an elevated-gap metabolic acidosis is present and the “delta–delta” calculation is used. The same guidelines for using the “delta–delta” calculation for mixed metabolic disorders apply to the diagnosis of triple acid–base disorders. Unfortunately, triple acid–base disorders often occur in the setting of a contraction (metabolic) alkalosis, which often raises the BUN to values above 23 mg/dL, thus making the use of the “delta–delta” calculation less valid. Without getting into too much detail, a triple acid–base disorder is diagnosed as follows. The standard acid–base formulas described in Chapter 3 are used to delineate a mixed respiratory and metabolic disorder. The “delta–delta” calculation is then used. Any large difference between the change in HCO3⫺ and the change in the anion gap (i.e., the “delta–delta”) must be evaluated in light of the expected compensatory change in the [HCO3ⴚ] due to the respiratory disorder. If this compensatory change in HCO3⫺ is taken into account and the “delta–delta” discrepancy is still greater than 8 to 10 mEq/L, a triple acid–base disorder is probably present. Cases 9 and 10 in Chapter 7 are examples of the approach to triple acid–base disorders. A triple acid–base disorder is usually discovered when the standard formulas determine the presence of a respiratory process (acidosis or alkalosis) with a metabolic alkalosis in the setting of an elevated anion gap. The respiratory process and the metabolic alkalosis are diagnosed in the usual way, and the elevated-gap metabolic acidosis is suggested by the presence of an elevated anion gap.

Longenecker_CH05_035-039.qxd

10/12/06

2:02 PM

Page 35

Chapter 5 24

ACervical Practical Approach Neoplasia and Cancer to the Arterial Blood Gas The preceding four chapters presented the theory behind acid–base disorders. Chapters 5 through 7 bridge from acid–base theory to the patients you will evaluate. These chapters present a practical strategy for the evaluation of a patient with acid–base abnormalities. This strategy identifies three objectives to reach in the process of evaluating acid–base disturbances: (1) narrow the options down to one definite acid–base disorder, (2) identify any additional acid–base processes, and (3) investigate the causes of all acid–base disorders present (see Table 5-1). The seven steps illustrated by Figure 5-1 provide a straightforward method to approach the vast majority of acid–base disorders. There are some exceptions which will also be covered. STEP 1: Evaluate the pH and narrow down to two possible processes. If the pH is < 7.36:

The pH can only be acidemic if an acidosis is present. Therefore, either a metabolic acidosis or respiratory acidosis or both are present (see Figure 5-1). If the pH is > 7.44:

The pH can only be alkalemic if an alkalosis is present. Therefore, either a metabolic alkalosis or a respiratory alkalosis or both are present (see Figure 5-1). A way to conceptualize this step is to start with the four acid–base disorders, as shown in Figure 5-1. Then set the two alkaloses aside (if pH  7.36). You have not ruled out the alkaloses as yet; you merely set them aside momentarily in order to identify one of the four processes as definitely present. STEP 2: Evaluate the partial pressure of carbon dioxide (pCO2) and narrow down to one definite process. For a pH < 7.36, either a metabolic or respiratory acidosis exists. • If the pCO2 is  40 millimeters of mercury (mm Hg), you know a metabolic acidosis must be present. A respiratory acidosis cannot result in this combination of pH and pCO2. • If the pCO2 is  40 mm Hg, you know a respiratory acidosis must be present. A metabolic acidosis cannot result in this combination of pH and pCO2. For a pH > 7.44, either a metabolic or respiratory alkalosis exists. • If the pCO2 is  40 mm Hg, you know a respiratory alkalosis must be present. A metabolic alkalosis cannot result in this combination of pH and pCO2. • If the pCO2 is  40 mm Hg, you know a metabolic alkalosis must be present. A respiratory alkalosis cannot result in this combination of pH and pCO2. 35

Longenecker_CH05_035-039.qxd

36

10/12/06

2:02 PM

Page 36

CHAPTER 5

TABLE 5-1

OBJECTIVES AND STEPS IN APPROACHING AN ACID–BASE PROBLEM

Objective 1: Narrow down from four processes to one definite process. Step 1: Evaluate the pH and narrow down to two possible processes. Step 2: Evaluate the pCO2 and narrow down to one definite process. Objective 2: Identify any additional processes. Step 3: Apply the equation or formulas appropriate for the one definite process you identified. Step 4: Rule in or rule out additional processes. Objective 3: Investigate the causes of all acid–base disorders present. Step 5: Check the anion gap. Step 6: Check the urine pH. Step 7: Investigate the differential diagnosis for each disorder present. pCO2  partial pressure of carbon dioxide.

STEP 3: Use the appropriate compensation formula. One common problem that students have is deciding which formula to use. The decision process, however, is straightforward: simply choose the formula for the one acid–base disorder you identified as definitely present in Steps 1 and 2. Note that narrowing down to one definite process in Steps 1 and 2 does not rule out the other three disorders; they are only set aside until Step 4. Another primary acid–base disorder may in fact be present (to a lesser or greater degree). The purpose for using the appropriate formula is to discover if any other acid–base process is present (see Step 4).

STEP 1: Check pH

Metabolic Acidosis Metabolic Acidosis

STEP 3: Choose Formula

STEP 2: Check pCO2

Respiratory Acidosis < 7.36

< 40

Metabolic Acidosis

pCO2 = 1.5[HCO3−] + 8

> 40 Respiratory Acidosis

Respiratory Acidosis Metabolic Alkalosis Respiratory Alkalosis

> 7.44 Metabolic Alkalosis Respiratory Alkalosis

> 40

< 40

Metabolic Alkalosis

pCO2 = 0.9[HCO3−] + 16

STEP 4: Identify Other Disorders The purpose of these formulas is to evaluate for the presence of a mixed disorder. If the calculated value matches the actual value, then a pure disorder is present. If they do not match, then a mixed disorder is present or compensation has not had time to occur. STEP 5: Check Anion Gap

Respiratory Alkalosis

STEP 6: Check Urine pH STEP 7: Generate a differential diagnosis

● Figure 5-1 Algorithm for approaching most acid–base problems. In Step 1, evaluation of the pH narrows down the possible disorders to two. Evaluation of the pCO2 in Step 2 narrows down to one definite acid–base disorder. The choice of formula in Step 3 depends solely on the definite process identified in Step 2. In Step 4 the formula chosen is used to identify the presence or absence of a second acid–base disorder. Checking the anion gap and urine pH in Steps 5 and 6 prepares for Step 7, in which a differential diagnosis is generated.

Longenecker_CH05_035-039.qxd

10/12/06

2:02 PM

Page 37

A PRACTICAL APPROACH TO THE ARTERIAL BLOOD GAS

37

STEP 4: Determine if any other processes are present. Once you have chosen the appropriate formula, apply it and compare the calculated (expected) value with the actual value. If the measured pH, pCO2, or bicarbonate [HCO3] does not coincide with the calculated value, then another acid–base disturbance is present. You can identify the other acid–base disturbances by the direction of deviation from your calculated value, as follows. For a metabolic acidosis:

• If the actual pCO2 is higher than that calculated by Winter’s formula, then a respiratory acidosis is present in addition to the metabolic acidosis. Only a respiratory acidosis could cause the pCO2 to rise above the calculated (expected) value. • If the actual pCO2 is lower than that calculated by Winter’s formula, then a respiratory alkalosis is present in addition to the metabolic acidosis. Only a respiratory alkalosis could cause the pCO2 to fall below the calculated (expected) value. For a metabolic alkalosis:

• If the actual pCO2 is significantly higher ( 5 mm Hg) than that calculated by the formula for metabolic alkalosis, then a respiratory acidosis is probably also present. Remember that the variations of this relationship are wide. Therefore, be careful when applying this rule. However, if the pCO2 is greater than 50 to 55 mm Hg, a respiratory acidosis is almost certainly present in addition to the metabolic alkalosis. • If the actual pCO2 is more than 5 mm Hg less than that calculated by the formula for metabolic alkalosis, then a respiratory alkalosis is probably present. Again, be careful when applying this rule. However, if the pCO2 is less than 40 mm Hg, a respiratory alkalosis is present in addition to the metabolic alkalosis. For a respiratory acidosis or respiratory alkalosis:

• First, determine clinically whether the process is acute or chronic and choose the appropriate formula. • If the pH or the [HCO3] is higher than the value you calculate, then a metabolic alkalosis is present in addition to the respiratory disorder. Only a metabolic alkalosis could cause the pH and [HCO3] to rise above the calculated (expected) value. • If the pH or the [HCO3] is lower than the value you calculate, then a metabolic acidosis is present in addition to the respiratory disorder. Only a metabolic acidosis could cause the pH and [HCO3] to fall below the calculated (expected) value. What if the pH is normal?

The algorithm for approaching acid–base problems (see Figure 5-1) does not work if the pH is in the normal range (7.36 to 7.44). However, if the pH is normal and the [HCO3] and pCO2 are abnormal, you are able to make two statements (assuming a triple acid–base disorder is not present): • An acidosis and an alkalosis are present. • They are present to the same degree. The only combinations that are possible are: • A metabolic alkalosis with a respiratory acidosis • A metabolic acidosis with a respiratory alkalosis • A metabolic alkalosis with a metabolic acidosis We now consider each of these possible combinations in turn.

Longenecker_CH05_035-039.qxd

38

10/12/06

2:02 PM

Page 38

CHAPTER 5

pH in normal range (7.36–7.44)

pCO2 < 36 mm Hg [HCO3−] 44 mm Hg [HCO3−] >27 mEq/L

Mixed respiratory alkalosis and metabolic acidosis (Apply the formula for both, in either order.)

Mixed respiratory acidosis and metabolic alkalosis (Apply the formula for both, in either order.)

● Figure 5-2 Algorithm for evaluating acid–base disorders when the pH is in the normal range. mEq/L  milliequivalents per liter; mm Hg  millimeters of mercury; pCO2  partial pressure of carbon dioxide.

A metabolic alkalosis with a respiratory acidosis In both a metabolic alkalosis and a respi-

ratory acidosis, the [HCO3] and pCO2 increase. Therefore, if the pH is normal and both the [HCO3] and pCO2 are increased, then this combination is present (Figure 5-2; see Table 4-4). You are free to choose the formula for either process (metabolic alkalosis or respiratory acidosis). You will discover that regardless of which formula you choose, you will come up with the same answer (see Chapter 7, Case 11).

A metabolic acidosis with a respiratory alkalosis In both a metabolic acidosis and respiratory alkalosis, the [HCO3] and the pCO2 decrease. Therefore, if the pH is normal and both [HCO3] and pCO2 are decreased, then this combination is present (see Figure 5-2 and Table 4-3). You are free to choose the formula for either process (metabolic acidosis or respiratory alkalosis). You will discover that regardless of which formula you choose, you will come up with the same answer (see Chapter 7, Case 12). A metabolic alkalosis with a metabolic acidosis In a metabolic alkalosis, the [HCO3] and

pCO2 are both increased. Conversely, in a metabolic acidosis, [HCO3] and pCO2 are decreased. If these two processes are present to the same degree (which must be true if the pH is normal), then the HCO3 and the pCO2 changes are also equal and cancel each other out. Therefore, in this combination, the pH, [HCO3], and pCO2 are all in the normal range (see Table 4-2). The only clue from the laboratory data that this combination is present would be an elevated serum anion gap (assuming that the metabolic acidosis present is an elevated-gap acidosis). If the metabolic acidosis were not an elevated-gap acidosis, the mixed disorder would not be discernible from measurement of the pH, HCO3, pCO2, and the anion gap. For more detailed information about mixed metabolic disorders, see Chapter 4. STEP 5: Evaluate the anion gap. An elevated anion gap usually indicates the presence of an elevated-gap metabolic acidosis, although many other disorders can alter the anion gap (see Chapter 1, “The Serum Anion Gap,” and Tables 1-3 and 1-4). In general, the higher the serum anion gap, the more severe the elevated-gap metabolic acidosis. However, in some cases of an elevated-gap metabolic acidosis, there is rapid clearance of the unmeasured anion (A–) by the kidneys. If this occurs, the anion gap may not be very elevated, even in the setting of a severe elevated-gap metabolic acidosis.

Longenecker_CH05_035-039.qxd

10/12/06

2:02 PM

Page 39

A PRACTICAL APPROACH TO THE ARTERIAL BLOOD GAS

39

STEP 6: Check the urine pH. The urine is normally acidic unless the serum is alkalemic. If the urine is alkalotic (pH  6.0) in the face of an acidosis, a renal tubular acidosis (RTA) or a urinary tract infection caused by a urease-producing bacterium may be present. STEP 7: Generate the differential diagnosis for each disorder present. Proceed to Chapter 6.

Longenecker_CH06_040-051.qxd

10/11/06

10:48 AM

Page 40

Chapter 6

Differential Diagnosis Determining whether or not a simple or mixed acid–base disorder exists is not an end in itself. When you determine that one or more acid–base disorders are present, you must pursue a cause for each one. This chapter contains a basic differential diagnosis for each of the four acid–base disorders. Although a short description of each disorder is included, this chapter does not provide a complete differential diagnosis, diagnostic strategy, or therapeutic approach to these disorders. Rather, it forms a basic framework in which to learn more about these disorders. The knowledge you need to acquire for each disorder is far beyond the scope of this book.

Metabolic Acidosis

I

THE ELEVATED-GAP METABOLIC ACIDOSES The elevated-gap metabolic acidoses include (Table 6-1): M Methanol/ethanol U Uremia D Diabetic ketoacidosis (DKA) P I L E S

Paraldehyde ingestion Ischemia (causes lactic acidosis) Lactic acidosis (sepsis, hypotension, hypoxia, ischemia) Ethylene glycol Salicylates

METHANOL AND ETHYLENE GLYCOL POISONING38,39,40 Methanol and ethylene glycol are both metabolized by alcohol dehydrogenase (ADH), the same enzyme that metabolizes ethanol, to formic acid and oxalic acid, respectively. Both of these endproducts produce an elevated-gap acidosis and are highly toxic. Formic acid toxicity causes acute vision loss (with papilledema and optic nerve damage), coma, and death. Oxalic acid toxicity causes coma, brain damage, acute renal failure associated with oxalate crystal–induced obstructive nephropathy, and death. Both methanol and ethylene glycol ingestion are associated with lactic acidosis, which contributes to the elevated-gap acidosis. Diagnosis of both kinds of poisonings requires a high index of suspicion. Blood levels for methanol and ethylene glycol are not usually reported quickly. In addition, the levels may be low or zero because these alcohols are often rapidly metabolized by ADH, especially in the alcoholic patient. In ethylene glycol poisoning, a microscopic analysis of the urine often reveals oxalate crystals. The osmolal gap is diagnostically useful in these situations (see Chapter 1, “The Serum Osmolal Gap”). If the osmolal gap is increased and the ethanol level is zero or very low, it is important to consider other causes of an elevated osmolal gap including ethylene glycol, methanol, sorbitol, 40

Longenecker_CH06_040-051.qxd

10/11/06

10:48 AM

Page 41

DIFFERENTIAL DIAGNOSIS

TABLE 6-1

41

DIFFERENTIAL DIAGNOSIS FOR AN ELEVATED-GAP METABOLIC ACIDOSIS

Disorder

Clinical Findings

Laboratory Tests

Methanol

History of alcohol use; altered vision; abdominal pain; optic disk edema; fixed, dilated pupils

Methanol level Serum osmolal gap (elevated) Ethanol level (may be normal)

Ethanol

History of alcohol use, change in mental status

Ethanol level Serum osmolal gap (elevated)

Uremia

Nausea, vomiting, fluid overload

BUN (80) Creatinine (elevated)

DKA

History of diabetes, polyuria/ polydipsia

Glucose level Serum and urine ketones

Paraldehyde Lactic acidosis

Unusual cause, history of alcohol use Sepsis, hypotension, severe hypoxia, ischemia

Paraldehyde level Lactate level (elevated)

Ethylene glycol

History of alcohol use, suicide attempt, change in mental status

Ethylene glycol level Oxalate crystals in urine

Salicylates

Suicide attempt/accidental, chronic overuse, tinnitus, light-headedness, nausea, concomitant respiratory alkalosis

Salicylate level Lactate level

BUN  blood urea nitrogen; DKA  diabetic ketoacidosis.

mannitol, or isopropanol ingestion. However, because they are rapidly metabolized, a normal osmolal gap does not rule out ethylene glycol or methanol poisonings. If ethanol is present, it can be accounted for in the following equation for the osmolal gap: Osmolal gap  [Serum osmolality]  c (Na   2) 

BUN 2.8



Glucose 18



Ethanol 4.3

d

(eq 6–1)

Note: Normal osmolal gap < 10 milliosmoles per kilogram (mOsm/kg) If the osmolal gap is greater than 10 mOsm/kg after adjusting for the ethanol that is present (using Equation 6-1), other causes of an increased osmolal gap, including methanol and ethylene glycol, must be considered. If the patient presents soon after ingestion of the methanol or ethylene glycol, ethanol can be administered intravenously to competitively inhibit ADH and slow down the metabolism of methanol or ethylene glycol to their toxic metabolites. In severe cases, however, dialysis is the mainstay of therapy because it removes the toxic metabolites directly. ALCOHOLIC KETOACIDOSIS (AKA)41,42,43 Patients with chronic alcoholism will often present to the emergency room with high serum levels of ethanol along with an elevated-gap metabolic acidosis. The predominant nonvolatile acid in AKA is β-hydroxybutyric acid, although a smaller amount of acetoacetate is usually also present. When a patient stops eating in the setting of ethanol intake, hepatic glycogen stores are depleted, insulin production is inhibited, and increased free fatty acids (FFAs) are metabolized for energy. A shift in the reduced nicotinamide adenine dinucleotide:nicotinamide adenine dinucleotide (NADH:NAD) ratio associated with alcohol ingestion facilitates the metabolism of the FFAs to ketoacids. Ethanol is metabolized by ADH, and NAD is used as a cofactor in its metabolism.

Longenecker_CH06_040-051.qxd

42

10/11/06

10:48 AM

Page 42

CHAPTER 6

Ethanol metabolism thus produces high levels of NADH and an increased NADH:NAD ratio, which inhibits oxidation of fatty acids and promotes their metabolism to β-hydroxybutyrate and acetoacetate. In addition, these patients are often dehydrated because of vomiting and poor fluid intake. Dehydration can lead to a concomitant contraction metabolic alkalosis. Moreover, some patients may develop aspiration pneumonia (leading to respiratory alkalosis) or respiratory depression (leading to respiratory acidosis). AKA is the most common scenario in which a triple acid–base disorder is found. Diagnostic tests include the arterial blood gas (ABG), serum anion gap (elevated), ethanol level, and nitroprusside test for acetoacetate (used qualitatively, not quantitatively, because it always underestimates the severity of the acidosis). Hypoalbuminemia is common in these patients as well, leading to a falsely “low” serum anion gap. Alcohol levels may be elevated or zero. Most patients have electrolyte abnormalities with potassium (K), phosphate, and magnesium (Mg) deficiencies, which should be corrected. Therapy involves hydration with normal saline and glucose solution. Thiamine deficiency, also common in these patients, can lead to a lactic acidosis, especially if glucose is infused before thiamine has been repleted. UREMIA44,45 In mild renal insufficiency (e.g., associated with diabetic nephropathy), a renal tubular acidosis (RTA) [normal-gap acidosis] occurs initially. However, as the glomerular filtration rate (GFR) falls below 10 to 20 milliliters per minute (mL/min), organic anions from the metabolism of proteins and amino acids build up in the blood, resulting in an elevated-gap metabolic acidosis. These patients often have a mixed (elevated-gap and normal-gap) metabolic acidosis. As the renal failure progresses, the patient develops increased levels of blood urea nitrogen (BUN) and a worsening elevated-gap metabolic acidosis, a condition called uremia, which is characterized by fluid and electrolyte disturbances, pericarditis, bone abnormalities, and disordered vitamin D metabolism. Patients with chronic progressive renal failure and uremia eventually need dialysis or renal transplantation, or they will die from their disease. DIABETIC KETOACIDOSIS (DKA)46,47 DKA occurs in Type I diabetics and is a direct result of insulin deficiency and glucagon overload. The lack of insulin results in increased blood glucose levels, hepatic glycogen depletion, and release of fatty acids from tissues. The excess glucagon (a counter-regulatory hormone) promotes ketogenesis and production of acetoacetate and β-hydroxybutyrate, resulting in an elevated-gap metabolic acidosis. In DKA, glucose overwhelms the proximal tubule and is delivered to the distal tubule, where it causes an osmotic diuresis. Patients quickly become dehydrated and hypokalemic in conjunction with the increased blood glucose level and metabolic acidosis. The acidosis and lack of insulin cause a shift of K out of cells and hydrogen (H) into cells. Therefore, the serum K is usually normal or high, while total body K is invariably low. Diagnostic tests include serum glucose, serial ABGs, electrolyte panel, and serum/urine ketones by the nitroprusside reaction (note that this test can underestimate the severity of the acidosis). Therapy includes insulin, intravenous (IV) fluids, and electrolyte repletion. LACTIC ACIDOSIS: L-LACTIC ACIDOSIS AND D-LACTIC ACIDOSIS48 Lactic acid is produced in body tissues (e.g., muscle, skin, erythrocytes) and is metabolized to pyruvate in the liver. Lactic acidosis can therefore occur if excess lactic acid is produced or if lactic acid metabolism is reduced. Two types of lactic acid exist: L-lactic acid and D-lactic acid. L-lactic acid is produced endogenously by the body. D-lactic acid is produced by some lactobacilli. D-lactic acidosis can occur in intestinal bacterial overgrowth and is very uncommon. L-lactic acidosis comprises most cases of lactic acidosis. Heavy exercise and seizures produce excess lactic

Longenecker_CH06_040-051.qxd

10/11/06

10:48 AM

Page 43

DIFFERENTIAL DIAGNOSIS

43

acid, but the L-lactic acidosis that results is usually short lived because the exercise or seizure eventually comes to an end. In sepsis, hypoxemia, or tissue ischemia, however, excess lactic acid production due to uncoupling of intracellular oxidative pathways can be prolonged and massive, resulting in severe L-lactic acidosis. States of hypoperfusion (e.g., shock due to myocardial infarction, severe hemorrhage, sepsis) also cause L-lactic acidosis. A variety of drugs, including metformin, salicylates, ethanol, methanol, ethylene glycol, nitroprusside, cocaine, and cyanide, are associated with L-lactic acidosis. In addition, some inborn errors of metabolism are associated with chronic L-lactic acidosis. Diagnostic tests include an ABG, electrolyte panel (serum anion gap), serum lactic acid level, and other diagnostic tests to define the cause of the lactic acidosis. Thiamine deficiency (most commonly seen in the alcoholic patient), when coupled with administration of glucose, results in a lactic acidosis unless thiamine is given prior to the glucose. If this sequence is overlooked, Wernicke-Korsakoff syndrome can result. Although bicarbonate (HCO3) therapy is usually reserved for patients with severe lactic acidosis, the definitive therapy is to recognize and treat the source of lactic acid. SALICYLATE POISONING49,50 Salicylate poisoning can be seen in a suicide attempt or in chronic overuse of aspirin. Subjective symptoms include tinnitus, vertigo, nausea, and light-headedness. The first major acid–base effect usually seen is a stimulation of the brain’s respiratory center in the medulla, resulting in increased ventilation and a respiratory alkalosis. The second major acid–base effect is uncoupling of the oxidative phosphorylation pathways (by salicylate) and inhibition of carbohydrate metabolism in the tissues. These processes result in production of lactic acid and an elevated-gap metabolic acidosis. The concomitant respiratory alkalosis always makes the metabolic acidosis appear less severe than it really is. For example, a patient with a mild case of salicylate poisoning presents with a respiratory alkalosis alone (pH  7.40). A patient with a moderate case presents with a mixed respiratory alkalosis and an elevated-gap metabolic acidosis of approximately equal severity (pH 7.35 to 7.40). A pH of less than 7.35 usually indicates a severe lactic acidosis. Complicating the issue is that the acidosis also increases the volume of distribution of salicylates in body tissues. Vomiting may also cause a contraction alkalosis. Severe effects of salicylates include seizures, myocardial failure, respiratory failure, and coma. Diagnostic tests include serum salicylate and lactic acid levels. The blood glucose level can be elevated in salicylate toxicity. Therapy includes evacuation of stomach contents and alkalinization of the urine to a pH of 8.0 (to keep the salicylate in ionized form in the urine). Because it effectively removes salicylates from the serum, dialysis should be considered in severe cases [serum salicylate levels above 50 milligrams per deciliter (mg/dL)]. THE NORMAL-GAP METABOLIC ACIDOSES The normal-gap metabolic acidoses include (Table 6-2): D U R H A M

Diarrhea Ureteral diversion RTA Hyperalimentation Ammonium chloride/acetazolamide Miscellaneous

GASTROINTESTINAL LOSS OF HCO3 Gastrointestinal loss of HCO3 and fluid (whether by acute diarrhea, cholestyramine ingestion, or pancreatic fistula) can result in a normal gap metabolic acidosis. The renal tubular cells respond to

Longenecker_CH06_040-051.qxd

44

10/11/06

10:48 AM

Page 44

CHAPTER 6

TABLE 6-2

DIFFERENTIAL DIAGNOSIS FOR A NORMAL-GAP METABOLIC ACIDOSIS

Disorder

Clinical Findings

Diarrhea

Is acute secretory diarrhea present? Orthostasis

Ureteral diversion RTA

Prior urologic surgery, Crohn’s disease Asymptomatic

Parenteral nutrition

Is patient on TPN?

Acetazolamide

Is patient on acetazolamide (used to treat glaucoma)?

Ammonium chloride

Is patient taking ammonium chloride?

Miscellaneous

Pancreatic fistula, ingestion of CaCl, hyperparathyroidism, cholestyramine, posthypercapnia

Laboratory Tests

Urine pH  5.5 Fractional excretion of HCO3 Urine pH  6.0

HCO3  bicarbonate; RTA  renal tubular acidosis; TPN  total parenteral nutrition.

these processes by greatly increasing renal reclamation and generation of HCO3 in order to replenish the HCO3 (see Table 2-1). As a patient with severe diarrhea becomes dehydrated, a contraction alkalosis also develops. The contraction alkalosis drives the pH back up—even to normal if it is severe enough. If vomiting and dehydration accompany the diarrhea, the pH may rise into the alkalemic range. In cases of gastrointestinal loss of HCO3 large K losses can result and severe hypokalemia is common, further worsening the metabolic alkalosis. In hypokalemia, H ions in the serum are exchanged for intracellular K ions, thus further lowering serum [H]. After bladder surgery, the ureters are sometimes transplanted into the sigmoid colon (ureteral diversion).51 The large amount of urinary chloride (Cl) delivered to the sigmoid colon is exchanged by the colonic mucosal cells for HCO3, resulting in gastrointestinal losses of HCO3. Bicarbonate wasting by the intestinal tract occurs by the same mechanism when large amounts of calcium chloride (CaCl) are ingested. Gastrointestinal loss of HCO3 is usually clinically apparent. The ABG and electrolyte panel reveal a normal-gap acidosis. Because a large amount of ammonium ion (NH4) is excreted in the urine, the urine [Cl] will actually be higher than the combined value of ([Na]  [K]), resulting in a large negative value for the urine anion gap (see Chapter 1, “The Urine Anion Gap and Osmolal Gap”). RTA: ACIDOSIS CAUSED BY RENAL TUBULAR DYSFUNCTION5,17,19,52,53 RTA is a special case of a normal-gap metabolic acidosis that is caused by defects in the urine acidification function of the renal tubular cells. In patients with RTA, HCO3 is inappropriately wasted by the kidney (i.e., acid is not appropriately secreted), even in the face of acidemia. All patients with RTA are hyperchloremic because Cl is retained when HCO3 is wasted. The urine from all types of RTA has a [Cl] that is less than the combined value of ([Na]  [K]), resulting in a positive urine anion gap when a negative value is expected (see Chapter 1, “The Urine Anion Gap and Osmolal Gap”). The diagnosis of an RTA can usually be made on the basis of the urine anion gap (see Chapter 1, “The Urine Anion Gap and Osmolal Gap”). An elevated urine pH ( 6.0) in the face of acidemia is suggestive of an RTA. However, the urine pH alone is insufficient to diagnose an RTA. Other tests for RTA include urine acidification tests and the fractional excretion of HCO3.53

Longenecker_CH06_040-051.qxd

10/11/06

10:48 AM

Page 45

DIFFERENTIAL DIAGNOSIS

45

The term RTA encompasses a large group of heterogenous disorders that cause dysfunction of the renal tubular cells. Unfortunately, the classification of these disorders has not been fully standardized. Classically, RTA has been divided into four types: • • • •

Type I: disorders affecting the distal tubule Type II: disorders affecting the proximal tubule Type III: disorders affecting both the proximal and distal tubules Type IV: disorders causing an RTA with hyperkalemia (primarily hypoaldosteronism)

Most of the recent classification systems, however, divide RTA into three broad categories: • Distal (type I) • Proximal (type II) • Hyperkalemic (type IV) Distal RTA

In distal (type I) RTA, H secretion (i.e., HCO3 generation) in the distal tubule is defective. The urine pH is inappropriately alkaline in the face of the normal-gap acidosis and acidemia. Some diseases that cause this defect include chronic pyelonephritis; amyloidosis; autoimmune disorders such as lupus erythematosus, rheumatoid arthritis, or Sjögren’s syndrome; genetic diseases such as Marfan syndrome and Ehlers-Danlos syndrome; drugs such as toluene and amphotericin B; and hereditary distal RTA. Because the tubule cannot secrete H ions into the lumen, the kidney must excrete other cations such as calcium (Ca), K, and sodium (Na). This results in severe hypokalemia, disorders of bone metabolism, and sometimes calcium nephrolithiasis. The urine pH is usually alkaline in type I RTA. Distal RTA is treated primarily by providing sufficient alkali (e.g., sodium citrate or potassium citrate) to maintain the [HCO3] in the normal range, thus avoiding the bone and kidney complications seen in this type of RTA. Proximal RTA

In proximal (type II) RTA, the proximal tubule has a diminished ability to reclaim HCO3. The failure to adequately reclaim HCO3 creates a normal-gap metabolic acidosis with a low serum [HCO3]. The low [HCO3] results in a decreased filtered load of HCO3 in the proximal tubule that actually may be low enough for the tubule to reclaim the entire filtered load. Thus, in the steady state, the urine is usually acidic (pH  5.0). Because urinary acidification occurs in the steady state, the wasting of K and Ca and the associated bone abnormalities (as in type I RTA) do not occur. Proximal RTA can be caused by diseases such as multiple myeloma, heavy metal toxicity, amyloidosis, genetic diseases (e.g., Wilson’s disease), and hereditary proximal RTA. When the type II RTA is associated with defective reabsorption of other filtered molecules, such as amino acids, glucose, and phosphates, the disorder is known as Fanconi’s syndrome. Therapy for proximal RTA includes treatment of the underlying cause and, if necessary, alkali therapy (e.g., sodium citrate) to maintain [HCO3] in the normal range. Hyperkalemic RTA

Hyperkalemic (type IV) RTA is primarily seen in patients with hypoaldosteronism. Aldosterone not only stimulates Na reabsorption and K secretion but also stimulates the generation of HCO3 and secretion of H in the distal tubule. Because of this defect, type IV RTA is considered one type of distal RTA. Thus, states of hypoaldosteronism result in diminished ability of the distal tubule to generate HCO3. A normal-gap acidosis characterized by hyperchloremia and hyperkalemia

Longenecker_CH06_040-051.qxd

46

10/11/06

10:48 AM

Page 46

CHAPTER 6

results. This disorder can be seen with primary hypoaldosteronism, mineralocorticoid deficiency syndromes, and hyporeninemic hypoaldosteronism associated with advanced diabetic nephropathy and other kidney diseases. Angiotensin-converting enzyme (ACE) inhibitors and spironolactone can also cause this type of RTA by making the distal tubule less responsive to aldosterone. Therapy for hyperkalemic RTA includes mineralocorticoid replacement and medication to lower serum K levels. ACID INGESTION OR INFUSION Ammonium chloride and the amino acids of parenteral nutrition are metabolized in the body to hydrochloric acid (HCl) and produce a transient normal-gap metabolic acidosis. The decreased pH and [HCO3] stimulate renal tubular reclamation and generation of HCO3 (secretion of H) [see Table 2-1]. This type of metabolic acidosis occurs only if the addition of the acid overwhelms the tubular ability to secrete H and generate ammonia (NH3) for its excretion in the urine; it is usually a short-lived process. ACETAZOLAMIDE AND INGESTION OF CARBONIC ANHYDRASE INHIBITORS Acetazolamide, a drug used to treat glaucoma, inhibits carbonic anhydrase in the eye as well as in the renal tubular cells (see Figures 1-2 and 1-3). Because both renal reclamation and generation of HCO3 depend on carbonic anhydrase, inhibition of this enzyme causes a loss of HCO3 in the urine and a normal-gap metabolic acidosis. Because acetazolamide interferes with renal tubular function, it actually causes an RTA. POSTHYPOCAPNIA Renal tubular cells compensate for a prolonged respiratory alkalosis by decreasing reclamation and generation of HCO3, a process that takes 12 to 24 hours to achieve its full effect (see Table 2-3). If the respiratory alkalosis resolves rapidly, reclamation and generation of HCO3 will return to normal levels over a period of 1 to 2 days. During this period, a resolving normal-gap metabolic acidosis will be present. II

Metabolic Alkalosis

“SALINE-RESPONSIVE” (CHLORIDE-RESPONSIVE) METABOLIC ALKALOSES The salineresponsive metabolic alkaloses include (Table 6-3): D A M P E N

Diuretics Adenoma (secretory villous adenoma) of the colon Miscellaneous (Bartter’s syndrome, penicillin, K deficiency, bulimia) Posthypercapnia Emesis Nasogastric (NG) tube

VOLUME DEPLETION For a discussion of the metabolic alkalosis associated with diuretics, emesis, NG tube, and villous adenoma, see Chapter 2 and Table 2-2. The causes of volume-depletion–related metabolic alkalosis are usually evident clinically. Administration of normal (0.9%) saline reverses these causes of metabolic alkalosis. Note that hypokalemia (often severe) usually accompanies a contraction alkalosis and should be corrected along with the volume depletion. POSTHYPERCAPNIA Renal tubular cells compensate for a prolonged respiratory acidosis by increasing reclamation and generation of HCO3, a process that takes 12 to 24 hours to achieve its full effect (see Table 2-3).

Longenecker_CH06_040-051.qxd

10/11/06

10:48 AM

Page 47

DIFFERENTIAL DIAGNOSIS

TABLE 6-3

47

DIFFERENTIAL DIAGNOSIS FOR A SALINE-RESPONSIVE METABOLIC ALKALOSIS

Disorder

Clinical Findings

Laboratory Tests

Diuretics

History of diuretic use (prescribed or surreptitious)

BUN/creatinine (elevated)

Adenoma

History of diarrhea in setting of heme-positive stools

Colonoscopy Barium enema Stool osmolality

Miscellaneous

IV penicillin, severe potassium depletion, magnesium deficiency, Bartter’s syndrome, history of bulimia

Serum electrolyte Chemistry panels

Posthypercapnia

Hypoventilation resolved; renal compensation not yet resolved

Emesis

History of vomiting

NG tube

NG tube in place and connected to suction

BUN  blood urea nitrogen; IV  intravenous; NG  nasogastric.

If the respiratory acidosis resolves rapidly, reclamation and generation of HCO3 will return to normal levels over a period of several days. During this period, a resolving metabolic alkalosis will be present. However, if the patient is volume depleted, the metabolic alkalosis will persist until sodium chloride (NaCl) and fluid are added to the system (see Chapter 2, “Metabolic Alkalosis”). BARTTER’S SYNDROME54,55 Bartter’s syndrome, a rare autosomal recessive disease, is caused by a tubular defect in Na reabsorption in the loop of Henle. The mild volume depletion that occurs stimulates aldosterone production. The NaCl that is inappropriately delivered to the distal tubule feeds the Na,Kadenosine triphosphatase (ATPase), which is stimulated by the aldosterone. The net result is a hypokalemic metabolic alkalosis. Periodic paralysis or muscle weakness due to hypokalemia can occur in Bartter’s syndrome. Therapy includes K repletion. Although Bartter’s syndrome is listed under the saline-responsive metabolic alkaloses, giving a diet high in NaCl to these patients can worsen K wasting. If NaCl is given, some suggest a K-sparing diuretic. ACE inhibitors are sometimes helpful in Bartter’s syndrome. MISCELLANEOUS CAUSES OF METABOLIC ALKALOSIS Other causes of a saline-responsive metabolic alkalosis include administration of penicillin, extreme K depletion, and Mg deficiency. A very important cause of metabolic alkalosis to remember is bulimia.56 THE “SALINE–NON-RESPONSIVE” METABOLIC ALKALOSES The saline–non-responsive metabolic alkaloses include (Table 6-4): A

Alkali ingestion with decreased GFR

B E L C H

11-β-hydroxylase deficiency Exogenous steroids Licorice ingestion Cushing’s syndrome and disease Hyperaldosteronism

Longenecker_CH06_040-051.qxd

48

10/11/06

10:48 AM

Page 48

CHAPTER 6

TABLE 6-4

DIFFERENTIAL DIAGNOSIS FOR A SALINE–NON-RESPONSIVE METABOLIC ALKALOSIS

Disorder

Clinical Findings

Laboratory Tests

Alkali ingestion

Ingestion of alkali (Mg or Al hydroxide) in renal failure, milk-alkali syndrome (milk and CaCO3)

Creatinine (elevated) Hypercalcemia

β-hydroxylase deficiency

HTN, hypokalemia (a variant of congenital adrenal hyperplasia)

Hypokalemia Renin/aldosterone levels Urine 17-ketosteroid (increased) Blood 11-deoxycortisol (increased)

Exogenous steroids

Patient is on steroids

Licorice ingestion

Large ingestion of licorice

Cushing’s syndrome and disease

Cushingoid, HTN, diabetes, striae

Dexamethasone suppression test Urine cortisol

Hyperaldosteronism

HTN, headaches, fatigue, muscle weakness

Serum aldosterone Serum renin Hypokalemia

Al  aluminum; HTN  hypertension; Mg  magnesium.

ALKALI INGESTION WITH DECREASED GFR Ordinarily, alkali ingestion does not cause a metabolic alkalosis. When renal function is compromised, however, a load of HCO3 or its precursors can lead to a metabolic alkalosis in the setting of euvolemia. This scenario is seen in milk-alkali syndrome,57,58 in which large amounts of milk and calcium carbonate are ingested (in excess of a treatment for peptic ulcer disease or osteoporosis). The kidneys first develop a hypercalcemic nephropathy, resulting in diminished GFR, reduced ability to filter the large load of HCO3, and a subsequent metabolic alkalosis. This type of metabolic alkalosis is not responsive to saline because the primary defect is reduced GFR, not dehydration. The diagnosis of milk-alkali syndrome is made by history, elevated serum Ca levels, renal insufficiency, and a metabolic alkalosis. SYNDROMES OF MINERALOCORTICOID EXCESS59,60 Exogenous steroid administration, licorice ingestion, Cushing’s syndrome, 11-β-hydroxylase deficiency, and primary hyperaldosteronism are all associated with metabolic alkalosis, hypokalemia, and volume overload in the setting of increased mineralocorticoid levels (see Chapter 2, “Metabolic Alkalosis in Mineralocorticoid Excess: The Saline–Non-Responsive Metabolic Alkalosis”). Diagnosis is aided by measuring serum renin and aldosterone levels. Licorice contains a potent analogue of aldosterone and can cause a metabolic alkalosis if ingested in large quantities. Treatment is directed to the underlying cause. HYPOKALEMIA AND METABOLIC ALKALOSIS All cases of metabolic alkalosis result in some degree of K deficiency. Vomiting, NG tubes, and diuretics all result in direct K loss as well as distal tubular loss due to increased levels of aldosterone in the setting of dehydration. In cases of excess mineralocorticoid syndromes, K is secreted by the distal tubules. Furthermore, hypokalemia itself stimulates HCO3 reclamation and generation, thus exacerbating the metabolic alkalosis. It is crucial to replace the K that has been lost while providing therapy for a metabolic alkalosis.

Longenecker_CH06_040-051.qxd

10/11/06

10:48 AM

Page 49

DIFFERENTIAL DIAGNOSIS

III

49

Respiratory Acidosis23,61

A useful way to generate the differential diagnosis of a respiratory acidosis is to consider the mechanisms of ventilation anatomically (Figure 6-1 and Table 6-5). When a patient presents with a respiratory acidosis, consider processes that involve the brain, spinal cord, phrenic nerve, diaphragm, chest wall, pleural space, alveoli, bronchioles, bronchi, trachea, and upper airway. Because the diagnostic tests for these disorders are varied, selection of which tests to obtain depends on the clinical situation. The most common cause of respiratory acidosis is chronic obstructive pulmonary disease (COPD). Patients who are hypoventilating for any of the listed reasons may need to be intubated and placed on mechanical ventilation until they regain the ability to breathe adequately on their own. A respiratory acidosis is often accompanied by hypoxia. The decision to intubate depends on the degree of acidosis, hypercarbia, and hypoxia. It is possible to overventilate the patient with obstructive airways disease (e.g., asthma), with deadly results. Obstructed airways do not allow for the high air flow needed to exhale rapidly. If the rate and volume of mechanical ventilation are too high, the lungs will not have enough time TABLE 6-5

DIFFERENTIAL DIAGNOSIS FOR A RESPIRATORY ACIDOSIS

Structure

Differential Diagnosis

Laboratory Tests

Brain

Stroke, hemorrhage, mass, hypoglycemia, trauma, toxins (opiates, ethanol, sedatives)

Head CT/MRI Toxicology screen Serum glucose, pO2

Spinal cord

Transection at C3–C5, trauma, tumor, transverse myelitis

Spine MRI Cervical spine films

Phrenic nerve

Pancoast’s tumor, mediastinal tumor or mass

Thoracic CT Chest x-ray

Diaphragm

Hypophosphatemia, fatigue, myasthenia gravis, tetanus, Guillain-Barré syndrome, botulism, ALS

Serum phosphate AChR antibodies, Tensilon test Nerve conduction studies

Chest wall

Kyphoscoliosis, morbid obesity, pectus excavatum

Pulmonary function tests (show restrictive pattern)

Pleural space

Pneumothorax, flail chest, pleural effusion, restrictive lung disease

Chest x-ray Chest CT Pulmonary function tests

Alveoli

Severe pulmonary edema, emphysema, bronchiectasis, interstitial fibrosis

Chest x-ray Chest CT Pulmonary function tests

Bronchioles, bronchi

COPD, asthma, sarcoidosis

Chest x-ray Chest CT Pulmonary function tests

Trachea, upper airway

Foreign-body aspiration, obstructive sleep apnea, retropharyngeal abscess, angioneurotic edema, anaphylaxis, ventilator hypoventilation

Laryngoscopy Chest x-ray Bronchoscopy Head/neck CT/MRI

AChR  acetylcholine receptor; ALS  amyotrophic lateral sclerosis; COPD  chronic obstructive pulmonary disease; CT  computed tomography; MRI  magnetic resonance imaging; pO2  partial pressure of oxygen.

Longenecker_CH06_040-051.qxd

50

10/11/06

10:48 AM

Page 50

CHAPTER 6

Brain Cerebrum Midbrain

Upper airway trachea

Spinal cord C3–C5 Phrenic nerve

Pleural space Bronchi Bronchioles Alveoli

Chest wall Diaphragm ● Figure 6-1 Structures involved in the ventilatory process form the basis for a differential diagnosis of respiratory acidosis.

to exhale the ventilated breath due to the slow exhalation rate. The resulting phenomenon, “breath stacking,” is characterized by increasing lung volumes and intrathoracic pressure that reduce blood return to the heart. The blood pressure can drop rapidly and dramatically, resulting in cardiac arrest and death. This complication can be avoided by reducing the frequency and volume of ventilator breaths. The reduced frequency allows enough time for the full breath of air to be exhaled from the lungs before the next ventilator breath. Mechanical underventilation results in a respiratory acidosis.

TABLE 6-6

DIFFERENTIAL DIAGNOSIS FOR A RESPIRATORY ALKALOSIS

Category

Differential Diagnosis

Laboratory Tests

Hypoxia

Pneumonia, pulmonary embolism, pulmonary edema, interstitial fibrosis

Chest x-ray V/Q scan Chest CT Pulmonary function tests

Hyperdynamic states

Pain, fever, sepsis, pregnancy, hyperthyroidism, hepatic failure, anxiety, ventilator overventilation

Blood cultures β-HCG Thyroid-stimulating hormone Albumin Prothrombin time Partial thromboplastin time Liver function tests

CNS disorders

CVA, tumor, infection, intracerebral bleed, subarachnoid hemorrhage

Brain CT/MRI Lumbar puncture

Drugs

Salicylates, catecholamines, progesterone, nicotine

Salicylate level

CNS  central nervous system; CT  computed tomography; CVA  cerebrovascular accident (stroke); HCG  human chorionic gonadotropin; MRI  magnetic resonance imaging; V/Q  ventilation-perfusion.

Longenecker_CH06_040-051.qxd

10/11/06

10:48 AM

Page 51

DIFFERENTIAL DIAGNOSIS

IV

51

Respiratory Alkalosis23

The causes of respiratory alkalosis fall into four categories (Table 6-6) of processes that can stimulate respiration: (1) hypoxia, (2) hyperdynamic states, (3) central nervous system (CNS) disorders, and (4) drugs. The diagnosis of these disorders is made on clinical grounds and by a clinically directed selection of diagnostic tests. The clinical picture associated with respiratory alkalosis is usually dominated by manifestations of the etiologic disorder. Tachypnea is present, though it can be subtle at times. A wide range of symptoms may also accompany respiratory alkalosis including light-headedness, paresthesias, muscle cramps, angina, nausea, vomiting, and confusion. Treatment of a respiratory alkalosis is directed toward the specific underlying cause. If the alkalosis is mild or moderate (pH  7.50) and the patient is mildly symptomatic, carefully monitored breathing into a bag may alleviate symptoms. If the alkalosis is severe (pH  7.55), the patient may need to be sedated, intubated, and placed on a ventilator.

Longenecker_CH07_052-078.qxd

10/12/06

2:04 PM

Page 52

Chapter 7

Tutorial This chapter presents 12 cases involving acid–base disturbances. Each case is solved using a stepby-step application of the method outlined in this book (see Chapter 5).

Case 1 A 20-year-old patient with diabetes is admitted with lethargy, polydipsia, and polyuria. LABORATORY TEST RESULTS Arterial blood gas

pH: 7.24 pCO2: 24 mm Hg [HCO3]: 10 mEq/L

Serum chemistries 

[Na ]: 130 mEq/L [K]: 4.5 mEq/L [Cl]: 94 mEq/L [Glucose]: 600 mg/dl

Urine tests

pH: 5.0

STEP 1: Evaluate the pH and narrow down to two possible processes. pH  7.36 Conclude: Either a metabolic or respiratory acidosis is present. STEP 2: Evaluate the pCO2 and narrow down to one definite process. pCO2  40 mm Hg Conclude: At least a metabolic acidosis is present. STEP 3: Apply the formula for a metabolic acidosis and compare the predicted pCO2 with the actual pCO2. Expected pCO2  1.5 [HCO3]  8  1.5 (10)  8  23 mm Hg Actual pCO2  24 mm Hg STEP 4: Determine if any other processes are present. The expected value closely matches the actual value of pCO2. Conclude: • A pure metabolic acidosis is present. • The metabolic acidosis is fully compensated. • No other processes are present. 52

Longenecker_CH07_052-078.qxd

10/12/06

2:04 PM

Page 53

TUTORIAL

53

STEP 5: Evaluate the anion gap. Anion gap  [Na]  ([Cl]  [HCO3]) It is important to recognize that in cases of hyperglycemia, the [Na] is falsely low. Use the following formula to correct the [Na] for the presence of a very high [glucose]: Corrected [Na]  0.016 ([glucose]  100)  [serum Na] Corrected [Na]  0.016 (600  100)  130  138 mEq/L Anion gap  138  (94  10)  34 Conclude: An elevated-gap metabolic acidosis is present. STEP 6: Check the urine pH. Urine pH  5.0 (appropriate in the setting of acidemia) STEP 7: Generate the differential diagnosis. The clinical history suggests that this patient has diabetic ketoacidosis (DKA); thus, you need to check for urine and serum ketones. Because other elevated-gap acidoses can coexist with DKA, it would also be wise to check serum osmolality and serum creatinine and to order serum and urine toxicology screens. Aspirin overdose is less likely in the absence of a concomitant respiratory alkalosis. If an osmolal gap exists, then you would need to search for ethanol, ethylene glycol, methanol, and so on, in addition to the DKA. Although patients with DKA are usually K-depleted, they often present with [K] in the normal range. This is because the acidosis and lack of insulin result in a shift of K out of cells and into the vascular space. As soon as the patient is rehydrated and given insulin, the [K] will fall, sometimes to dangerously low levels. It is necessary, therefore, to replete K as part of the therapy for DKA. Acid–base disorder:

Simple elevated-gap metabolic acidosis

Case 2 A 42-year-old obese woman with diabetes (diagnosed 2 months ago) presents to your clinic for a routine physical. She is asymptomatic and takes only glyburide. She has no history of hypertension, although her family history is positive for hypertension. On physical examination, she is found to have a blood pressure of 180/110 mm Hg. You order an electrolyte panel and notice that her [HCO3] is elevated. LABORATORY TEST RESULTS Arterial blood gas

Serum chemistries

Urine tests

pH: 7.49 pCO2: 45 mm Hg [HCO3]: 33 mEq/L

[Na+]: 142 mEq/L [K]: 4.5 mEq/L [Cl]: 98 mEq/L [Cr]: 1.1 mg/dL BUN: 14 mg/dL

pH: 6.5

STEP 1: Evaluate the pH and narrow down to two possible processes. pH  7.44 Conclude: A metabolic or respiratory alkalosis is present.

Longenecker_CH07_052-078.qxd

54

10/12/06

2:04 PM

Page 54

CHAPTER 7

STEP 2: Evaluate the pCO2 and narrow down to one definite process. pCO2  40 mm Hg Conclude: At least a metabolic alkalosis is present. STEP 3: Apply the formula for a metabolic alkalosis and compare the expected pCO2 with the actual pCO2. Expected pCO2  0.9 [HCO3]  16  0.9 (33)  16  46 mm Hg Actual pCO2  45 mm Hg STEP 4: Determine if any other processes are present. The actual pCO2 is approximately equal to the expected pCO2. Because the actual pCO2 is approximately equal to the expected pCO2, it is unlikely that a process other than the metabolic alkalosis is present. Remember that this formula is the least accurate of all the formulas for acid–base evaluation, and therefore should be used cautiously. STEP 5: Evaluate the anion gap. Anion gap  142  (98  33)  11 (normal, as expected) STEP 6: Check the urine pH. Urine pH  6.5 (appropriate in the setting of alkalemia) STEP 7: Generate the differential diagnosis. Clinically, the patient does not have any of the “saline-responsive” metabolic alkaloses. The remaining differential diagnoses, therefore, are the “saline–non-responsive” metabolic alkaloses. Given her new onset of hypertension, this patient should be evaluated for hyperaldosteronism and Cushing’s syndrome. Acid–base disorder:

Simple metabolic alkalosis

Case 3 A surgeon refers a 22-year-old man with a hernia to you because of some laboratory tests she found on preoperative testing. The patient has a medical history of kidney stones. LABORATORY TEST RESULTS Arterial blood gas

pH: 7.29 pCO2: 32 mm Hg [HCO3]: 15 mEq/L

Serum chemistries +

[Na ]: 138 mEq/L [K]: 3.0 mEq/L [Cl]: 110 mEq/L

STEP 1: Evaluate the pH and narrow down to two possible processes. pH  7.36 Conclude: A metabolic or respiratory acidosis exists.

Urine tests

pH: 6.0 [Na+]: 35 mEq/L [K]: 45 mEq/L [Cl]: 75 mEq/L

Longenecker_CH07_052-078.qxd

10/12/06

2:04 PM

Page 55

TUTORIAL

55

STEP 2: Evaluate the pCO2 and narrow down to one definite process. pCO2  40 mm Hg Conclude: At least a metabolic acidosis exists. STEP 3: Apply the formula for a metabolic acidosis and compare the expected pCO2 value with the actual pCO2 value. Expected pCO2  1.5 [HCO3]  8  1.5 (15)  8  30 mm Hg Actual pCO2  32 mm Hg STEP 4: Determine if any other processes are present. The actual and expected pCO2 values match closely. Conclude: • A metabolic acidosis exists. • It is fully compensated. • No other acid–base disorders are present. STEP 5: Evaluate the anion gap. Anion gap  138  (110  15)  13 (normal) Conclude: A normal-gap metabolic acidosis is present. STEP 6: Check the urine pH. Urine pH  6.0 (inappropriately high in the setting of acidemia) STEP 7: Generate the differential diagnosis. This patient has no diarrhea, takes no medications, and is not on total parenteral nutrition (TPN). The urine pH is inappropriately high; you expect it to be 5.0 in the presence of a metabolic acidosis. You therefore suspect a renal tubular acidosis (RTA). In the setting of a metabolic acidosis and normal renal function, the urine anion gap would be expected to be a large negative value (e.g., 200). In this case, the urine anion gap is: 75  (35  45)  5 mEq/L. Therefore, a RTA is present. Type IV RTA is unlikely because [K] is low (you would expect it to be high in type IV RTA). Type II RTA urine is usually (although not always) acidic in the steady state. Therefore, the most likely RTA is type I. The history of kidney stones also suggests a type I RTA. Another test that can help differentiate between proximal and distal sources of RTA is the fractional excretion of HCO3, which is elevated ( 15%) in proximal RTA but not in distal RTA ( 15%). Fractional excretion of HCO3 

[HCO3]urine兾[Cr]urine [HCO3]serum兾[Cr]serum

Acid–base disorder: Simple normal-gap metabolic acidosis

Case 4a A 72-year-old woman with a brain tumor (diagnosed 3 months ago) presents with an acute change in mental status that began about 1 hour ago. She is currently comatose and exhibits Kussmaul’s

Longenecker_CH07_052-078.qxd

56

10/12/06

2:04 PM

Page 56

CHAPTER 7

respirations. A computed tomography (CT) scan of the head reveals an intracerebral hemorrhage with midline shift. LABORATORY TEST RESULTS Arterial blood gas

Serum chemistries

Urine tests

pH: 7.57 pCO2: 20 mm Hg [HCO3]: 18 mEq/L

[Na+]: 136 mEq/L [Cl]: 103 mEq/L

pH: 7.0

STEP 1: Evaluate the pH and narrow down to two possible processes. pH  7.44 Conclude: Either a metabolic or respiratory alkalosis is present. STEP 2: Evaluate the pCO2 and narrow down to one definite process. pCO2  40 mm Hg Conclude: At least a respiratory alkalosis is present. STEP 3: Based on the history, apply the formula for an acute respiratory alkalosis. Decide if you will use the pH or [HCO3] or both for your equation. If you decide to use the pH:

• For every decrease of 10 mm Hg in the pCO2, expect the pH to increase (from 7.40) by 0.08. • Because the pCO2 decreased by 20 mm Hg, the pH should increase by 0.16 to a value of 7.56 (expected pH). • Expected pH  7.56 • Actual pH  7.57 If you decide to use the [HCO3ⴚ]:

• For every decrease of 10 mm Hg in the pCO2, expect the [HCO3] to decrease (from 24) by 2 mEq/L. • Because the pCO2 decreased by 20 mm Hg, the [HCO3] should decrease by 4 to a value of 20 mEq/L (expected [HCO3]). • Expected [HCO3ⴚ]  20 mEq/L • Actual [HCO3ⴚ]  18 mEq/L STEP 4: Determine if any other processes are present. Both the actual pH and [HCO3] closely match the expected changes predicted by the formulas for an acute respiratory alkalosis. The clinical story also agrees with this diagnosis. Conclude: • An acute respiratory alkalosis is present. • The kidneys have not had time to compensate (i.e., acute setting). • No other acid–base disorder is present. Note that the same answer results by looking at either the pH or the [HCO3]. In general, only one formula is necessary (the pH formula is easiest). However, it is useful to verify the results of one by checking the other.

Longenecker_CH07_052-078.qxd

10/12/06

2:04 PM

Page 57

TUTORIAL

57

STEP 5: Evaluate the anion gap. Anion gap  136  (103  18)  15 (normal; unlikely that an elevated-gap metabolic acidosis is present) STEP 6: Check the urine pH. Urine pH  7.0 (appropriate in the setting of alkalemia) STEP 7: Generate the differential diagnosis. The acute respiratory alkalosis is probably related to the central nervous system (CNS) bleed, although pulmonary, systemic, or drug-related causes should also be considered. For example, an aspiration pneumonia could also be present. Acid–base disorder: Simple acute (i.e., not yet compensated) respiratory alkalosis

Case 4b The patient from Case 4a survives 2 more days with the same breathing pattern.

LABORATORY TEST RESULTS Arterial blood gas

pH: 7.47 pCO2: 21 mm Hg [HCO3]: 15 mEq/L

Serum chemistries +

[Na ]: 136 mEq/L [Cl]: 110 mEq/L

Urine tests

pH: 6.5

STEP 1: Evaluate the pH and narrow down to two possible processes. pH  7.44 Conclude: A respiratory or metabolic alkalosis is present. STEP 2: Evaluate the pCO2 and narrow down to one definite process. pCO2  40 mm Hg Conclude: At least a respiratory alkalosis is present. STEP 3: Apply the formula for a chronic respiratory alkalosis. (You choose the chronic formula because you know the clinical history.) Decide if you will use the pH or the [HCO3] or both for your equation. If you decide to use the pH:

• For every decrease of 10 mm Hg in the pCO2, expect the pH to increase (from 7.40) by 0.03. • Because the pCO2 decreased by approximately 20 mm Hg, the pH should increase by 0.06 to a value of 7.46 (expected pH). • Expected pH  7.46 • Actual pH  7.47

Longenecker_CH07_052-078.qxd

58

10/12/06

2:04 PM

Page 58

CHAPTER 7

If you decide to use the [HCO3ⴚ]:

• For every decrease of 10 mm Hg in the pCO2, expect the [HCO3] to decrease (from 24) by 5 mEq/L. • Because the pCO2 decreased by approximately 20 mm Hg, the [HCO3] should decrease by 10 to a value of 14 mEq/L (expected [HCO3]). • Expected [HCO3ⴚ]  14 mEq/L • Actual [HCO3ⴚ]  15 mEq/L STEP 4: Determine if any other processes are present. Both the actual pH and actual [HCO3] match the expected changes closely. Given the clinical history, conclude:

• A chronic respiratory alkalosis is present. • The kidneys have compensated for this degree of alkalosis. • No other acid–base disorder is present. Note again that the same answer results using either the pH or the [HCO3]. STEP 5: Evaluate the anion gap. Anion gap  136  (110  15)  11 (normal; unlikely that an elevated-gap metabolic acidosis is present) STEP 6: Check the urine pH. Urine pH  6.5 (appropriate in the setting of alkalemia) STEP 7: Generate the differential diagnosis. The chronic respiratory alkalosis is probably related to the CNS bleed, although pulmonary, systemic, or drug-related causes should also be considered. Acid–base disorder: Simple chronic (i.e., fully compensated) respiratory alkalosis

Case 4c In Case 4b, the patient’s clinical history was known. What if you do not know clinically whether the respiratory disorder is chronic or acute? LABORATORY TEST RESULTS Arterial blood gas

Serum chemistries

Urine tests

pH: 7.47 pCO2: 21 mm Hg [HCO3]: 15 mEq/L

[Na+]: 136 mEq/L [Cl]: 109 mEq/L

pH: 6.0

STEP 1: Evaluate the pH and narrow down to two possible processes. pH  7.44 Conclude: A metabolic or respiratory alkalosis is present.

Longenecker_CH07_052-078.qxd

10/12/06

2:04 PM

Page 59

TUTORIAL

59

STEP 2: Evaluate the pCO2 and narrow down to one definite process. pCO2  40 mm Hg Conclude: At least a respiratory alkalosis is present. STEP 3: Apply the formula for a respiratory alkalosis. Unfortunately, you do not know if the process is chronic or acute. Therefore, you must apply both formulas and delineate all the combinations possible. EVALUATE USING THE FORMULA FOR CHRONIC RESPIRATORY ALKALOSIS. If you decide to use the pH:

• For every decrease of 10 mm Hg in the pCO2, expect the pH to increase (from 7.40) by 0.03. • Because the pCO2 decreased by approximately 20 mm Hg, the pH should increase by 0.06 to a value of 7.46 (expected pH). • Expected pH  7.46 • Actual pH  7.47 If you decide to use the [HCO3ⴚ]:

• For every decrease of 10 mm Hg in the pCO2, expect the [HCO3] to decrease (from 24) by 5 mEq/L. • Because the pCO2 decreased by 20 mm Hg, the [HCO3] should decrease by 10 to a value of 14 mEq/L (expected [HCO3]). • Expected [HCO3ⴚ]  14 mEq/L • Actual [HCO3ⴚ]  15 mEq/L The actual and expected values coincide. Conclude: A simple chronic respiratory alkalosis could account for this arterial blood gas (ABG). EVALUATE USING THE FORMULA FOR AN ACUTE RESPIRATORY ALKALOSIS. If you decide to use the pH:

• For every decrease of 10 mm Hg in the pCO2, expect the pH to increase (from 7.4) by 0.08. • Because the pCO2 decreased by approximately 20 mm Hg, the pH should increase by 0.16 to 7.56 (expected pH). • Expected pH  7.56 • Actual pH  7.47 If you decide to use the [HCO3ⴚ]:

• For every decrease of 10 mm Hg in the pCO2, expect the [HCO3]to decrease (from 24) by 2 mEq/L. • Because the pCO2 decreased by approximately 20 mm Hg, the [HCO3] should decrease by 4 to 20 mEq/L (expected [HCO3]). • Expected [HCO3ⴚ]  20 mEq/L • Actual [HCO3ⴚ]  15 mEq/L STEP 4: Determine if any other processes are present. As stated above, a chronic respiratory alkalosis could account for this ABG. However, if we apply the formula for an acute respiratory alkalosis, we find that the actual values for the pH

Longenecker_CH07_052-078.qxd

60

10/12/06

2:04 PM

Page 60

CHAPTER 7

and [HCO3] are both lower than the expected values. The questions you need to ask at this point are: • Which process could lower the pH from its expected value of 7.56 to the actual value of 7.47? • Which process could lower the [HCO3] from its expected value of 20 to the actual value of 15? The answer to both of these questions is a metabolic acidosis (a metabolic alkalosis would cause the pH and [HCO3] values to be higher than expected). The steps outlined above are used to diagnose mixed disorders. Note once again that whether you use the pH or the [HCO3], the same answer results. You do not need to use both, although it is useful to double-check your answer. Note how important the clinical history is to the evaluation of an ABG. In this exercise, because we removed the history, we ended up with two possible acid–base scenarios, each of which is completely compatible with the ABG: 1. A simple chronic respiratory alkalosis 2. A mixed acute respiratory alkalosis and metabolic acidosis STEP 5: Evaluate the anion gap. Anion gap  136  (109  15)  12 (normal; unlikely that an elevated-gap metabolic acidosis is present) STEP 6: Check the urine pH. Urine pH  6.0 (appropriate in the setting of alkalemia) STEP 7: Generate the differential diagnosis. The differential diagnosis would include all the entities that can cause chronic and acute respiratory alkalosis and a normal-gap metabolic acidosis (see Tables 6-2 and 6-6). The clinical history and physical examination are crucial to narrow down the differential diagnosis. Acid–base disorder: Without the clinical history, we can conclude only that either (1) a simple chronic respiratory alkalosis exists, or (2) a mixed acute respiratory alkalosis and metabolic acidosis are present.

Case 4d Notice that the acid–base status of the patient in Cases 4a and 4b changed from an acute respiratory alkalosis to a chronic respiratory alkalosis over a period of time. We would expect, for the given level of hyperventilation (enough to maintain her pCO2 at 20 mm Hg), that at an intermediate point in time the pH and [HCO3] would assume values between the acute and chronic values. We will therefore analyze the following ABG, taken 12 hours after the patient was admitted to the hospital.

ARTERIAL BLOOD GAS RESULTS Arterial blood gas

pH: 7.52 pCO2: 21 mm Hg [HCO3]: 17 mEq/L

Serum chemistries

Urine tests

Longenecker_CH07_052-078.qxd

10/12/06

2:04 PM

Page 61

TUTORIAL

61

STEP 1: Evaluate the pH and narrow down to two possible processes. pH  7.44 Conclude: A metabolic or respiratory alkalosis is present. STEP 2: Evaluate the pCO2 and narrow down to one definite process. pCO2  40 mm Hg Conclude: At least a respiratory alkalosis is present. STEP 3: Apply the formula for a respiratory alkalosis. Unfortunately, because the respiratory changes have been present for 12 hours, it is probable that the process is midway between chronic and acute. Therefore, apply both formulas and delineate all of the combinations possible. EVALUATE USING THE FORMULA FOR CHRONIC RESPIRATORY ALKALOSIS. If you decide to use the pH:

• For every decrease of 10 mm Hg in the pCO2, expect the pH to increase (from 7.40) by 0.03. • Because the pCO2 decreased by approximately 20 mm Hg, the pH should increase by 0.06 to a value of 7.46 (expected pH). • Expected pH  7.46 • Actual pH  7.52 If you decide to use the [HCO3ⴚ]:

• For every decrease of 10 mm Hg in the pCO2, expect the [HCO3] to decrease (from 24) by 5 mEq/L. • Because the pCO2 decreased by approximately 20 mm Hg, the [HCO3]should decrease by 10 to a value of 14 mEq/L (expected [HCO3]). • Expected [HCO3ⴚ]  14 mEq/L • Actual [HCO3ⴚ]  17 mEq/L Notice that both the actual values for the pH and the [HCO3] are higher than expected. The expected values represent the target (i.e., full) compensation that the kidneys provide by excretion of [HCO3] for this level of hyperventilation. Only two processes can explain the disparity between the expected and actual pH and [HCO3] values: 1. A chronic (i.e., fully compensated) respiratory alkalosis mixed with a small metabolic alkalosis (small because [HCO3] changed by only 3 mEq/L) 2. Compensation for a respiratory alkalosis, which has started but is not yet complete (moving from an acute to a chronic respiratory alkalosis) EVALUATE USING THE FORMULA FOR AN ACUTE RESPIRATORY ALKALOSIS. If you decide to use the pH:

• For every decrease of 10 mm Hg in the pCO2, expect the pH to increase (from 7.40) by 0.08. • Because the pCO2 decreased by 20 mm Hg, the pH should increase by 0.16 to a value of 7.56 (expected pH). • Expected pH  7.56 • Actual pH  7.52

Longenecker_CH07_052-078.qxd

62

10/12/06

2:04 PM

Page 62

CHAPTER 7

If you decide to use the [HCO3ⴚ]:

• For every decrease of 10 mm Hg in the pCO2, expect the [HCO3] to decrease (from 24) by 2 mEq/L. • Because the pCO2 decreased by 20 mm Hg, the [HCO3] should decrease by 4 to a value of 20 mEq/L (expected [HCO3]). • Expected [HCO3ⴚ]  20 mEq/L • Actual [HCO3ⴚ]  17 mEq/L Notice that both the actual values for the pH and the [HCO3] are lower than expected. The expected values represent the pH and [HCO3] that are expected in the acute setting prior to the effects of renal compensation (i.e., uncompensated). Only two processes can explain the disparity between the expected and actual pH and [HCO3] values: 1. An acute (i.e., not yet compensated) respiratory alkalosis mixed with a small metabolic acidosis (small because [HCO3] changed by only 3 mEq/L) 2. Compensation for a respiratory alkalosis, which has started but is not yet complete (moving from an acute to chronic respiratory alkalosis) STEP 4: Determine if any other processes are present. Notice that the same ABG (i.e., pH, 7.52; pCO2, 21; [HCO3], 17) can result from several different combinations of processes. Reviewing Step 3, we find three possibilities: • A mixed chronic respiratory alkalosis and a small metabolic alkalosis • A mixed acute respiratory alkalosis and a small metabolic acidosis • A respiratory alkalosis in which the process of compensation has started but is not complete (i.e., between an acute respiratory alkalosis and a chronic respiratory alkalosis) If the same ABG can be explained by any of these three possibilities, how does one choose the correct one? The correct diagnosis can be made only on the basis of the clinical history. If the patient has been hyperventilating for more than 24 hours, the first option is correct. If the patient has been hyperventilating only a short time ( 8 hours), the second option is correct. If the patient has been hyperventilating between these two times, the third option is correct. Without the clinical history, it is impossible to choose among the three possibilities. Given the clinical history of this case, the third option is correct. STEPS 5, 6, and 7: See these Steps in Cases 4a and 4b. Acid–base disorder: A respiratory alkalosis in which the process of compensation has started but is not complete

Case 5 A 20-year-old man is brought to the emergency room by his sister, who tells you he took a bottle of pills. LABORATORY TEST RESULTS Arterial blood gas

pH: 7.35 pCO2: 15 mm Hg [HCO3]: 8 mEq/L

Serum chemistries +

[Na ]: 140 mEq/L [K]: 3.5 mEq/L [Cl]: 104 mEq/L

Urine tests

pH: 5.0

Longenecker_CH07_052-078.qxd

10/12/06

2:04 PM

Page 63

TUTORIAL

63

STEP 1: Evaluate the pH and narrow down to two possible processes. pH  7.36 Conclude: A metabolic or respiratory acidosis is present. STEP 2: Evaluate the pCO2 and narrow down to one definite process. pCO2  40 mm Hg Conclude: At least a metabolic acidosis is present. STEP 3: Apply the formula for metabolic acidosis (Winters’ formula). Predicted pCO2  1.5[HCO3]  8  1.5 (8)  8  20 mm Hg Actual pCO2  15 mm Hg STEP 4: Determine if any other processes are present. The actual pCO2 is less than predicted. The only process that could decrease the pCO2 beyond that predicted is a respiratory alkalosis. Therefore, you have diagnosed a mixed metabolic acidosis and respiratory alkalosis. A simple metabolic acidosis does not explain this patient’s arterial blood gas (ABG). STEP 5: Evaluate the anion gap. Anion gap  140  (104  8)  28 (elevated) Conclude: The metabolic acidosis is an elevated-gap metabolic acidosis. STEP 6: Check the urine pH. Urine pH  5.0 (appropriate in the setting of acidemia) STEP 7: Generate the differential diagnosis. The differential diagnosis would include all the elevated-gap acidoses listed in Table 6-1. However, only two diagnoses on this list actually cause both an elevated-gap metabolic acidosis and a respiratory alkalosis: salicylate overdose and sepsis. Salicylate overdose causes a respiratory alkalosis by stimulation of the central respiratory centers and produces an elevated-gap metabolic lactic acidosis by poisoning the oxidative mechanisms within cells. If this patient had any of the other causes of an elevated-gap metabolic acidosis, you would need to consider one or more of the differential diagnoses found on the respiratory alkalosis list (see Table 6-6) as a concomitant process. The most likely diagnosis in this case is salicylate overdose. Note that because of the respiratory alkalosis, the pH is almost in the normal range. It is important to realize that in salicylate overdose (and in sepsis or other combinations of a metabolic acidosis and respiratory alkalosis), a quick look at the pH may falsely reassure you that the acidosis is not that “bad,” when in reality this patient has a severe metabolic acidosis. Acid–base disorder:

A mixed elevated-gap metabolic acidosis and respiratory alkalosis

Case 6a A 57-year-old patient with a long history of smoking presents to you in the clinic. He is not in distress, but he tells you he develops dyspnea on exertion. You send him for pulmonary function tests (PFTs) and an arterial blood gas (ABG).

Longenecker_CH07_052-078.qxd

64

10/12/06

2:04 PM

Page 64

CHAPTER 7 LABORATORY TEST RESULTS

Arterial blood gas

Serum chemistries

Urine tests

pH: 7.35 pCO2: 50 mm Hg [HCO3]: 27 mEq/L

[Na+]: 143 mEq/L [Cl]: 105 mEq/L

pH: 5.0

STEP 1: Evaluate the pH and narrow down to two possible processes. pH  7.36 Conclude: A metabolic or respiratory acidosis is present. STEP 2: Evaluate the pCO2 and narrow down to one definite process. pCO2  40 mm Hg Conclude: At least a respiratory acidosis is present. STEP 3: Apply the formula for a respiratory acidosis. Because the patient clinically does not have an acute process, use the formulas for a chronic respiratory acidosis. You can use either the pH or the [HCO3] to elevate this patient’s ABG. If you decide to use the pH:

• For every increase of 10 mm Hg in the pCO2, expect the pH to decrease (from 7.40) by 0.03. • Because the pCO2 increased by 10 mm Hg, the pH should decrease by 0.03 to a value of 7.37 (expected pH). • Expected pH  7.37 • Actual pH  7.35 If you decide to use the [HCO3ⴚ]:

• For every increase of 10 mm Hg in the pCO2, expect the [HCO3] to increase (from 24) by 4 mEq/L. • Because the pCO2 increased by 10 mm Hg, the [HCO3] should increase by 4 to a value of 28 mEq/L (expected [HCO3]). • Expected [HCO3ⴚ]  28 mEq/L • Actual [HCO3ⴚ]  27 mEq/L STEP 4: Determine if any other processes are present. The actual pH and [HCO3] closely match the expected changes, so you can conclude that a pure chronic respiratory acidosis is present. Note that the actual pH does not exactly match the expected pH. Because this difference is small, it falls within the range of error and is clinically insignificant. If the actual pH were 7.33 (with an expected value of 7.37) and the actual [HCO3] were 24 mEq/L, you could conclude that a small (perhaps clinically insignificant) metabolic acidosis was also present. If the pH were 7.32 or lower, and the [HCO3] were 23 mEq/L or lower, you could conclude that a moderate metabolic acidosis was present along with the chronic respiratory acidosis.

Longenecker_CH07_052-078.qxd

10/12/06

2:04 PM

Page 65

TUTORIAL

65

STEP 5: Evaluate the anion gap. Anion gap  143  (105  27)  11 (normal) STEP 6: Check the urine pH. Urine pH  5.0 (appropriate in the setting of acidemia) STEP 7: Generate the differential diagnosis. In this case, it is important to obtain a chest x-ray and PFTs to differentiate between obstructive and restrictive causes of a chronic respiratory acidosis. In this patient, the PFTs showed an obstructive defect. Acid–base disorder: Simple chronic respiratory acidosis

Case 6b The patient in Case 6a presents to the emergency room 1 month later in respiratory distress. He is wheezing and his respiratory rate is 33 breaths/minute.

LABORATORY TEST RESULTS Arterial blood gas

Serum chemistries

Urine tests

pH: 7.29 pCO2: 61 mm Hg [HCO3]: 29 mEq/L

[Na+]: 142 mEq/L [Cl]: 100 mEq/L

pH: 5.0

This case presents the problem of a mixed acute and chronic respiratory acidosis, a scenario you will see often in the emergency room. As you will see, the clinical setting and the previous ABG are essential in evaluating the current ABG. STEP 1: Evaluate the pH and narrow down to two possible processes. pH  7.36 Conclude: A metabolic or respiratory acidosis is present. STEP 2: Evaluate the pCO2 and narrow down to one definite process. pCO2  40 mm Hg Conclude: At least a respiratory acidosis is present. STEP 3: Apply the appropriate formula. We know that at baseline, this patient has a chronic respiratory acidosis and now presents with an acute episode of what appears to be an acute respiratory acidosis. Should we choose the acute or chronic formula? In a case like this, it is important to use both. By using both formulas, one can delineate all the possible combinations that could result in this ABG. Then, based on the clinical history, the best option is chosen. Again, for the respiratory disorders, you may choose either the pH or the [HCO3] to evaluate this ABG.

Longenecker_CH07_052-078.qxd

66

10/12/06

2:04 PM

Page 66

CHAPTER 7

EVALUATE USING THE FORMULA FOR A CHRONIC RESPIRATORY ACIDOSIS. If you decide to use the pH:

• For every increase of 10 mm Hg in the pCO2, expect the pH to decrease (from 7.40) by 0.03. • Because the pCO2 increased by approximately 20 mm Hg, the pH should decrease by 0.06 to a value of 7.34 (expected pH). • Expected pH  7.34 • Actual pH  7.29 If you decide to use the [HCO3ⴚ]:

• For every increase of 10 mm Hg in the pCO2, expect the [HCO3] to increase (from 24) by 4 mEq/L. • Because the pCO2 increased by approximately 20 mm Hg, the [HCO3] should increase by 8 to a value of 32 mEq/L (expected [HCO3]). • Expected [HCO3ⴚ]  32 mEq/L • Actual [HCO3ⴚ]  29 mEq/L Notice that both the actual pH and the actual [HCO3] are lower than the expected values. Remember that the expected pH and [HCO3] are the target values for full compensation by the kidneys, given this degree (i.e., pCO2  61 mm Hg) of hypoventilation. Therefore, two possible processes exist: 1. A mixed chronic respiratory acidosis and a small metabolic acidosis 2. A respiratory acidosis that is only partially compensated (i.e., an “acute-on-chronic” or a mixed acute and chronic respiratory acidosis) EVALUATE USING THE FORMULA FOR AN ACUTE RESPIRATORY ACIDOSIS. If you decide to use the pH:

• For every increase of 10 mm Hg in the pCO2, expect the pH to decrease (from 7.40) by 0.08. • Because the pCO2 increased by approximately 20 mm Hg, the pH should decrease by 0.16 to a value of 7.24 (expected pH). • Expected pH  7.24 • Actual pH  7.29 If you decide to use the [HCO3ⴚ]:

• For every increase of 10 mm Hg in the pCO2, expect the [HCO3] to increase (from 24) by 1 mEq/L. • Because the pCO2 increased by approximately 20 mm Hg, the [HCO3] should increase by 2 to a value of 26 mEq/L (expected [HCO3]). • Expected [HCO3ⴚ]  26 mEq/L • Actual [HCO3ⴚ]  29 mEq/L Notice that the actual pH and [HCO3] values are higher than the expected values. Remember that the expected values are those that would occur if only an acute respiratory acidosis were present. Therefore, we can conclude that one of two processes is present: 1. A mixed acute respiratory acidosis and a small metabolic alkalosis 2. A respiratory acidosis that is only partially compensated (i.e., an “acute-on-chronic” or mixed acute and chronic respiratory acidosis)

Longenecker_CH07_052-078.qxd

10/12/06

2:04 PM

Page 67

TUTORIAL

67

STEP 4: Determine if any other processes are present. Summarizing Step 3, it is evident that this particular ABG (i.e., pH, 7.29; pCO2, 61; [HCO3], 29) can result from three different processes: • A chronic respiratory acidosis mixed with a small metabolic acidosis • An acute respiratory acidosis mixed with a small metabolic alkalosis • A respiratory acidosis that is not fully compensated (or an “acute-on-chronic” respiratory acidosis) Choosing among these three possibilities requires the clinical history. The current patient had a baseline chronic respiratory acidosis and now presents with an acute exacerbation of chronic obstructive pulmonary disease (COPD). Therefore, the third option is correct. Note that the other two choices represent situations in which the respiratory acidosis (chronic and acute, respectively) is the predominant process. In fact, the metabolic components in those cases are so small that they would probably not be considered clinically significant. STEP 5: Evaluate the anion gap. Anion gap  142  (100  29)  13 (normal) STEP 6: Check the urine pH. Urine pH  5.0 (appropriate in the setting of acidemia) STEP 7: Generate the differential diagnosis. The diagnosis is probable COPD exacerbation, although other pulmonary processes such as pneumonia, pulmonary embolism, pulmonary edema, and pneumothorax also would need to be considered. Acid–base disorder: Mixed acute and chronic (“acute-on-chronic”) respiratory acidosis

Case 7 A 45-year-old diabetic patient presents with obtundation. LABORATORY TEST RESULTS Arterial blood gas

pH: 7.01 pCO2: 80 mm Hg [HCO3]: 20 mEq/L

Serum chemistries +

[Na ]: 140 mEq/L [K]: 5.5 mEq/L [Cl]: 97 mEq/L

STEP 1: Evaluate the pH and narrow down to two possible processes. pH  7.36 Conclude: A respiratory or a metabolic acidosis is present. STEP 2: Evaluate the pCO2 and narrow down to one definite process. pCO2  40 mm Hg Conclude: At least a respiratory acidosis is present.

Urine tests

pH: 5.0

Longenecker_CH07_052-078.qxd

68

10/12/06

2:04 PM

Page 68

CHAPTER 7

STEP 3: Apply the formula for an acute respiratory acidosis and decide if you will use the pH or the [HCO3] or both for your equation. If you decide to use the pH:

• For every increase of 10 mm Hg in the pCO2, expect the pH to decrease (from 7.40) by 0.08. • Because the pCO2 increased by 40 mm Hg, the pH should decrease by 0.32 to a value of 7.08 (expected pH). • Expected pH  7.08 • Actual pH  7.01 If you decide to use the [HCO3ⴚ]:

• For every increase of 10 mm Hg in the pCO2, expect the [HCO3] to increase (from 24) by 1 mEq/L. • Because the pCO2 increased by 40 mm Hg, the [HCO3] should increase by 4 to 28 mEq/L (expected [HCO3]). • Expected [HCO3ⴚ]  28 mEq/L • Actual [HCO3ⴚ]  20 mEq/L STEP 4: Determine if any other processes are present. Both the pH and the [HCO3] are lower than predicted by the formulas. This finding suggests that in addition to the acute respiratory acidosis, an acute metabolic acidosis is also present (the only process that could lower the pH and [HCO3] beyond the expected values). It is important to remember, however, that the reliability of these formulas decreases as pH values become more extreme. The fact that the [HCO3] is lower than normal is very helpful in this case. Clearly, the patient has a severe respiratory acidosis. However, if the respiratory acidosis were the only process present, we would expect the [HCO3] to be elevated. The fact that the [HCO3] is lower than normal indicates that a metabolic acidosis is present in addition to the respiratory acidosis. STEP 5: Evaluate the anion gap. Anion gap  140  (97  20)  23 Conclude: An elevated-gap metabolic acidosis is also present. STEP 6: Check the urine pH. Urine pH  5.0 (appropriate in the setting of acidemia) STEP 7: Generate the differential diagnosis. The differential diagnosis is very wide for this critically ill patient and should include all of the entities listed in Tables 6-1 and 6-5. It should be evident that the respiratory acidosis is the predominant acid–base disorder present. However, the cause of the moderate elevated-gap metabolic acidosis should also be investigated seriously, with diabetic ketoacidosis (DKA) being the primary suspect. Acid–base disorder: A mixed acute respiratory acidosis and an elevated-gap metabolic acidosis

Case 8 A 78-year-old nursing home patient has been vomiting for several days and has rapidly developed a fever and increasing shortness of breath over the past several hours. Her respiratory rate is 35 breaths/minute, and she has consolidative signs in the right base of the lung.

Longenecker_CH07_052-078.qxd

10/12/06

2:04 PM

Page 69

TUTORIAL

69

LABORATORY TEST RESULTS Arterial blood gas

Serum chemistries

Urine tests

pH: 7.69 pCO2: 20 mm Hg [HCO3]: 28 mEq/L

[Na+]: 138 mEq/L [Cl]: 97 mEq/L

pH: 8.0

STEP 1: Evaluate the pH and narrow down to two possible processes. pH  7.44 Conclude: A respiratory or metabolic alkalosis is present. STEP 2: Evaluate the pCO2 and narrow down to one definite process. pCO2  40 mm Hg Conclude: At least a respiratory alkalosis is present. STEP 3: Apply the formula for an acute respiratory alkalosis and decide if you will use the pH or the [HCO3] or both for your equation. If you decide to use the pH:

• For every decrease of 10 mm Hg in the pCO2, expect the pH to increase (from 7.40) by 0.08. • Because the pCO2 decreased by 20 mm Hg, the pH should increase by 0.16 to a value of 7.56 (expected pH). • Expected pH  7.56 • Actual pH  7.69 If you decide to use the [HCO3ⴚ]:

• For every decrease of 10 mm Hg in the pCO2, expect the [HCO3] to decrease (from 24) by 2 mEq/L. • Because the pCO2 decreased by 20 mm Hg, the [HCO3] should decrease by 4 to a value of 20 mEq/L (expected [HCO3]). • Expected [HCO3ⴚ]  20 mEq/L • Actual [HCO3ⴚ]  28 mEq/L STEP 4: Determine if any other processes are present. Both the actual pH and the actual [HCO3] are higher than would be expected for an acute respiratory alkalosis. Only a metabolic alkalosis could cause this increase in the pH and [HCO3]. Therefore, you can conclude that the patient has a mixed respiratory alkalosis (acute) and metabolic alkalosis. STEP 5: Evaluate the anion gap. Anion gap  138  (97  28)  13 (normal, as expected) STEP 6: Check the urine pH. Urine pH  8.0 (appropriate in the setting of alkalemia)

Longenecker_CH07_052-078.qxd

70

10/12/06

2:04 PM

Page 70

CHAPTER 7

STEP 7: Generate the differential diagnosis. Severe alkalemia is a life-threatening emergency. In addition to causing cerebral vasoconstriction, muscular tetany, and seizures, it can result in refractory ventricular arrhythmias. The metabolic alkalosis is likely due to the patient’s protracted emesis and probable volume depletion. The respiratory alkalosis may be due to an aspiration pneumonia or other acute pulmonary process [e.g., pulmonary embolism, pneumothorax, congestive heart failure (CHF)]. It would be important to obtain serial K levels while the patient is rehydrated with normal saline, as hypokalemia is common and severe in this situation. Acid–base disorder: A mixed acute respiratory alkalosis and metabolic alkalosis

Case 9 A 20-year-old diabetic patient presents with nausea and vomiting for several days, and with fever and shortness of breath that have developed over the past 8 hours. LABORATORY TEST RESULTS Arterial blood gas

Serum chemistries

Urine tests

pH: 7.59 pCO2: 25 mm Hg [HCO3]: 24 mEq/L

[Na+]: 140 mEq/L [Cl]: 95 mEq/L [Glucose]: 610 mg/dL BUN: 26 mg/dL

pH: 8.0 Ketones: positive

STEP 1: Evaluate the pH and narrow down to two possible processes. pH  7.44 Conclude: A metabolic or respiratory alkalosis is present. STEP 2: Evaluate the pCO2 and narrow down to one definite process. pCO2  40 mm Hg Conclude: At least a respiratory alkalosis is present. STEP 3: Apply the formula for an acute respiratory alkalosis. Decide if you will use the pH or the [HCO3] or both for your equation. If you decide to use the pH:

• For every decrease of 10 mm Hg in the pCO2, expect the pH to increase (from 7.40) by 0.08. • Because the pCO2 decreased by 15 mm Hg (1.5 “tens”), the pH should increase by 0.12 to a value of 7.52 (expected pH). • Expected pH  7.52 • Actual pH  7.59 If you decide to use the [HCO3ⴚ]:

• For every decrease of 10 mm Hg in the pCO2, expect the [HCO3] to decrease (from 24) by 2 mEq/L. • Because the pCO2 decreased by 15 mm Hg (1.5 “tens”), the [HCO3] should decrease by 3 to a value of 21 mEq/L (expected [HCO3]).

Longenecker_CH07_052-078.qxd

10/12/06

2:04 PM

Page 71

TUTORIAL

71

• Expected [HCO3ⴚ]  21 mEq/L • Actual [HCO3ⴚ]  24 mEq/L STEP 4: Determine if any other processes are present. Both the pH and the [HCO3] are higher than would be expected for a simple acute respiratory alkalosis. Only a metabolic alkalosis could cause this increase in pH and [HCO3]. Therefore, you can conclude that the patient has a mixed respiratory alkalosis and metabolic alkalosis. STEP 5: Check the anion gap. Remember that in cases of hyperglycemia, the [Na] is falsely low. This must be corrected for: Corrected [Na]  0.016 ([glucose]  100)  [serum Na] Corrected [Na]  0.016 (610  100)  140  148 mEq/L Anion gap  148  (95  24)  29 The anion gap is elevated significantly. A metabolic acidosis is by far the most common cause of an increased anion gap, although other less common causes of an increased anion gap should at least be considered (see Table 1-3). By your evaluation thus far, you have diagnosed a respiratory alkalosis and a metabolic alkalosis. Because of the increased anion gap (assuming you did not find a different cause for the increased gap), you should use the “delta–delta” calculation to evaluate for a triple acid–base disorder (see Chapter 4). First, calculate the amount by which the gap increased above its usual value of 12. The gap is 29, indicating a “delta” of 17. If the gap increased by 17, the [HCO3] should decrease by 17 from 24 mEq/L to a value of 7 mEq/L. Given this degree of an elevated-gap acidosis (gap  29), we would expect the [HCO3] to fall to 7. In addition, we also expect a further fall of 3 mEq/L in the [HCO3] due to the acute respiratory alkalosis. Given these two processes, we would expect the [HCO3] to be 4 mEq/L. The [HCO3] is in fact 24, resulting in a “delta–delta” discrepancy of 20. Because this discrepancy is greater than 8 mEq/L, a significant metabolic alkalosis is also present. You have diagnosed a triple acid–base disorder. Remember that it is impossible to diagnose a triple acid–base disorder in the absence of an elevated-gap metabolic acidosis. Note also that the blood urea nitrogen (BUN) is 26 mg/dL. Remember that the “delta–delta” calculation becomes less reliable as the BUN increases above 23 mg/dL; therefore, you would need to use it cautiously in this case. Nonetheless, given the very large “delta–delta” discrepancy, it probably is still safe to use it because the BUN is only slightly above the limit of 23 mg/dL. STEP 6: Check the urine pH. Urine pH  8.0 (appropriate in the setting of alkalemia) STEP 7: Generate the differential diagnosis. It is important to remember that each disorder diagnosed needs an explanation. The respiratory alkalosis may be caused by pneumonia, pulmonary edema, or another pulmonary process. The metabolic alkalosis is likely caused by a contraction alkalosis secondary to the emesis. The elevated-gap metabolic acidosis is likely due to diabetic ketoacidosis (DKA). You would need to order a full range of diagnostic tests to investigate the causes of all three acid–base disorders. Acid–base disorder: A triple acid–base disorder: • Respiratory alkalosis • Metabolic alkalosis • Elevated-gap metabolic acidosis

Longenecker_CH07_052-078.qxd

72

10/12/06

2:04 PM

Page 72

CHAPTER 7

Case 10 A 47-year-old man with alcoholism presents with vomiting after binge drinking for the past 2 days. He is brought to the emergency room by his friend, who tells you he took a large number of diazepam pills 1 hour prior and became “very sleepy.” His respiratory rate is 8 breaths/minute and he is unresponsive. LABORATORY TEST RESULTS Arterial blood gas

Serum chemistries

Urine tests

pH: 7.27 pCO2: 62 mm Hg [HCO3]: 28 mEq/L

[Na+]: 135 mEq/L [Cl]: 85 mEq/L BUN: 21 mg/dL

pH: 5.0

STEP 1: Evaluate the pH and narrow down to two possible processes. pH  7.44 Conclude: A metabolic or respiratory acidosis is present. STEP 2: Evaluate the pCO2 and narrow down to one definite process. pCO2  40 mm Hg Conclude: At least a respiratory acidosis is present. STEP 3: Apply the formula for an acute respiratory acidosis and decide if you will use the pH or the [HCO3] or both for your equation. If you decide to use the pH:

• For every increase of 10 mm Hg in the pCO2, expect the pH to decrease (from 7.40) by 0.08. • Because the pCO2 increased by 20 mm Hg, the pH should decrease by 0.16 to a value of 7.24 (expected pH). • Expected pH  7.24 • Actual pH  7.27 If you decide to use the [HCO3ⴚ]:

• For every increase of 10 mm Hg in the pCO2, expect the [HCO3] to increase (from 24) by 1 mEq/L. • Because the pCO2 increased by 20 mm Hg, the [HCO3] should increase by 2 to a value of 26 mEq/L (expected [HCO3]). • Expected [HCO3ⴚ]  26 mEq/L • Actual [HCO3ⴚ]  28 mEq/L STEP 4: Determine if any other processes are present. Note that the actual pH and [HCO3] are both slightly higher than expected. In addition to the significant respiratory acidosis, a small metabolic alkalosis may also be present. STEP 5: Evaluate the anion gap. Anion gap  135  (28  85)  22

Longenecker_CH07_052-078.qxd

10/12/06

2:04 PM

Page 73

TUTORIAL

73

Conclude: A moderate elevated-gap metabolic acidosis is also present (assuming you rule out the less common causes of an increased gap in Table 1-3). This surprise finding significantly changes our conclusions from Step 4. It appears that a triple acid–base disorder may be present, necessitating evaluation using the “delta–delta” calculation. We know that the gap increased by 10 mEq/L (from 12 to 22). We would therefore expect the [HCO3] to fall by approximately 10 mEq/L (from 24 down to a value of 14 mEq/L). In addition, we would expect the [HCO3] to rise by 2 mEq/L because of the acute respiratory acidosis. Taking into account this contribution of the respiratory acidosis, we would expect the [HCO3] to be approximately 16 mEq/L. In fact, the [HCO3] is 28 mEq/L, reflecting a “delta–delta” disparity of 12 mEq/L, which in turn is explained by the presence of a significant metabolic alkalosis. Because this discrepancy is greater than 8 mEq/L, you can conclude that a triple acid–base disorder probably exists. If the discrepancy had been less than 8 mEq/L, the differences seen could have arisen from random error. The largest and most obvious acid–base disorder present is the respiratory acidosis. Although the metabolic acidosis and the metabolic alkalosis are smaller than the respiratory acidosis, they are both present to a significant degree. Both should be diagnosed and treated, in addition to the more obvious respiratory acidosis. It is important to add that if you did not know whether the respiratory acidosis was acute or chronic, it would be impossible to diagnose a triple acid–base disorder with confidence. Without this knowledge, you would not know how to predict the incremental change in the [HCO3] contributed by the compensatory response to the respiratory acidosis (a significant difference between the acute and chronic settings). STEP 6: Check the urine pH. Urine pH  5.0 (appropriate in the setting of acidemia) STEP 7: Generate the differential diagnosis. The most likely cause of the respiratory acidosis is respiratory depression from alcohol intoxication and benzodiazepine overdose, although other causes should be investigated (e.g., intracerebral hemorrhage or pulmonary conditions). The mild metabolic alkalosis is likely due to emesis and resultant volume depletion. The hyponatremia and hypochloremia reflect some dehydration as well. The elevated-gap metabolic acidosis is likely caused by an alcoholic ketoacidosis (AKA), although other causes of an elevated-gap metabolic acidosis should be investigated. It is important to remember to give thiamine prior to starting glucose infusion (to avoid lactic acidosis and Wernicke-Korsakoff syndrome.) Acid–base disorder: A triple acid–base disorder: • Respiratory acidosis • Elevated-gap metabolic acidosis • Metabolic alkalosis

Case 11 A 65-year-old man with chronic obstructive pulmonary disease (COPD) and congestive heart failure (CHF) presents with increasing shortness of breath and wheezing for the past 4 hours. He is currently on furosemide.

Longenecker_CH07_052-078.qxd

74

10/12/06

2:04 PM

Page 74

CHAPTER 7 LABORATORY TEST RESULTS

Arterial blood gas

Serum chemistries

Urine tests

pH: 7.40 pCO2: 60 mm Hg [HCO3]: 37 mEq/L

[Na+]: 140 mEq/L [Cl]: 90 mEq/L

pH: 5.0

This case demonstrates the approach to an abnormal arterial blood gas (ABG), which also has a normal pH. Because the approach presented in Figure 5-1 cannot be used in this context, we will use the approach illustrated in Figure 5-2. Note that Step 2 includes evaluation of [HCO3], which was not necessary for Step 2 in Cases 1 through 10. STEP 1: Evaluate the pH in light of the pCO2 and [HCO3]. pH  7.40 Conclude: Because the pCO2 and the [HCO3] are abnormal in the setting of a normal pH, we can conclude immediately that: • A mixed acid–base disorder is present. • At least one acidosis and one alkalosis are present. • All alkaloses are exactly balancing all acidoses present. STEP 2: Evaluate the pCO2 and [HCO3]. Evaluate the pCO2. pCO2  40 mm Hg First, determine which processes can elevate the pCO2 above 40 mm Hg. These processes include: • Respiratory acidosis • Metabolic alkalosis Evaluate the [HCO3ⴚ]. [HCO3]  24 mEq/L Then, determine which processes can elevate the [HCO3] above 24 mEq/L. These processes include: • Respiratory acidosis • Metabolic alkalosis STEP 3: Apply the appropriate formulas. At this point, given the conclusions to Steps 1 and 2, it is reasonable to hypothesize that a respiratory acidosis and a metabolic alkalosis are present and are balancing each other to a pH of 7.40. Therefore, you can choose either formula and test your hypothesis. It does not matter which of the two formulas you start with. If you choose the formula for a metabolic alkalosis: • Expected pCO2  0.9 [HCO3]  16  0.9 (37)  16  49 mm Hg • Actual pCO2  60 mm Hg Although this formula is not very reliable, it is clear that the actual pCO2 (60 mm Hg) is much higher than expected. The only other process that could increase the pCO2 above the expected value is a respiratory acidosis. Therefore, you can conclude that a mixed disorder (a respiratory acidosis and metabolic alkalosis) is present.

Longenecker_CH07_052-078.qxd

10/12/06

2:04 PM

Page 75

TUTORIAL

75

IF YOU CHOOSE THE FORMULAS FOR A RESPIRATORY ACIDOSIS: We know that at baseline, this patient has a chronic respiratory acidosis and now presents with an acute episode of what appears to be an acute respiratory acidosis. Should we choose the acute or chronic formula? In a case like this, it is important to use both. By using both formulas, one can delineate all the possible combinations that could result in this ABG. Then, based on the clinical history, one can choose the best option. Again, for the respiratory disorders, you may choose either the pH or the [HCO3] to evaluate this ABG. EVALUATE USING THE FORMULA FOR A CHRONIC RESPIRATORY ACIDOSIS. If you decide to use the pH: • For every increase of 10 mm Hg in the pCO2, expect the pH to decrease (from 7.40) by 0.03. • Because the pCO2 increased by 20 mm Hg, the pH should decrease by 0.06 to a value of 7.34 (expected pH). • Expected pH  7.34 • Actual pH  7.40 If you decide to use the [HCO3ⴚ]: • For every increase of 10 mm Hg in the pCO2, expect the [HCO3] to increase (from 24) by 4 mEq/L. • Because the pCO2 increased by 20 mm Hg, the [HCO3] should increase by 8 to a value of 32 mEq/L (expected [HCO3]). • Expected [HCO3ⴚ]  32 mEq/L • Actual [HCO3ⴚ]  mEq/L Notice that both the pH and [HCO3] are higher than the expected values. Remember that the expected pH and [HCO3] are the target values for full compensation by the kidneys, given this degree (i.e., pCO2  60 mm Hg) of hypoventilation. Therefore, two mechanisms could account for these findings: 1. A mixed chronic respiratory acidosis and a mild metabolic alkalosis (clinically, it is possible but unusual for a patient to chronically have a pCO2 of 60 mm Hg) 2. A mixed acute and chronic (i.e., not yet fully compensated, or “acute-on-chronic”) respiratory acidosis and a moderate metabolic alkalosis EVALUATE USING THE FORMULA FOR AN ACUTE RESPIRATORY ACIDOSIS. If you decide to use the pH: • For every increase of 10 mm Hg in the pCO2, expect the pH to decrease (from 7.40) by 0.08. • Because the pCO2 increased by 20 mm Hg, the pH should decrease by 0.16 to a value of 7.24 (expected pH). • Expected pH  7.24 • Actual pH  7.40 If you decide to use the [HCO3ⴚ]: • For every increase of 10 mm Hg in the pCO2, expect the [HCO3] to increase (from 24) by 1 mEq/L. • Because the pCO2 increased by 20 mm Hg, the [HCO3] should increase by 2 to a value of 26 mEq/L (expected [HCO3]). • Expected [HCO3ⴚ]  26 mEq/L • Actual [HCO3ⴚ]  37 mEq/L

Longenecker_CH07_052-078.qxd

76

10/12/06

2:04 PM

Page 76

CHAPTER 7

Note that the pH and [HCO3] are much higher than the expected values. If the respiratory acidosis is acute, then a mixed disorder is present. Two mechanisms are possible: 1. A mixed acute respiratory acidosis and a severe metabolic alkalosis 2. A mixed “acute-on-chronic” (i.e., not yet fully compensated) respiratory acidosis and a moderate metabolic alkalosis STEP 4: Determine if a mixed acid–base disorder is present. Summarizing Step 3, it is evident that this particular ABG (i.e., pH, 7.40; pCO2, 60; [HCO3], 37) can result from three different processes: • A mixed chronic respiratory acidosis and a mild metabolic alkalosis • A mixed acute respiratory acidosis and a severe metabolic alkalosis • A mixed “acute-on-chronic” (i.e., not yet fully compensated) respiratory acidosis and a moderate metabolic alkalosis Choosing among these three possibilities requires the clinical history. The current patient had a baseline chronic respiratory acidosis and now presents with an acute exacerbation of COPD. Therefore, the third option is correct. Because the pH is 7.40, it is clear that the respiratory acidosis exactly balances out the metabolic alkalosis. STEP 5: Evaluate the anion gap. Anion gap  140  (90  37)  13 (normal, as expected) STEP 6: Check the urine pH. Urine pH  5.0 (appropriate in the setting of a pH of 7.40) STEP 7: Generate the differential diagnosis. The patient has a known history of COPD and presents with worsening shortness of breath and CO2 retention. The most likely diagnosis is a COPD exacerbation. With regard to the metabolic alkalosis, we are told the patient is on furosemide for CHF. It is common to see patients with COPD and CHF present with diuretic-induced contraction metabolic alkalosis. Acid–base disorder: A mixed “acute-on-chronic” respiratory acidosis and a metabolic alkalosis, of equal severity

Case 12 A 27-year-old diabetic patient presents with 1 hour of acute shortness of breath. He has been nauseated and has had increased urination over the past 2 days. Because of his nausea, he decided not to take his insulin and has been bedridden the past 2 days. In the emergency room, you discover he has a family history of a hypercoagulable disorder.

LABORATORY TEST RESULTS Arterial blood gas

pH: 7.40 pCO2: 20 mm Hg [HCO3]: 12 mEq/L

Serum chemistries +

[Na ]: 136 mEq/L [Cl]: 102 mEq/L [Glucose]: 400 mg/dL

Urine tests

pH: 5.0 Ketones: positive

Longenecker_CH07_052-078.qxd

10/12/06

2:04 PM

Page 77

TUTORIAL

77

Because the approach in Figure 5-1 cannot be used in this context, we will use the approach illustrated in Figure 5-2. Note that Step 2 includes evaluation of [HCO3], which was not necessary for Step 2 in Cases 1 through 10. STEP 1: Evaluate the pH in light of the pCO2 and [HCO3]. pH  7.40 Conclude: Because the pCO2 and the [HCO3] are abnormal in the setting of a normal pH, we can conclude immediately that: • A mixed acid–base disorder is present. • At least one acidosis and one alkalosis are present. • All alkaloses are exactly balancing all acidoses present. STEP 2: Evaluate the pCO2 and [HCO3]. Evaluate the pCO2. pCO2  40 mm Hg First, determine the processes that drive the pCO2 below 40 mm Hg. These processes include: • Respiratory alkalosis • Metabolic acidosis Evaluate the [HCO3ⴚ]. [HCO3]  24 mEq/L Then, determine which processes can decrease the [HCO3] below 24 mEq/L. These processes include: • Respiratory alkalosis • Metabolic acidosis STEP 3: Apply the appropriate formula. At this point, given the findings in Steps 1 and 2, it is reasonable to hypothesize that a respiratory alkalosis and a metabolic acidosis are present and are balancing each other to a pH of 7.40. Therefore, you can choose either formula and test your hypothesis. It does not matter which of the two formulas you start with—the same answer will result. Given the clinical history of an acute onset of shortness of breath, you can assume the respiratory alkalosis is acute. IF YOU CHOOSE THE FORMULA FOR A METABOLIC ACIDOSIS: • Expected pCO2  1.5 [HCO3]  8  1.5 (12)  8  26 mm Hg • Actual pCO2  20 mm Hg The actual pCO2 is lower than the expected value. A respiratory alkalosis is present in addition to the metabolic acidosis. You have diagnosed a mixed disorder. Conclude: This patient has a mixed metabolic acidosis and an acute respiratory alkalosis. IF YOU CHOOSE THE FORMULA FOR ACUTE RESPIRATORY ALKALOSIS: If you decide to use the pH:

• For every decrease of 10 mm Hg in the pCO2, expect the pH to increase (from 7.4) by 0.08. • Because the pCO2 decreased by 20 mm Hg, the pH should increase by 0.16 to a value of 7.56 (expected pH).

Longenecker_CH07_052-078.qxd

78

10/12/06

2:04 PM

Page 78

CHAPTER 7

• Expected pH  7.56 • Actual pH  7.40 If you decide to use the [HCO3ⴚ]:

• For every decrease of 10 mm Hg in the pCO2, expect the [HCO3] to decrease (from 24) by 2 mEq/L. • Because the pCO2 decreased by 20 mm Hg, the [HCO3] should decrease by 4 to a value of 20 mEq/L (expected [HCO3]). • Expected [HCO3ⴚ]  20 mEq/L • Actual [HCO3ⴚ]  12 mEq/L STEP 4: Determine if any other processes are present. Both the actual pH and actual [HCO3] values are lower than the expected values. Only a metabolic acidosis could cause this change; therefore, you have diagnosed a mixed disorder. Conclude: This patient has a mixed acute respiratory alkalosis and a metabolic acidosis. Note that this is the same conclusion that results from using the formula for the metabolic acidosis. STEP 5: Evaluate the anion gap. Corrected [Na]  0.016 ([glucose]  100)  [serum Na] Corrected [Na]  0.016 (400  100)  136  141 mEq/L Anion gap  141  (102  12)  27 Conclude: An elevated-gap metabolic acidosis is present. Remember to correct for falsely low [Na] in cases of hyperglycemia. STEP 6: Check the urine pH. Urine pH  5.0 (appropriate in the setting of acidemia) STEP 7: Generate the differential diagnosis. The elevated-gap metabolic acidosis is most likely diabetic ketoacidosis (DKA), given that this diabetic patient has not been taking his insulin. The acute respiratory alkalosis may be caused by a pulmonary embolus, given the family history of a hypercoagulable disorder and the patient’s history of 2 days of bed rest. A full evaluation, including a chest x-ray and possibly a pulmonary ventilation/perfusion (V/Q) scan, is warranted. Because the pH is 7.40, it is clear that the metabolic acidosis exactly balances the respiratory alkalosis. Acid–base disorder: A mixed elevated-gap metabolic acidosis and a respiratory alkalosis, of equal severity

Longenecker_APP_79-80.qxd

10/7/06

3:00 PM

Page 79

Appendix

List of Abbreviations A⫺ ABG AChR ACTH ADH AKA Al ALS ATP B⫺ BUN C3 C5 Ca⫹⫹ CaCl CHF Cl⫺ CNS CO2 COPD Cr CT CVA DKA ECF FFAs GFR H⫹ HA HB HCG HCl HCO3⫺ H2CO3 H2O HPO42⫺ H2PO4⫺ IV K⫹

Anion Arterial blood gas Acetylcholine receptor Adrenocorticotropic hormone Alcohol dehydrogenase Alcoholic ketoacidosis Aluminum Amyotrophic lateral sclerosis Adenosine triphosphate Buffering anion Blood urea nitrogen Third cervical vertebrae Fifth cervical vertebrae Calcium ion Calcium chloride Congestive heart failure Chloride ion Central nervous system Carbon dioxide Chronic obstructive pulmonary disease Creatinine Computed tomography scan (CAT scan) Cerebrovascular accident Diabetic ketoacidosis Extracellular fluid Free fatty acids Glomerular filtration rate Hydrogen ion Acid with counterbalancing anion A⫺ (strong acid) Acid with counterbalancing anion B⫺ (buffer) Human chorionic gonadotropin Hydrochloric acid Bicarbonate Carbonic acid Water Divalent inorganic phosphate ion Monovalent inorganic phosphate ion Intravenous Potassium ion 79

Longenecker_APP_79-80.qxd

80

10/7/06

3:00 PM

Page 80

APPENDIX

LFTs mEq mEq/L Mg⫹⫹ mg/dL mL/min mm Hg mmol mOsm mOsm/kg MRI Na⫹ NaCl NAD NADH nEq nEq/L NG NH3 NH4⫹ pCO2 PFTs pH pK pO2 PRBCs PT PTT RBC RTA TPN TSH V/Q scan

Liver function tests (transaminases) Milliequivalents Milliequivalents per liter Magnesium ion Milligrams per deciliter Milliliters per minute Millimeters of mercury Millimoles Milliosmoles Milliosmoles per kilogram Magnetic resonance imaging Sodium ion Sodium chloride Nicotinamide adenine dinucleotide Reduced nicotinamide adenine dinucleotide Nanoequivalents Nanoequivalents per liter Nasogastric Ammonia Ammonium ion Partial pressure of carbon dioxide Pulmonary function tests Negative logarithm of the hydrogen ion concentration The pH at which an acid is 50% dissociated Partial pressure of oxygen Packed red blood cells Prothrombin time Activated partial thromboplastin time Red blood cell Renal tubular acidosis Total parenteral nutrition Thyroid-stimulating hormone Ventilation/perfusion scan

Longenecker_Sugg_081-085.qxd

10/12/06

2:05 PM

Page 81

Suggested Reading CHAPTER 1 General acid–base references

1. Schrier RW: Renal and Electrolyte Disorders. Boston, Little, Brown, 1997. A comprehensive, easy-to-read textbook on renal disorders, including a great chapter on acid–base pathophysiology. 2. Narins RG, Emmett M: Simple and mixed acid–base disorders: a practical approach. Medicine 59(3):161–187, 1980. A classic introductory reference on acid–base disorders. 3. Bia M, Thier SO: Mixed acid–base disturbances: a clinical approach. Med Clin North Am 65(2):347–361, 1981. Provides an excellent description of various mixed acid–base disorders and their treatment. 4. Riley LJ, Ilson BE, Narins RG: Acute metabolic acid–base disorders. Crit Care Clin 5(4):699–723, 1987. An in-depth review of metabolic acidoses and alkaloses and their treatment. 5. McLaughlin ML, Kassirer JP: Rational treatment of acid–base disorders. Drugs 39(6):841–855, 1990. A concise overview of the treatment of various acid–base disorders. 6. Preuss HG: Fundamentals of clinical acid–base evaluation. Clin Lab Med 13(1):103–116, 1993. A brief overview of clinical acid–base evaluation. A helpful conceptual framework is provided. Acid–base physiology

7. Fernandez PC, Cohen RM, Feldman GM: The concept of bicarbonate distribution space: the crucial role of body buffers. Kidney Int 36:747–752, 1989. A basic-science approach to understanding the bicarbonate and nonbicarbonate buffer systems. 8. Green J, Kleeman CR: The role of bone in regulation of systemic acid–base balance. Kidney Int 39:9–26, 1991. An in-depth physiologic review of the important role of bone in acid–base physiology. 9. Lowenstein J: Acid and Basics: A Guide to Understanding Acid–Base Disorders. New York, Oxford University Press, 1993. A cellular and biochemical approach to acid–base physiology. Excellent explanation of acid production and elimination. 10. Vander AJ: Renal Physiology. New York, McGraw-Hill, 1995. A concise textbook on renal physiology that uses diagrams and graphs liberally.

81

Longenecker_Sugg_081-085.qxd

82

10/12/06

2:05 PM

Page 82

SUGGESTED READING

The anion gap controversy (see also references 32–37)

11. Emmett M, Narins RG: Clinical use of the anion gap. Medicine 56(1):38–54, 1977. A classic reference on use of the anion gap. Reviews numerous metabolic acid–base disorders. 12. Gabow PA, Kaehny WD, Fennessey PV, et al: Diagnostic importance of an increased serum anion gap. N Engl J Med 303:854–858, 1980. A study of 57 hospitalized patients with elevated serum anion gap. 13. Oh MS, Carroll HJ: The anion gap. N Engl J Med 297:814–817, 1977. A classic review article on the anion gap. 14. Salem MM, Mujais SK: Gaps in the anion gap. Arch Intern Med 152:1625–1629, 1992. Reviews the limitations of using the anion gap in the evaluation of acid–base disorders. 15. Winter SD, Pearson JR, Gabow PA, et al: The fall of the serum anion gap. Arch Intern Med 150:311–313, 1990. A study showing that the anion gap may be lower when evaluated with modern instrumentation. Urine electrolytes and urinary gap

16. Kamel SK, Ethier JH, Richardson RMA, et al: Urine electrolytes and osmolality: when and how to use them. Am J Nephrol 10:89–102, 1990. An in-depth review of the role of urine chemistry tests. 17. Batlle DC, Hizon M, Cohen E, et al: The use of the urinary anion gap in the diagnosis of hyperchloremic metabolic acidosis. N Engl J Med 318:594–598, 1988. A study of 46 patients showing a negative correlation between urinary anion gap and urinary ammonium level. CHAPTER 2 The primary acid–base syndromes

18. Halperin ML: Metabolic aspects of metabolic acidosis. Clin Invest Med 16(4):294–305, 1993. One of the leading experts in acid–base physiology discusses the approach to a patient with a metabolic acidosis. 19. Kearns T, Wolfson AB: Metabolic acidosis. Emerg Med Clin North Am 7(4):823–835, 1989. A good review of metabolic acidosis syndromes. 20. Sabatini S, Kurtzman NA: The maintenance of metabolic alkalosis: factors which decrease bicarbonate excretion. Kidney Int 25:357–361, 1984. A review of the various factors that maintain a metabolic alkalosis. 21. Cogan MG, Liu FY, Berger BE, et al: Metabolic alkalosis. Med Clin North Am 67(4):903–913, 1983. A dated but useful review of the mechanisms and treatment of various metabolic alkaloses. 22. Halperin ML, Scheich A: Should we continue to recommend that a deficit of KCl be treated with NaCl? A fresh look at chloride-depletion metabolic alkalosis. Nephron 67:263–269, 1994. A somewhat difficult-to-read but state-of-the-art discussion of chloride-depletion metabolic alkalosis. 23. Kaehny WD: Respiratory acid–base disorders. Med Clin North Am 67(4):915–927, 1983. A dated but useful review of the mechanisms and treatment of various respiratory acidoses and alkaloses.

Longenecker_Sugg_081-085.qxd

10/12/06

2:05 PM

Page 83

SUGGESTED READING

83

CHAPTER 3 Original derivations of formulae

24. Albert MS, Dell RB, Winters RW: Quantitative displacement of acid–base equilibrium in metabolic acidosis. Ann Intern Med 66(2):312–322, 1967. Original derivation of Winters’ formula. 25. van Ypersele de Strihou C, Frans A: The respiratory response to metabolic alkalosis and acidosis in disease. Clinical Science and Molecular Medicine 45(4):439–448, 1973. Original derivation of the formula for metabolic alkalosis. 26. Brackett NC, Cohen JJ, Schwartz WB: Carbon dioxide titration curve of normal man: effect of increasing degrees of acute hypercapnia on acid–base equilibrium. N Engl J Med 272:6–12, 1965. Original derivation of the formula for acute respiratory acidosis. 27. Engel K, Dell RB, Rahill WJ, et al: Quantitative displacement of acid–base equilibrium in chronic respiratory acidosis. J Appl Physiol 24(3):288–295, 1968. Original derivation of the formula for chronic respiratory acidosis. 28. Eichenholz A, Blumenthals AS, Walker FE: The pattern of compensatory response to chronic hypercapnia in patients with chronic obstructive pulmonary disease. J Lab Clin Med 68(2):265–278, 1966. Original derivation of the formula for chronic respiratory acidosis in chronic obstructive pulmonary disease. 29. Gledhill N, Beirne GJ, Dempsey JA: Renal response to short-term hypocapnia in man. Kidney Int 8:376–386, 1975. Original derivation of the formula for acute respiratory alkalosis. 30. Gennari FJ, Goldstein MB, Schwartz WB: The nature of the renal adaptation to chronic hypocapnia. J Clin Invest 51:1722–1730, 1972. Original derivation of the formula for chronic respiratory alkalosis. 31. Chiodi H: Respiratory adaptations to chronic high altitude hypoxia. J Appl Physiol 10:81–87, 1957. Original derivation of the formula for chronic respiratory alkalosis associated with high-altitude hypoxia.

CHAPTER 4 The delta–delta calculation controversy (see also references 11–15)

32. Goodkin DA, Krishna GG, Narins RG: The role of the anion gap in detecting and managing mixed metabolic acid–base disorders. Clin Endocrinol Metab 13(2):333–349, 1984. A classic review of the application of the delta–delta calculation. 33. Wrenn K: The delta gap: an approach to mixed acid–base disorders. Ann Emerg Med 19:1310–1313, 1990. An explanation of the use of the delta–delta calculation. Several cases are presented. 34. Perez GO, Oster JR: Acid–base disorders, Part II. Use of AG/HCO3⫺ in evaluating mixed acid–base disorders. A patient management problem. South Med J 79(7):882–886, 1986. Discussion questions regarding a complicated case of a diabetic patient and use of the delta–delta calculation. 35. DiNubile MJ: The increment in the anion gap: overextension of a concept? Lancet 2:951–953, 1988. A concise review of the common, flawed assumptions on which the delta–delta calculation is based.

Longenecker_Sugg_081-085.qxd

84

10/12/06

2:05 PM

Page 84

SUGGESTED READING

36. Paulson WD, Gadallah MF: Diagnosis of mixed acid–base disorders in diabetic ketoacidosis. Am J Med Sci 306(5):295–300, 1993. A study of the range of delta–delta values in patients with diabetic ketoacidosis compared with healthy patients. 37. Paulson WD: Anion gap-bicarbonate relation in diabetic ketoacidosis. Am J Med 81:995–1000, 1986. A study of the range of delta–delta values in patients with diabetic ketoacidosis compared with healthy patients. This study shows that the delta–delta calculation is not valid in diabetic patients with a blood urea nitrogen greater than 23 mg/dL. CHAPTER 6 Elevated-gap metabolic acidosis syndromes

38. Burkhart KK, Kulig KW: The other alcohols: methanol, ethylene glycol, and isopropanol. Emerg Med Clin North Am 8(4):913–925, 1990. A thorough, practical approach to the “other” alcohols, including their pathophysiology, differential diagnosis, and therapy. 39. Kruse JA: Methanol poisoning. Intensive Care Med 18:391–397, 1992. A well-written and well-organized review of the toxicology, clinical and laboratory findings, diagnosis, therapy, and prognosis of methanol poisoning. 40. Scully RE, Galdabini JJ, McNeely BU: Weekly clinicopathological exercises. CASE 38-1979. N Engl J Med 301(12):650–657, 1979. Discussion of a case of ethylene glycol poisoning. 41. Wrenn KD, Slovis CM, Minion GE, et al: The syndrome of alcoholic ketoacidosis. Am J Med 91:119–128, 1991. Discussion of a case series of 74 patients with alcoholic ketoacidosis and their associated clinical and laboratory findings. A review of alcoholic ketoacidosis is also included. 42. Fulop M: Alcoholic ketoacidosis. Endocrinol Metab Clin North Am 22(2):209–219, 1993. An excellent and detailed review of alcoholic ketoacidosis. 43. Adams SL: Alcoholic ketoacidosis. Emerg Med Clin North Am 8(4):749–759, 1990. An excellent review of alcoholic ketoacidosis from an emergency medicine perspective. 44. Schrier RW: Renal and Electrolyte Disorders. Boston, Little, Brown, 1992, pp 561–567, 187–189. A discussion of the uremic syndrome and its associated acid–base disturbances. 45. Malluche H, Faugere MC: Renal bone disease 1990: an unmet challenge for the nephrologist. Kidney Int 38:193–211, 1990. A comprehensive review and discussion of bone disease in chronic renal failure. 46. Lebovitz HE: Diabetic ketoacidosis. Lancet 345:767–772, 1995. A concise, practical review of the presentation, laboratory abnormalities, diagnosis, and therapy of diabetic ketoacidosis. 47. Foster DW, McGarry JD: The metabolic derangements and treatment of diabetic ketoacidosis. N Engl J Med 309(3):159–169, 1983. An excellent, thorough review of the metabolic pathways, clinical presentation, diagnosis, and treatment of diabetic ketoacidosis. 48. Stackpoole PW: Lactic acidosis. Endocrinol Metab Clin North Am 22(2):221–245, 1993. An excellent, in-depth review of the various causes, presentation, and therapy of lactic acidosis. 49. Thibault GE: Clinical problem solving: the landlady confirms the diagnosis. N Engl J Med 326:1272–1275, 1992. An instructive clinical approach to problem solving centered around a case of salicylate poisoning.

Longenecker_Sugg_081-085.qxd

10/12/06

2:05 PM

Page 85

SUGGESTED READING

85

50. Gabow PA, Anderson RJ, Potts DE, et al: Acid–base disturbances in the salicylate-intoxicated adult. Arch Intern Med 138:1481–1484, 1978. Describes a case series of 67 adults with salicylate intoxication. Normal-gap metabolic acidosis syndromes

51. Hall MC, Koch MO, McDougal WS: Metabolic consequences of urinary diversion through intestinal segments. Urol Clin North Am 18(4):725–735, 1991. A thorough review of the pathophysiology and complications of urinary diversions through the intestinal tract. 52. Smulders YM, Frissen PHJ, Slaats EH, et al: Renal tubular acidosis: pathophysiology and diagnosis. Arch Intern Med 156:1629–1636, 1996. An excellent overview of the pathophysiology and diagnosis of renal tubular acidosis disorders. Provides a nice algorithm for the diagnostic approach. 53. Lash JP, Arruda JAL: Laboratory evaluation of renal tubular acidosis. Clin Lab Med 13(1):117–129, 1993. Explains, in detail, the use of various laboratory tests in the diagnosis of renal tubular acidosis. Metabolic alkalosis syndromes

54. Clive DM: Bartter’s syndrome: the unsolved puzzle. Am J Kidney Dis 25(6):813–823, 1995. An in-depth review of the clinical features, diagnosis, and treatment options (or paucity thereof) for Bartter’s syndrome. 55. Bartter FC: Bartter’s syndrome: juxtaglomerular hyperplasia, normotension, hypokalemic alkalosis. JAMA 216(1):152, 1971. An answer from Dr. Bartter to a question regarding therapy for a patient with Bartter’s syndrome and hypothyroidism. 56. Oster JR: The binge-purge syndrome: a common albeit unappreciated cause of acid–base and fluid–electrolyte disturbances. South Med J 80(1):58–66, 1987. An excellent review reminding the clinician to think of bulimia when presented with a patient with various metabolic disorders. 57. Abreo K, Adlakha A, Kilpatrick S, et al: The milk-alkali syndrome: a reversible form of acute renal failure. Arch Intern Med 153:1005–1010, 1993. A report and review of five cases of milk-alkali syndrome. 58. Muldowney WP, Mazbar SA: Rolaids-Yogurt syndrome: a 1990’s version of milk-alkali syndrome. Am J Kidney Dis 27(2):270–272, 1996. A case is presented that reminds the reader that milk-alkali can result from various sources of calcium and alkali. 59. Young WF, Hogan MJ, Klee GG, et al: Primary aldosteronism: diagnosis and treatment. Mayo Clin Proc 65:96–110, 1990. A comprehensive review of the presentation, diagnosis, and therapy of primary aldosteronism. 60. White PC: Disorders of aldosterone biosynthesis and action. N Engl J Med 331:250–258, 1994. A review of the biochemical pathways involved in aldosterone synthesis and the diagnosis and treatment of the clinical syndromes of mineralocorticoid deficiency and excess. Respiratory acid–base syndromes (see also reference 23)

61. Hill NS: Fluid and electrolyte considerations in diuretic therapy for hypertensive patients with chronic obstructive pulmonary disease. Arch Intern Med 146:129–133, 1986. A practical commentary on some of the issues faced in patients with chronic obstructive pulmonary disease who often need to be on diuretics.

Longenecker_Index_086-088.qxd

10/11/06

5:02 PM

Page 86

Index A Abbreviations, list of, 79–80 ABG. See Arterial blood gas (ABG) Acetazolamide, ingestion of carbonic anhydrase inhibitors and, metabolic acidosis vs., 46 Acid(s) ingestion or infusion of, metabolic acidosis vs., 46 nonvolatile, 2, 3f elimination of, 3f, 4–6, 5f, 6f Acid–base disorders, 12–19, 13f–15f, 13t, 16t, 18f, 18t, 19f. See also specific disorders, e.g., Metabolic acidosis case examples, 52–78 differential diagnosis of, 40–51. See also specific disorders, e.g., Metabolic alkalosis metabolic acidosis, 40–46, 41t, 44t metabolic alkalosis, 46–48, 47t, 48t respiratory acidosis, 49–50, 49t, 50f respiratory alkalosis, 50t, 51 evaluation of formulas in, 26–27, 27t laboratory tests in, 8–11, 9f, 9t, 10t ABG, 8 base excess, 11 serum anion gap, 8–9, 9f, 9t, 10t serum osmolal gap, 9–10 urine anion gap and osmolal gap, 10–11 urine pH, 10 metabolic acidosis, 12–15, 13f–15f, 13t metabolic alkalosis, 15–17, 15f, 16t mixed, 28–34, 29f, 29t–31t, 32f. See also Mixed acid–base disorders possible combinations of, 29, 29t primary and compensatory changes in, 20, 21t respiratory alkalosis, 17–18, 18f, 18t triple acid–base disorder case examples, 70–73 identification of, 33–34 Acid–base physiology, 1–11, 2t, 3f, 5f–7f, 9f, 9t, 10t buffer system, 3f, 4 definitions related to, 1, 2t interface between respiratory and metabolic components, 7–8, 7f intracellular acid and base production, 2–4, 3f metabolic component, 3f, 4–6, 5f, 6f overview of, 1–8, 3f, 5f–7f respiratory component, 3f, 4 Acid–base problems, approach to, 35–39, 36f, 36t, 38f Acidemia, defined, 1, 2t Acidosis(es) defined, 1, 2t diabetic, metabolic acidosis vs., 41t, 42 hyperchloremic, 14 lactic, metabolic acidosis vs., 41t, 42–43 metabolic. See Metabolic acidosis renal tubular dysfunction and, metabolic acidosis vs., 44–45, 44t respiratory. See Respiratory acidosis AKA. See Alcoholic ketoacidosis (AKA) Alcoholic ketoacidosis (AKA), metabolic acidosis vs., 41–42, 41t

86

Alkalemia, defined, 1, 2t Alkali ingestion, with decreased GFR, in “saline—non-responsive” metabolic alkalosis, 48, 48t Alkalosis(es) contraction, 16t, 17 defined, 1, 2t metabolic. See Metabolic alkalosis respiratory. See Respiratory alkalosis Alveolar ventilation, 3f, 4 Anion gap, 8–9, 9f decreased, causes of, 9, 10t increased, causes of, 9, 9t serum, in acid–base evaluation, 8–9, 9f, 9t, 10t urine, osmolal gap and, in acid–base evaluation, 10–11 Arterial blood gas (ABG) in acid–base evaluation, 8 practical approach to, 35–39, 36f, 36t, 38f

B B⫺. See Buffering anion (B⫺) Bartter’s syndrome, metabolic alkalosis vs., 46–47 Base excess, in acid—base evaluation, 11 Bicarbonate ([HCO3⫺]), normal serum values for, 1-6, 2t, 5f, 6f Bicarbonate [HCO3⫺], gastrointestinal loss of, metabolic acidosis vs., 43–44, 44t Buffer system, transporting acid and mitigating pH changes and, 3f, 4 Buffering anion (B⫺), 3f, 4

C Carbonic anhydrase inhibitors, ingestion of, acetazolamide and, metabolic acidosis vs., 46 CO2, elimination of, lungs in, 3f, 4 Compensation basics of, 20–27 defined, 1, 2t, 20 degrees of, prediction of, 21–26, 22f–25f, 25t, 26t metabolic, 20 for metabolic acidosis, 21–22, 22f for metabolic alkalosis, 22–23, 23f renal, in respiratory alkalosis, 18, 18t respiratory, 17, 20 for respiratory acidosis, 24–25, 24f, 25t for respiratory alkalosis, 25–26, 25f, 26t Contraction alkalosis, 16t, 17 Cushing’s syndrome, metabolic alkalosis vs., 48t

D “Delta-delta” calculation, in mixed metabolic acid–base disorders, 32–33, 32f Diabetic ketoacidosis (DKA), metabolic acidosis vs., 41t, 42 Diarrhea, metabolic acidosis vs., 44t DKA. See Diabetic ketoacidosis (DKA)

Longenecker_Index_086-088.qxd

10/17/06

12:28 PM

Page 87

INDEX

E Ethanol, metabolic acidosis and, 41t Ethylene glycol poisoning, metabolic acidosis vs., 40–41, 41t Exogenous steroids, metabolic alkalosis vs., 48t

F Fanconi’s syndrome, 45

G GFR. See Glomerular filtration rate (GFR) Glomerular filtration rate (GFR), decreased, alkali ingestion with, in “saline–non-responsive” metabolic alkalosis, 48, 48t

H [H⫹], normal serum values for, 1, 2t [HCO3⫺]. See Bicarbonate ([HCO3⫺]) Henderson-Hasselbalch equation, 12, 13f described, 20 β-Hydroxylase deficiency, metabolic alkalosis vs., 48t Hyperaldosteronism, metabolic alkalosis vs., 48t Hypercapnia, defined, 4 Hyperchloremic acidosis, 14 Hyperventilation, defined, 4 Hypocapnia, defined, 4 Hypokalemia, metabolic alkalosis and, 48 Hypoventilation, defined, 4

I Ingestion(s) alkali, with decreased GFR, in “saline–non-responsive” metabolic alkalosis, 48, 48t licorice, metabolic alkalosis vs., 48t Intracellular acid and base production, 2–4, 3f

K

87

elevated-gap acute respiratory acidosis and, case example, 67–68 case example, 52–53 differential diagnosis of, 40–43, 41t respiratory alkalosis and, case examples, 62–63, 76–78 normal-gap case example, 54–55 differential diagnosis of, 43–46, 44t renal response to, 12–13, 13t respiratory compensation for, 21–22, 22f respiratory response to, 13–14 types of, 14–15, 14f Metabolic alkalosis, 15–17, 15f, 16t acute respiratory acidosis and, case example, 68–70 “acute-on-chronic” respiratory acidosis and, case example, 73–76 case example, 53–54 causes of, 15, 47 differential diagnosis of, 46–48, 47t, 48t Bartter’s syndrome, 46–47 posthypercapnia, 46–47 volume depletion vs., 46 hypokalemia and, 48 renal response to, 15–17, 15f, 16t respiratory compensation for, 22–23, 23f respiratory response to, 17 “saline–non-responsive,” differential diagnosis of, 47–48, 48t “saline-responsive” (chlorine-responsive), differential diagnosis of, 46–47, 47t Metabolic compensation, 20 Metabolic component, elimination of nonvolatile acids, 3f, 4–6, 5f, 6f Metabolic processes, defined, 1 Methyl poisoning, metabolic acidosis vs., 40–41, 41t Mineralocorticoid(s), excess of, syndromes of, metabolic alkalosis vs., 48t Mixed acid–base disorders, 28–34, 29f, 29t–31t, 32f basics of, 28, 29f mixed metabolic acid–base disorders, identification of, 30t, 31–33, 32f patterns of, 29, 29t–31t triple acid–base disorders, identification of, 33–34 Mixed disorder, defined, 1, 2t Mixed metabolic acid–base disorders “delta-delta” calculation in, 32–33, 32f identification of, 30t, 31–33, 32f types of, 30t, 31

Ketoacidosis, diabetic, metabolic acidosis vs., 41t, 42

N L Lactic acidosis, metabolic acidosis vs., 41t, 42–43 Licorice ingestion, metabolic alkalosis vs., 48t Lung(s), in CO2 elimination, 3f, 4

Net nonvolatile acid production, 2–4, 3f Nonvolatile acids, 2, 3f elimination of, 3f, 4–6, 5f, 6f

O M Metabolic acidosis, 12–15, 13f–15f, 13t causes of, 12 differential diagnosis of, 40–46, 41t, 44t acetazolamide and ingestion of carbonic anhydrase inhibitors, 46 acid ingestion or infusion, 46 acidosis caused by renal tubular dysfunction, 44–45, 44t AKA, 41–42, 41t diarrhea, 44t DKA, 41t, 42 gastrointestinal loss of HCO3⫺, 43–44, 44t lactic acidosis, 41t, 42–43 methanol and ethylene glycol poisoning, 40–41, 41t posthypocapnia, 46 RTA, 44–46, 44t salicylate poisoning, 41t, 43 uremia, 41t, 42

Osmolal gap, urine anion gap and, in acid–base evaluation, 10–11

P Paraldehyde, metabolic acidosis vs., 41t pCO2, normal serum values for, 1, 2t pH defined, 1 mitigating changes in, transporting acid and, 3f, 4 normal serum values for, 1, 2t urine, in acid–base evaluation, 10 Poisoning ethylene glycol, metabolic acidosis vs., 40–41, 41t methyl, metabolic acidosis vs., 40–41, 41t salicylate, metabolic acidosis vs., 41t, 43 Posthypercapnia, in “saline-responsive” metabolic alkalosis, 46–47 Posthypocapnia, metabolic acidosis vs., 46

Longenecker_Index_086-088.qxd

88

10/11/06

5:02 PM

Page 88

INDEX

R Renal compensation, in respiratory alkalosis, 18, 18t Renal tubular dysfunction, acidosis caused by, metabolic acidosis vs., 44–45, 44t Respiratory acidosis acute elevated-gap metabolic acidosis and, case example, 67–68 metabolic alkalosis and, case example, 68–70 “acute-on-chronic,” metabolic alkalosis and, case example, 73–76 chronic acute and, case example, 65–67 case example, 63–65 compensation for, 24–25, 24f, 25t defined, 2t differential diagnosis of, 49–50, 49t, 50f Respiratory alkalosis, 17–18, 18f, 18t acute, case example, 55–57 buffer response to, 18, 18f case examples, 58–62 causes of, 17 chronic, case example, 57–58 compensation for, 25–26, 25f, 26t defined, 2t differential diagnosis of, 50t, 51 elevated-gap metabolic acidosis and, case examples, 62–63, 76–78 renal response to, 18, 18t Respiratory compensation, 17, 20

Respiratory component, elimination of CO2 by lungs, 3f, 4 Respiratory processes, defined, 1

S Salicylate poisoning, metabolic acidosis vs., 41t, 43 Serum anion gap, in acid–base evaluation, 8–9, 9f, 9t, 10t Serum osmolal gap, in acid–base evaluation, 9–10 Steroid(s), exogenous, metabolic alkalosis vs., 48t Syndromes of mineralocorticoid excess, metabolic alkalosis vs., 48

T Triple acid–base disorder case examples, 70–73 identification of, 33–34

U Uremia, metabolic acidosis vs., 41t, 42 Urine anion gap, osmolal gap and, in acid–base evaluation, 10–11

V Ventilation, alveolar, 3f, 4 Volume depletion, in “saline-responsive” metabolic alkalosis, 46